Government Accounting Missed Questions Flashcards

1
Q

A city government reported a $9,000 increase in net position in the motor pool internal service fund, a $12,000 increase in net position in the water enterprise fund, and a $7,000 increase in the employee pension fund. The motor pool internal service fund provides service primarily to the police department. What amount should the city report as the change in net position for business-type activities in its statement of activities?

A.
$9,000
B.
$12,000
C.
$21,000
D.
$28,000
A

The correct answer is (B).

Business type activities that are normally financed through user charges and reports the consolidated results of all enterprise funds. It does not include internal service funds, which are accounted for in governmental activities. The employee pension fund is a fiduciary fund and not included in business-type activities. The $12,000 increase in net position from Water enterprise is included.

(A) is incorrect because the motor pool internal service fund is excluded from business-type activities.

(C) is incorrect because the calculation also includes motor pool internal service fund which should be excluded.

(D) is incorrect because the motor pool Internal service fund and employee pension trust fund should be excluded from business-type activities.

How well did you know this?
1
Not at all
2
3
4
5
Perfectly
2
Q

Oak County incurred the following expenditures in issuing long-term bonds:

Issue cost $ 400,000
Debt insurance 90,000
When Oak establishes the accounting for operating debt service, what amount should be deferred and amortized over the life of the bonds?

A.$90,000
B.$0
C.$400,000
D.$490,000

A

The correct answer is (B).

The general long-term debt of a state or local government is reported:

Government-wide financial statements: Accrual basis of accounting.
Governmental Funds (Debt Service Funds): Modified Accrual basis of accounting.
In Government Funds (Debt Service Funds) under Modified Accrual basis of accounting costs are recognized using the expenditure principle, and recorded in the period the obligation to pay arises, irrespective of when the goods/services are being actually used by the government entity.

Accordingly, the modified accrual basis does not allow for amortization of issue costs and debt insurance premiums, which must be recognized in full when the liability is incurred.

The amount deferred and amortized over the life of the bonds is $0.

$490,000 is expensed in the period incurred.

How well did you know this?
1
Not at all
2
3
4
5
Perfectly
3
Q

Users need information on how the government applies restricted resources when they have a choice between using restricted or unrestricted resources to evaluate which of the following?

A.Whether the government made improper payments from the restricted and unrestricted resources
B.Whether the government complied with laws and regulations associated with federal funds
C.How ending balances may be applied to continuation of programs and services in future periods
D.None of the above

A

The correct answer is: C

Explanation:
Governments rely on grants and other types of contributions to fund a variety of programs. The note disclosing how the government applies restricted resources when they have a choice between using restricted versus unrestricted resources provides users with information to evaluate how ending balances may be applied to continuation of programs and services in future periods (not whether the government made improper payments or whether the government complied with laws and regulations).

How well did you know this?
1
Not at all
2
3
4
5
Perfectly
4
Q

On January 1, Fonk City approved the following general fund resources for the new fiscal period:

Property taxes	$ 5,000,000
Licenses and permits	400,000
Intergovernmental revenues	150,000
Transfers in from other funds	350,000
What amount should Fonk record as estimated revenues for the new fiscal year?

A.$5,400,000
B.$5,550,000
C.$5,750,000
D.$5,900,000

A

The correct answer is: B
Explanation:
The correct answer is B. Estimated revenues are revenues that are expected to be available to be spent in the period.

Ref Summary Amount

a Property taxes $5,000,000

b Licenses & Permits $400,000

c Inter-governmental revenues $150,000

d Estimated Revenue (a+b+c) $5,550,000

Transfers in from other funds are not considered because it is an estimated other financing source, not revenue. Inter-governmental revenue is included in the calculation of estimated revenue. Transfers in from other funds of $350,000 are not included in the calculation for estimated revenue.

How well did you know this?
1
Not at all
2
3
4
5
Perfectly
5
Q

Which of the following is included in the governmental fund statements?

A.Cash flows statements
B.Component unit information
C.Reconciliation between the fund statements and the government-wide statements
D.Major fiduciary funds

A

The correct answer is: C
Explanation:
Governmental fund financial statements present a reconciliation to the government-wide statements. Cash flows statements appear only in the fund statements for proprietary funds. Individual component unit information is presented in individual columns in the government-wide statements, after the primary government’s fund statements, or in the notes to the financial statements. Governmental fund financial statements do not include major fiduciary funds. Even fiduciary fund statements present fund types, not major funds.

How well did you know this?
1
Not at all
2
3
4
5
Perfectly
6
Q

One feature of state and local government accounting and financial reporting is that fixed assets used for general government activities

A.
Often are not expected to contribute to the generation of revenues.
B.
Do not depreciate as a result of such use.
C.
Are acquired only when direct contribution to revenues is expected.
D.
Should not be maintained at the same level as those of businesses so that current financial resources can be used for other government services.

A

The correct answer is: A
Explanation:
Governmental entities invest large amounts of resources in non-revenue producing capital assets such as government office buildings, highways, bridges, and sidewalks.

How well did you know this?
1
Not at all
2
3
4
5
Perfectly
7
Q

A government’s assets include inventory of $2 million, roads constructed for $25 million with accumulated depreciation of $10 million, and equipment acquired for $5 million with accumulated depreciation of $1 million. Its liabilities include an outstanding balance of $5 million for bonds payable issued to construct the roads and a $1 million short-term loan for inventory purchases. What amount should be reported as the net investment in capital assets in the government-wide statement of net position?

A.$26 million
B.$25 million
C.$14 million
D.$10 million

A

The correct answer is: C
Explanation:
The correct answer is (C).

$14,000,000 should be reported as the net investment in capital assets in the government-wide statement of net position. Net assets on the government-wide statement of financial position are divided into - Restricted, Unrestricted and Net Investment in Capital Assets.

Assets:
Roads Constructed $25,000,000
Accum. Depreciation ($10,000,000) $15,000,000
Equipment $5,000,000
Accum. Depreciation ($1,000,000) $4,000,000

Liabilities:
Bonds Payable $(5,000,000)
*Net Investment in Capital Assets $14,000,000

Note: Inventory is not a capital asset and therefore both the inventory and liability related to the purchase of inventory is also excluded.

How well did you know this?
1
Not at all
2
3
4
5
Perfectly
8
Q

Which basis of accounting is required for a city’s government-wide financial statements?

A.
Cash.
B.
Modified Cash.
C.
Modified Accrual.
D.
Accrual.
A

The correct answer is: D
Explanation:
A city’s government-wide financial statements are required to be completed on the accrual basis of accounting.

How well did you know this?
1
Not at all
2
3
4
5
Perfectly
9
Q

Except in specified circumstances, interest costs on long-term liabilities are reported under governmental accounting rules in which of the following manners?

A.
Separately as a direct program cost
B.
Separately as an indirect cost
C.
In a particular note to the financial statements
D.
None of the above
A

The correct answer is: B
Explanation:
In general, interest costs on long-term liabilities will be reported separately as an indirect cost (not in a particular note). Note disclosures will include information about how interest costs are reported and what amounts are included as direct expenses.

How well did you know this?
1
Not at all
2
3
4
5
Perfectly
10
Q

GASB Statement No. 34, Basic Financial Statements and Management’s Discussion and Analysis for State and Local Governments, requires large governments (annual revenues in the range above $100 million) to

Report all fixed assets (land, buildings, and equipment).
In fiscal periods beginning after June 15, 2001, report major general infrastructure asset acquisitions.
In fiscal periods beginning after June 15, 2005, retroactively report major general infrastructure assets acquired in fiscal years ending after June 30, 1980, and before adoption of GASB Statement No. 34.
A.
I only
B.
I and II only
C.
I and III only
D.
I, II, and III

A

The correct answer is: D

How well did you know this?
1
Not at all
2
3
4
5
Perfectly
11
Q

A city received a $9,000,000 federal grant to finance the construction of a homeless shelter. In which fund should the proceeds be recorded?

A.
Permanent
B.
General
C.
Capital projects
D.
Special revenue
A

Explanation:
The correct answer is (C).

The $900,000 the city received as a federal grant to finance the construction of a homeless shelter should be recorded in the capital projects fund. Capital projects fund accounts for major acquisition or construction activities of capital assets. A capital projects fund is used to establish resources to construct or acquire major, long-lived general government facilities. The construction of a homeless shelter via a federal grant is a perfect example.

How well did you know this?
1
Not at all
2
3
4
5
Perfectly
12
Q

Which of the following factors influences governmental generally accepted accounting and reporting principles?

A.The lack of SEC oversight for municipal financial instruments
B.State statutes that created the Governmental Accounting Standards Board (GASB)
C.Governmental structure and its mission to provide critical public services
D.Population levels for individual governments

A

The correct answer is: C
Explanation:
The essential mission of government is delivering services, financed with taxes. The SEC does have limited oversight for tax-exempt financial instruments, but the GASB is focused on the operating and organizational characteristics to define governmental accounting and reporting standards. The GASB was created by the Financial Accounting Foundation and is not subject to any individual state regulation. While the GASB has sometimes considered “small GAAP” versus “big GAAP,” the size of individual state and local governments is not a key factor for the Conceptual Framework.

How well did you know this?
1
Not at all
2
3
4
5
Perfectly
13
Q

Which fund would report the amount of bonds payable that are due within one year?

A.
Governmental funds
B.
Proprietary funds
C.
Both governmental and proprietary funds
D.
Neither governmental nor proprietary funds
A

The correct answer is: B
Explanation:
Proprietary funds report the current amounts due for any bonds outstanding on its statement of net position. At the fund level, governmental funds do not report the current portions of principal or interest due.

How well did you know this?
1
Not at all
2
3
4
5
Perfectly
14
Q

Several disclosures in the Significant Accounting Policies section of governmental financial statements describe which of the following?

A.
How the government complied with applicable laws and regulations
B.
Why the government exceeded its statutory budgetary authority
C.
Accounting policies for recognizing revenues and expenditures
D.
Why the notes are not necessary for a complete understanding of the government’s financial statements

A

The correct answer is: C
Explanation:
The Significant Accounting Policies section includes a number of explanations about what policies the government has adopted for recognition, such as how revenues are recognized, asset capitalization, program revenues, and the government’s definition of operating transactions. Disclosures do not describe how the government complied with applicable laws and regulations, why the government exceeded its statutory budgetary authority, or why the notes are necessary.

How well did you know this?
1
Not at all
2
3
4
5
Perfectly
15
Q

Which of the following indicates why fiduciary funds are presented in the fund statements instead of the note disclosures?

A.
To enhance municipal bond disclosures
B.
To improve budgetary compliance
C.
To provide accountability for these resources
D.
To comply with federal grant reporting requirements
A

The correct answer is: C
Explanation:
The GASB considered using note disclosures to present fiduciary funds, but decided that financial statement presentations would enhance stewardship and accountability for these resources. Municipal bonds are not secured with fiduciary fund resources since these assets are not available for the government’s programs. Fiduciary funds are generally not required to adopt budgets. Federal grants are not generally classified as fiduciary funds.

How well did you know this?
1
Not at all
2
3
4
5
Perfectly
16
Q

Central County received proceeds from various towns and cities for capital projects financed by Central’s long-term debt. A special tax was assessed by each local government, and a portion of the tax was restricted to repay the long-term debt of Central’s capital projects. Central should account for the restricted portion of the special tax in which of the following funds?

A.
Internal service fund
B.
Enterprise fund
C.
Capital projects fund
D.
Debt service fund
A

Explanation:
The correct answer is (D).

Debt service funds account for the accumulation of resources for, and the payment of, general long-term debt principal and interest.

Therefore, the portion of the special tax that is restricted to repay the long-term debt of the government’s capital projects fund should be accounted for in a debt service fund.

Internal service funds are used to account for the activities of a department that provides services to other departments of the same government in a manner similar to a business. Internal service funds (a type of proprietary fund) account for only their own assets, liabilities, revenues, and expenses.

Enterprise funds are used to account for the activities of a department that provides services to others in a manner similar to a business. Enterprise funds (a type of proprietary fund) account for only their own assets, liabilities, revenues, and expenses.

Capital project funds are used to account for the construction of capital projects.

How well did you know this?
1
Not at all
2
3
4
5
Perfectly
17
Q

Which event(s) is(are) supportive of interperiod equity as a financial reporting objective of a governmental unit?

A balanced budget is adopted.
Residual equity transfers out equals residual transfers in.
A.
I only
B.
II only
C.
Both I and II
D.
Neither I nor II
A

The correct answer is: A
Explanation:
Interperiod equity is the idea of using the revenues from one period to pay for the expenditures of that same period and that period only. A balanced budget is a plan for this occurrence. Residual equity transfers are between funds, not between periods.

How well did you know this?
1
Not at all
2
3
4
5
Perfectly
18
Q

Which of the following characteristics describe the financial resources for capital projects funds?

A.
Revenues based on rates that will recover the cost of capital assets required to deliver services
B.
Bond issue proceeds, grants, or interest earnings
C.
Donations restricted, such that only investment earnings can be used for maintenance
D.
Prior year encumbrances

A

The correct answer is: B
Explanation:
Resources for capital project funds may arise from bond issue proceeds, grants, or interest earnings. Revenues based on rates that will recover the costs of capital assets required to deliver services describes the approach used in enterprise fund accounting. Donations restricted such that only investment earnings can be used for maintenance would be accounted for in a permanent fund. The encumbrance system is used in governmental funds (general, special revenue, and capital projects funds) to prevent over-expenditure and to demonstrate compliance with legal requirements. Encumbrances are a budgetary measurement element, not a criterion for determining fund type.

How well did you know this?
1
Not at all
2
3
4
5
Perfectly
19
Q

Which of the following should be included in financial reports to help users assess the government’s oper­ating results?

A.
Cash flow statements
B.
Other supplementary information
C.
Management discussion and analysis
D.
Explanation of the sources and uses of financial resources
A

The correct answer is: D
Explanation:
The second reporting objective discussed in Concepts Statement No. 1 suggests that governmental financial reporting should assist users in evaluating the government’s operating results by explaining sources and uses of financial resources. Cash flow statements are a specific element in proprietary fund statements and are not discussed in Concepts Statement No. 1. Other supplementary information is useful for enhancing financial statements, but is not necessarily required. The management discussion and analysis is one of the components of required supplementary information, but is not discussed in Concepts Statement No. 1.

How well did you know this?
1
Not at all
2
3
4
5
Perfectly
20
Q

Which of the following would be reported using the full accrual method of accounting?

A.
Governmental activities
B.
Business-type activities
C.
Neither governmental activities nor business-type activities
D.
Both governmental activities and business-type activities
A

The correct answer is: D
Explanation:
Both governmental activities and business-type activities are included on the face of the government-wide financial statements. The government-wide financial statements use the full accrual method of accounting, which means that D is the correct answer. All amounts on the government-wide financial statements are reported using the accrual method.

How well did you know this?
1
Not at all
2
3
4
5
Perfectly
21
Q

Lake County received the following proceeds that are legally restricted to expenditure for specified purposes:

Levies on affected property owners to install sidewalks $500,000
Gasoline taxes to finance road repairs 900,000
What amount should be accounted for in Lake’s special revenue funds?

A.
$1,400,000
B.
$900,000
C.
$500,000
D.
$0
A

The correct answer is: B
Explanation:
Special revenue fund sources includes fees, grants, specific taxes, and other earmarked revenue sources. The gasoline taxes to finance road repairs for $900,000 would be a specific tax, accounted under special revenue funds. Levies on affected property owners to install sidewalks is usually accounted for under the agency fund. It may also be accounted for in a debt service fund, for example: the levies are used to repay a bond that was issued to install the sidewalks. Options (a), (c) and (d) are incorrect based on the above explanation.

How well did you know this?
1
Not at all
2
3
4
5
Perfectly
22
Q

All of the following statements regarding notes to the basic financial statements of governmental entities are true except

A.
The notes contain disclosures related to required supplementary information.
B.
Some notes presented by governments are identical to notes presented in business financial statements.
C.
Notes that are considered essential to the basic financial statements need to be presented.
D.
It is acceptable to present notes in a very extensive format.

A

The correct answer is: A
Explanation:
The notes to the basic financial statements of governmental entities do not contain disclosures related to required supplementary information. The Management’s Discussion and Analysis (MD&A) is the required supplementary information in the government’s general purpose external report and it is presented before the financial statements. .

How well did you know this?
1
Not at all
2
3
4
5
Perfectly
23
Q

Fixed assets of an enterprise fund should be accounted for in the

A.
Enterprise fund but no depreciation on the fixed assets should be recorded.
B.
Enterprise fund and depreciation on the fixed assets should be recorded.
C.
General fixed asset account group and depreciation on the fixed assets should be recorded.
D.
General fixed asset account group but no depreciation on the fixed assets should be recorded.

A

The correct answer is: B
Explanation:
The enterprise fund is a self-supporting fund which provides goods and/or services to the general public. Revenues and expenses are recorded in the same manner as in commercial business enterprises. Fixed assets are recorded in the fund as well as the associated depreciation charges.

How well did you know this?
1
Not at all
2
3
4
5
Perfectly
24
Q

Maple City’s public employee retirement system (PERS) reported the following account balances at June 30:

Reserve for employer’s contributions $5,000,000
Actuarial deficiency in reserve for employer’s contributions 300,000
Reserve for employees’ contributions 9,000,000
Maple’s PERS fund balance at June 30, should be
A.
$ 5,000,000
B.
$ 5,300,000
C.
$14,000,000
D.
$14,300,000

A

The correct answer is: D
Explanation:
The PERS should be accounted for in a pension trust fund. Under the traditional approach of accounting for pension trust funds under NCGA Statement 1, the $14,300,000 fund balance of the pension trust fund can be determined by adding the reserve for employee contributions (i.e., $9,000,000), the reserve for employer contributions (i.e., $5,000,000), and the actuarial deficiency in reserve for employer’s contributions (i.e., $300,000).

How well did you know this?
1
Not at all
2
3
4
5
Perfectly
25
Q

Gold County received goods that had been approved for purchase but for which payment had not yet been made. Should the accounts listed below be increased?

Encumbrances	Expenditures
A	No	No
B	No	Yes
C	Yes	No
D	Yes	Yes
A

The correct answer is: B
Explanation:
The following entries would be made by a governmental unit when ordering and receiving goods:

To record purchase order issued when goods were approved for purchase:
Encumbrances (expected cost) XX
Reserve for encumbrances XX
To record expenditure upon receipt of the goods:
Reserve for encumbrances XX
Encumbrances XX
Expenditures (actual) XX
Vouchers payable XX

Thus, upon receipt of the goods, the Encumbrance account decreases and the Expenditures account increases.

How well did you know this?
1
Not at all
2
3
4
5
Perfectly
26
Q

What is the measurement focus and the basis of accounting for the government-wide financial statements?

Measurement focus	Basis of accounting
A	Current financial resources	Modified accrual
B	Economic resources	Modified accrual
C	Current financial resources	Accrual
D	Economic resources	Accrual
A

The correct answer is: D
Explanation:
Government-wide financial statements aggregate information for all governmental and business-type activities. GASB 34 requires an economic resources measurement focus and accrual basis of accounting for all amounts in the government-wide financial statements.

How well did you know this?
1
Not at all
2
3
4
5
Perfectly
27
Q

Which of the following indicates when the fund balance may not be reported as assigned?

A.
If the assignment causes a deficit in unassigned fund balance
B.
In Capital Projects funds
C.
If there is no component for committed fund balance
D.
In the General Fund
A

The correct answer is: A
Explanation:
The correct answer is (A)

Assigned fund balance is based on management’s intent, but cannot create a deficit in unassigned fund balance. Assigned fund balance cannot set aside resources that are not available. Assigned fund balance may be reported in any governmental fund. Residual balances in special revenue funds may be classified as assigned since management has created these funds for specific purposes. There is no requirement for a committed fund balance before assignments may be reported.

How well did you know this?
1
Not at all
2
3
4
5
Perfectly
28
Q

Kingwood Town paid $22,000 cash for a flatbed trailer to be used in the general operations of the town. The expected useful life of the trailer is 6 years with an estimated $7,000 salvage value. Which of the following amounts would be reported?

A.
$15,000 increase in equipment in the general fund.
B.
$15,000 increase in investment in governmental activities column for fixed assets.
C.
$22,000 increase in investment in governmental activities column for fixed assets.
D.
$22,000 increase in equipment in the general fund.

A

The correct answer is: C
Explanation:
Purchased assets are reported in the governmental activities column at cost ($22,000). Long-term (capital) assets are not recorded in any governmental fund.

How well did you know this?
1
Not at all
2
3
4
5
Perfectly
29
Q

Which of the following characteristics describe a multiple plan management of a PERS?

A.
Assets accumulated for benefit payments are available to pay benefits for any of the plan members.
B.
Assets are accumulated in separate reserves restricted for benefits of specific classes or groups of employees.
C.
The government serves as a custodian for the accumulated assets and distributes funds based on instructions from member governments.
D.
Assets are not accessible to creditors of member governments.

A

The correct answer is: B
Explanation:
The PERS is responsible for multiple plans when assets are accumulated in separate reserves restricted for benefits of specific classes or groups of employees. Assets being accumulated for benefit payments available to pay benefits for any of the plan members is a characteristic of the single plan. The government serving as a custodian for the accumulated assets and distributing funds based on instructions from member governments describes an agency fund. Creditor access is not a distinguishing feature for pension trust plans.

How well did you know this?
1
Not at all
2
3
4
5
Perfectly
30
Q

Which of the following statements is the most significant characteristic in determining the classification of an enterprise fund?

A.
The predominant customer is the primary government.
B.
The pricing policies of the activity establish fees and charges designed to recover its cost.
C.
The activity is financed by debt that is secured partially by a pledge of the net revenues from fees and charges of the activity.
D.
Laws or regulations require that the activity’s costs of providing services including capital costs be recovered with taxes or similar revenues.

A

The correct answer is: B
Explanation:
Enterprise funds must be used to account for a government’s business-type operations that are financed and operated like private businesses—where the government’s intent is that all costs (expenses, including depreciation) of providing goods or services to the general public on a continuing basis are to be financed or recovered primarily through user charges (operating revenue). Most government-owned public utilities (e.g., electricity, gas, water, and sewage systems) must be accounted for as enterprise funds.

How well did you know this?
1
Not at all
2
3
4
5
Perfectly
31
Q

Which of the following indicates when the governmental fund balance may be reported as committed?

A.
Constraints are imposed by external parties through grants or contracts
B.
Constraints are imposed by formal action of the government’s governing body
C.
Amounts are set aside for specific purposes based on management’s intent
D.
Amounts have not been restricted or assigned

A

The correct answer is: B
Explanation:
Committed fund balances reflect amounts that can only be used for specific purposes based on constraints imposed by formal action of the government’s highest level of decision-making authority. Constraints imposed by external parties through grants or contracts are reported as restricted fund balance, not committed. Amounts set aside for specific purposes based on management’s intent are reported as assigned fund balance, not committed. A restriction or assignment is not a prerequisite for fund balance classifications.

How well did you know this?
1
Not at all
2
3
4
5
Perfectly
32
Q

A county’s balances in the general fund included the following:

Appropriations	$435,000
Encumbrances	18,000
Expenditures	164,000
Vouchers payable	23,000
What is the remaining amount available for use by the county?
A.
$230,000
B.
$248,000
C.
$253,000
D.
$271,000
A

The correct answer is: C
Explanation:
Funds available for use = Appropriations (budgeted) - Encumbrances - Expenditures ( $435,000 - $18,000 - $164,000 ) = $253,000. Option (a) is incorrect because vouchers payable is excluded in available for funds calculations. Option (b) is incorrect because encumbrances are not excluded, instead vouchers payable are excluded from appropriations. Option (d) is incorrect because encumbrances should be excluded from appropriations to arrive at funds available.

How well did you know this?
1
Not at all
2
3
4
5
Perfectly
33
Q

During the current year, Vann County’s motor pool internal service fund sold two vehicles for $5,000. The vehicles had a cost of $6,000 and a carrying value of $4,000. How should Vann County’s motor pool internal service fund report this transaction in its fund financial statements?

A.
Revenue of $5,000
B.
Other financing source of $5,000
C.
Special item of $1,000
D.
Gain of $1,000
A

The correct answer is: D
Explanation:
The correct answer is (D).

The motor pool internal service fund is reported under the proprietary fund in the fund financial statements. The proprietary fund uses the accrual basis of accounting which records operating revenues, operating expenses and non-operating revenues and non-operating expenses to get the change in net position. Vann County’s motor pool internal service sold two vehicles for $5,000. Both vehicles together have a cost of $6,000 and a carrying value of $4,000. Because the carrying value was $4,000 and the vehicles sold for $5,000, Vann County’s motor pool internal service fund should report a gain of $1,000.

How well did you know this?
1
Not at all
2
3
4
5
Perfectly
34
Q

A special-purpose government is distinguished from a general purpose government by which of the following?

A.
It is established by a state government.
B.
It provides limited services, such as only governmental or business-type activities.
C.
It does not manage any fiduciary funds.
D.
It is not governed by a separately elected legislative body.
A

The correct answer is: B
Explanation:
The distinction between general purpose and special-purpose governments is based on the entity’s programs and services, not how the government was created. Special-purpose governments only provide limited services; unlike cities, counties, states, towns or villages that provide a broad range of services. It is possible that a special purpose government may only manage fiduciary programs. Governance structure is not the distinguishing feature of special-purpose governments.

How well did you know this?
1
Not at all
2
3
4
5
Perfectly
35
Q

In the General Fund Statement of Revenues, Expenditures, and Changes in Fund Balances, which of the following has an effect on the excess of revenues over expenditures?

A.
Purchase of fixed assets
B.
Payment to a debt-service fund
C.
Special items
D.
Proceeds from the sale of capital assets
A

Explanation:
The correct answer is (A).

The General Fund uses modified accrual accounting, in which the purchase of a fixed asset is an expenditure, which would affect the excess of revenue over expenditure.

Option (B) is incorrect because transfers out are reported as other financing uses which are presented below the excess of revenues (deficiency) over expenditure.

Option (C) is incorrect because special items are presented below the excess of revenues (deficiency) over expenditure.

Option (D) is incorrect because proceeds from the sale are reported in special items, which is presented below the excess of revenues (deficiency) over expenditure.

How well did you know this?
1
Not at all
2
3
4
5
Perfectly
36
Q

Which of the following statements about the statistical section of the Comprehensive Annual Financial Report (CAFR) of a governmental unit is true?

A.
Statistical tables may not cover more than two fiscal years.
B.
Statistical tables may not include nonaccounting information.
C.
The statistical section is not part of the basic financial statements.
D.
The statistical section is an integral part of the basic financial statements.

A

The correct answer is: C
Explanation:
The basic financial statements include both the government-wide statements and the fund statements, as well as the notes, but not the statistical section.

How well did you know this?
1
Not at all
2
3
4
5
Perfectly
37
Q

Which of the following is considered part of the required supplementary information for general-purpose external financial reporting of a local government?

A.
Fund financial statement.
B.
Combining non-major fund statement.
C.
Notes to the financial statements.
D.
Management discussion and analysis.
A

Explanation:
The correct answer is (D).

Required Supplementary Information is the information that is mandated to be disclosed by the reporting entity. This information, though not a part of the financial statements, are considered a prerequisite for a complete understanding and overview of financial information.

One such information is Management Discussion and Analysis (MD&A), which is the assessment of the organization’s performance for the said period by the management. MD&A are a compulsory part of Required Supplementary Information for general purpose external financial reporting of a local government.

How well did you know this?
1
Not at all
2
3
4
5
Perfectly
38
Q

Smith City levied property taxes totaling $1,000,000 during the current year, of which $150,000 was collected within 60 days after year-end. What amount should Smith City record as property tax revenue?

A.
$850,000
B.
$1,000,000
C.
$150,000
D.
$0
A

The correct answer is: B
Explanation:
The correct answer is (B).

Property taxes are considered the enforceable legal claim and is always recognized when billed. A government can expect to collect the amount billed to taxpayers once a property tax assessment is made as these taxes are mandatory.

As a result, property taxes that have been billed usually satisfy the requirements of being measurable and available, and are accrued in advance of collection. However, an allowance for uncollectible must be established.

Dr. Property Taxes Receivable-Current $1,000,000
Cr. Property Tax Revenue $1,000,000

How well did you know this?
1
Not at all
2
3
4
5
Perfectly
39
Q

Which of the following items is an example of imposed nonexchange revenue for a governmental entity?

A.
Personal income taxes
B.
Retail sales tax
C.
Federal grant money
D.
Property taxes
A

The correct answer is: D
Explanation:
Imposed nonexchange revenues represent assessments imposed on non-governmental entities and include property taxes and fines or forfeitures. Taxes on earnings or consumption are classified as derived tax revenues. Certain grants, entitlements, and donations are classified as voluntary nonexchange transactions. Voluntary nonexchange transactions result from legislative or contractual agreements involving parties that may or may not be governmental entities, but do not involve an exchange of equal value.

How well did you know this?
1
Not at all
2
3
4
5
Perfectly
40
Q

If a city government is the primary reporting entity, which of the following is an acceptable method to present component units in its combined financial
statements?

A.
Consolidation.
B.
Cost method.
C.
Discrete presentation.
D.
Government-wide presentation.
A

The correct answer is: C
Explanation:
Government-wide statements (GWS) aggregate information for all governmental and business-type activities. GASB 34 requires an economic resources
measurement focus and accrual basis of accounting for all amounts in the GWS. There are four required columns in the GWS, one each for: governmental
activities, business-type activities, the primary government (sum of the previous two), and component units. Most component units should be included in the
financial reporting entity by discrete presentation (reported in columns separate from primary government).

How well did you know this?
1
Not at all
2
3
4
5
Perfectly
41
Q

The debt service fund of a governmental unit is used to account for the accumulation of resources for, and the payment of, principal and interest in connection with a

Private-purpose trust fund	Proprietary funds
A	No	No
B	No	Yes
C	Yes	Yes
D	Yes	No
A

The correct answer is: A
Explanation:
Only general obligation long-term debt should be serviced through debt service funds. Fiduciary and proprietary fund debt are rarely general government obligations.

How well did you know this?
1
Not at all
2
3
4
5
Perfectly
42
Q

The mission of the Governmental Accounting Standards Board (GASB) includes which of the following?

A.
Providing a framework to educate users and support public policies, invest-ments, and management by those users
B.
Establishing standards to identify appropriate performance measures and criteria
C.
Outlining best practices for administration of governmental budgets
D.
Providing accounting and reporting standards for federal government agencies

A

The correct answer is: A
Explanation:
The GASB’s mission includes research and development of standards that will improve accounting and financial reporting for state and local governments. The GASB’s mission includes providing a framework to educate users and support public policies, investments, and management by those users.

How well did you know this?
1
Not at all
2
3
4
5
Perfectly
43
Q

Fund accounting is used by governmental units with resources that must be

A.
Composed of cash or cash equivalents.
B.
Incorporated into combined or combining financial statements.
C.
Segregated for the purpose of carrying on specific activities or attaining certain objectives.
D.
Segregated physically according to various objectives.

A

The correct answer is: C
Explanation:
Fund accounting is used when there are legal separations between sources and uses of funds. Fund accounting may be used to account for assets aside from cash and cash equivalents. Combined or combining financial statements may be used with or without fund accounting. The resources may be physically in the same account or location.

How well did you know this?
1
Not at all
2
3
4
5
Perfectly
44
Q

Which of the following is the paramount objective of financial reporting by state and local governments?

A.
Reliability
B.
Consistency
C.
Comparability
D.
Accountability
A

The correct answer is: D
Explanation:
Governments are focused on programs and services that will improve the quality of life in their communities. Governmental entities must provide accountability for how they raised resources for those programs and explain how they managed and safeguarded those public resources.

How well did you know this?
1
Not at all
2
3
4
5
Perfectly
45
Q

Zephyr City has adopted GASB Statement No. 34, Basic Financial Statements and Management’s Discussion and Analysis for State and Local Governments. Which activities and basis of accounting must appear in Zephyr’s statement(s) of cash flows?

Business-type activities on the accrual basis in the fund financial statements
Government-type activities on the modified accrual basis in the fund financial statements
Business-type and government-type activities on the accrual basis in the government-wide financial statements
A.
I only
B.
II only
C.
III only
D.
I and III only

A

The correct answer is: A
Explanation:
Governmental-type funds pose problems for developing a meaningful government-wide cash flows statement. Only funds for business-type activities are required in the statement of cash flows.

How well did you know this?
1
Not at all
2
3
4
5
Perfectly
46
Q

Which of the following is a significant issue for SEA reporting for governmental entities?

A.
SEA reporting provides authorization for tax initiatives.
B.
SEA reporting will help explain budgetary proposals.
C.
SEA reporting helps bring performance results information to citizens to help with their assessment, program selection, and implementation.
D.
SEA reporting is required by GASB accounting and reporting standards.

A

The correct answer is: C
Explanation:
SEA reporting emphasizes performance results and is designed to help public policy-makers evaluate which programs are critical for a community.

How well did you know this?
1
Not at all
2
3
4
5
Perfectly
47
Q

Park City uses encumbrance accounting and formally integrates its budget into the general fund’s accounting records. Park incurred salaries and wages of $800,000 for the month of April. What account should Park debit to record this $800,000?

A.
Encumbrances control
B.
Salaries and wages expense control
C.
Expenditures control
D.
Operating funds control
A

The correct answer is: C
Explanation:
While goods and services committed for by purchase order or contract are encumbered in governmental funds to avoid overspending appropriations, some expenditures are controlled by other means and need not be encumbered. Salaries and wages are set by contract and controlled by established payroll procedures and are not encumbered. Salaries and wages expense would not be appropriate for a governmental-type fund. Salaries and wages should be recorded as follows:

Expenditures control 800,000
Vouchers payable 800,000

How well did you know this?
1
Not at all
2
3
4
5
Perfectly
48
Q

Governmental budgetary comparisons may be presented as required supplementary information (RSI) or which of the following?

A.
As a reconciling item in the statement of net assets
B.
As supplementary information
C.
In the notes to the financial statements
D.
In the basic financial statements
A

The correct answer is: D
Explanation:
Governments are encouraged to present budgetary comparisons as part of required supplementary information (RSI). There was significant discussion about whether these reports should be part of the basic financial statements, in part to ensure that audit coverage provided additional credibility for the reports. The GASB allowed governments to elect to present budgetary comparisons as statements within the basic financial statements, but not as part of other supplementary information or notes to the financial statements.

How well did you know this?
1
Not at all
2
3
4
5
Perfectly
49
Q

The following information pertains to Pine City’s general fund for the current year:

Appropriations	$6,500,000
Expenditures	5,000,000
Other financing sources	1,500,000
Other financing uses	2,000,000
Revenues	8,000,000
After Pine's general fund accounts were closed at the end of the year, the unassigned fund balance increased by
A.
$3,000,000
B.
$2,500,000
C.
$1,500,000
D.
$1,000,000
A

The correct answer is: B
Explanation:
Appropriations is a budgetary account which will also be closed. However, since closing the budgetary accounts (which are not all given) simply reverses the
entry to record the budget, their closing has no effect on fund balance. The closing entry for the activity accounts given would increase the fund balance, as
follows:

Revenues	8,000,000	 
Other financing sources	1,500,000	 
Expenditures	 	5,000,000
Other financing uses	 	2,000,000
Unassigned fund balance (difference)	 	2,500,000
How well did you know this?
1
Not at all
2
3
4
5
Perfectly
50
Q

Which of the following is not reported in the enterprise fund Statement of Net Position?

A.
Net investment in capital assets
B.
Restricted
C.
Unrestricted
D.
Capital contributions
A

The correct answer is: D
Explanation:
Capital contributions is not a category used on the government-wide statement of net position. Net position will be displayed in the same three categories that we used on the government-wide statement of net position: invested in capital assets, net of related debt; restricted net position; and unrestricted net position.

How well did you know this?
1
Not at all
2
3
4
5
Perfectly
51
Q

Elm City contributes to and administers a single-employer defined benefit pension plan on behalf of its covered employees. The plan is accounted for in a pension trust fund. Actuarially determined employer contribution requirements and contributions actually made for the past three years, along with the
percentage of annual covered payroll, were as follows:

Contribution made Actuarial requirement
Amount Percent Amount Percent

Year 3 $11,000 26 $11,000 26
Year 2 5,000 12 10,000 24
Year 1 None None 8,000 20
What account should be credited in the pension trust fund to record the year 3 employer contribution of $11,000?

A.
Revenues control
B.
Other financing sources control
C.
Due from special revenue fund
D.
Pension benefit obligation
A

The correct answer is: A
Explanation:
All pension trust fund contributions and earnings are accounted for as fund revenues. The pension trust fund makes the following entry to record employer contributions:

Cash 11,000
Revenues control—employer contribution 11,000

How well did you know this?
1
Not at all
2
3
4
5
Perfectly
52
Q
A local government has the following liabilities on its adjusted trial balance at the end of the current year. What amount would the government report on its balance sheet as total liabilities?
$50,000 accrued payroll
$100,000 bonds payable – current portion
$900,000 bonds payable – long-term portion
$25,000 accounts payable
A.
$1,075,000
B.
$175,000
C.
$100,000
D.
$75,000
A

The correct answer is: D
Explanation:
The balance sheet is a fund level financial statement and thus uses the modified accrual method of accounting. This means that the $1,000,000 related to the bonds will not be reported, and the answer is the $50,000 in accrued payroll and the $25,000 in accounts payable.

How well did you know this?
1
Not at all
2
3
4
5
Perfectly
53
Q

The water department of a city provides water to the nearby districts for a monthly fee. In year 2, it decides to take machinery on lease from one of the firms in the city. The lease term was decided to be 3 years and the ownership of the machinery would not transfer to the department at the end of the term. Given that the useful life of the machinery was 2 years and the value of the leased asset was $120,000 in the books of the water department, what would be the amortization in the books in year 2?

A.
$60,000
B.
$40,000
C.
$120,000
D.
$0
A

The correct answer is: A
Explanation:
The correct answer is (A).

In leases other than a short-term lease and contracts that transfer ownership, the lessee recognizes a right-to-use asset in its books. The asset is required to be amortized over the life of the asset or the lease term, whichever is less. In the given question, the useful life of 2 years is less than the lease term of 3 years. The value of the ROU asset shall be amortized over a period of 2 years.

Annual amortization = 120,000/2 = $60,000

How well did you know this?
1
Not at all
2
3
4
5
Perfectly
54
Q

Which of the following is the purpose for governmental fund accounting?

A.
Track subsidiary profit centers
B.
Evaluate revenues and expenditures for specific programs and monitor compliance with legal restrictions
C.
Separate revenues and expenditures into as many self-supporting accounts as possible
D.
Identify statutory requirements for budget management

A

The correct answer is: B
Explanation:
A government’s primary mission is the delivery of programs and services. Fund accounting allows governments to track specific functions and ensure that operations comply with any legal restrictions that apply. Governments may have component units that are similar to subsidiaries in the private sector, but there are no profit objectives in governmental financial reporting. The GASB discourages the use of too many funds; the general fund is the only required fund. GASB Statement No. 34 reorganized funds to create a primary focus for each fund category, but statutory identification is not a primary objective for fund organization.

How well did you know this?
1
Not at all
2
3
4
5
Perfectly
55
Q

Which of the following mandates the use of an enterprise fund?

A.
Activity is financed with debt that is secured by a pledge of the net revenues from fees and charges of the activity.
B.
User fees are charged to external users for goods and services.
C.
The General Fund provides a subsidy to cover operating deficits.
D.
All surplus funds are transferred to the General Fund as a return on capital.

A

A.
Activity is financed with debt that is secured by a pledge of the net revenues from fees and charges of the activity.

Explanation:
GASB Statement No. 34 listed three criteria, any one of which requires the use of an enterprise fund. Activity being financed with debt that is secured by a pledge of the net revenues from fees and charges of the activity is the first of the listed criteria. User fees charged to external users for goods and services is an optional reason for using an enterprise fund. The General Fund might provide a subsidy, but this financing does not require the use of an enterprise fund. Surplus funds may be returned to the General Fund, but this transfer does not require the use of an enterprise fund.

How well did you know this?
1
Not at all
2
3
4
5
Perfectly
56
Q

The following information pertains to Park Township’s general fund at December 31:

Total assets, including $200,000 of cash	$1,000,000
Total liabilities	600,000
Reserved for encumbrances	100,000Appropriations do not lapse at year-end. At December 31, what amount should Park report as unassigned fund balance in its general fund balance sheet?
A.
$200,000
B.
$300,000
C.
$400,000
D.
$500,000
A

The correct answer is: B
Explanation:
Because Park’s appropriations do not lapse at year-end, the Budgetary Fund Balance Reserved for Encumbrances account is converted from an offsetting memorandum account in the general ledger to a true commitment of fund balance year-end. The amount that Park should report as Unassigned Fund Balance in its general fund balance sheet is computed as follows:

Total assets $1,000,000
Less: Total liabilities 600,000
Total fund balance $ 400,000
Less: Committed fund balance 100,000
Unassigned fund balance, December 31 $ 300,000

How well did you know this?
1
Not at all
2
3
4
5
Perfectly
57
Q

Which of the following is a minimum required report for the basic financial statements of a government entity?

A.
Fund financial statements.
B.
Management's discussion and analysis.
C.
Required supplementary information.
D.
Budgetary comparison schedules.
A

The correct answer is: A
Explanation:
The correct answer is (A).

The various elements of the basic financial statements of a government entity are:

Government-Wide Financial Statements
Fund Financial Statements
Notes to the Financial Statements
Hence, out of the given options, fund financial statements qualifies as the minimum required report for the basic financial statements of a government entity.

How well did you know this?
1
Not at all
2
3
4
5
Perfectly
58
Q

Which format must an enterprise fund use to report cash flow operating activities in the statement of cash flows?

A.
Indirect method, beginning with operating income.
B.
Indirect method, beginning with change in net position.
C.
Direct method.
D.
Either direct or indirect method.
A

The correct answer is: C
Explanation:
An enterprise fund is one of the governmental proprietary funds. Governments should present a statement of cash flows for proprietary funds based on the provisions of GASB statements. The GASB states the direct method of presenting cash flows from operating activities (including a reconciliation of operating cash flows to operating income) should be used.

How well did you know this?
1
Not at all
2
3
4
5
Perfectly
59
Q

Which of the following is one of the three standard sections of a governmental comprehensive annual financial report?

A.
Investment.
B.
Actuarial.
C.
Statistical.
D.
Single audit.
A

The correct answer is: C
Explanation:
The comprehensive annual financial report contains the following sections: Introduction Section; Financial Section; and the Statistical Section.

How well did you know this?
1
Not at all
2
3
4
5
Perfectly
60
Q

Shared revenues received by an enterprise fund of a local government for operating purposes should be recorded as

A.
Operating revenues.
B.
Nonoperating revenues.
C.
Other financing sources.
D.
Interfund transfers.
A

The correct answer is: B
Explanation:
Grants, entitlements, and shared revenues received by proprietary funds should be reported as nonoperating revenues unless they are externally restricted to capital acquisitions.

How well did you know this?
1
Not at all
2
3
4
5
Perfectly
61
Q

As described in GASB Concepts Statement No. 3, the distinction between required supplementary information (RSI) and other supplementary information (OSI) is generally based on which of the following?

A.
Whether the GASB has specified the requirement to present the information
B.
Whether the government has decided to present the information
C.
The public’s demand for information from the government
D.
None of the above

A

The correct answer is: A
Explanation:
The distinction between RSI and OSI is generally based on whether the GASB has specified the requirement to present the information, not the public’s demand for the information. Management can certainly add to RSI, but additional information must be presented separately so that users can consistently compare reports among governmental entities.

How well did you know this?
1
Not at all
2
3
4
5
Perfectly
62
Q

Dayne County’s general fund had the following disbursements during the year.

Payment of principal on long-term debt $100,000
Payments to vendors 500,000
Purchase of a computer 300,000
What amount should Dayne County report as expenditures in its governmental funds statement of revenues, expenditures, and changes in fund balances?

A.
$300,000
B.
$500,000
C.
$800,000
D.
$900,000
A

The correct answer is: D
Explanation:
Expenditures differ from expenses (as defined in commercial accounting) because expenditures include—in addition to current operating expenditures that benefit the current period—capital outlays for general fixed assets and repayment of general long-term debt principal.

Payments of principal on long-term debt $100,000
Payments to vendors 500,000
Purchase of a computer 300,000
Total expenditures $900,000
Option (a) is incorrect because payment to vendors and payment of principal on long-term debt is excluded. Option (b) is incorrect because payment of principal on long-term debt and purchase of computer is excluded. Option (c) is incorrect because payment of principal on long-term debt is excluded.

How well did you know this?
1
Not at all
2
3
4
5
Perfectly
63
Q

In preparing combined financial statements for a governmental entity, interfund receivables and payables should be

A.
Reported as reservations of fund balance.
B.
Reported as additions to or reductions from the unassigned fund balance.
C.
Reported as amounts due to and due from other funds.
D.
Eliminated.

A

The correct answer is: C
Explanation:
Interfund receivables and payables are reported as amounts due to and due from other funds. They are not reported as reservations of fund balance or additions to or reductions from the unassigned fund balance. The option of eliminating the interfund assets and liabilities is allowed, but requires that such eliminations be apparent from the headings, or be disclosed in the notes to the financial statements.

How well did you know this?
1
Not at all
2
3
4
5
Perfectly
64
Q

Expenditures of a governmental unit for insurance extending over more than one accounting period

A.
Must be accounted for as expenditures of the period of acquisition.
B.
Must be accounted for as expenditures of the periods subsequent to acquisition.
C.
Must be allocated between or among accounting periods.
D.
May be allocated between or among accounting periods or may be accounted for as expenditures of the period of acquisition

A

The correct answer is: D
Explanation:
Expenditures are recorded when fund liabilities are incurred or assets are expended, except in regard to inventory items, interest on general long-term debt, and prepaids such as insurance. This is due to the emphasis on the flow of financial resources in governmental accounting.

How well did you know this?
1
Not at all
2
3
4
5
Perfectly
65
Q

Which of the following transactions is an expenditure of a governmental unit’s general fund?

A.
Contribution of enterprise fund capital by the general fund
B.
Operating subsidy transfer from the general fund to an enterprise fund
C.
Routine employer contributions from the general fund to a pension trust fund
D.
Transfer from the general fund to a capital projects fund

A

The correct answer is: C
Explanation:
General funds use modified accrual accounting. Both capital purchases and operating expenditures are considered spending of funds and are treated as current year expenditure. Routine employer contribution to a pension trust fund is an expenditure. Option (a) is incorrect because contribution of enterprise fund capital by the general fund is not an expenditure. Option (b) is incorrect because operating transfers are reported as other financing uses under modified accrual accounting. Option (d) is incorrect because transfers out is treated as other financing uses and not expenditure

How well did you know this?
1
Not at all
2
3
4
5
Perfectly
66
Q

Cash receipts from grants and subsidies to decrease operating deficits should be classified in which of the following sections of the statement of cash flows for governmental, not-for-profit entities?

A.
Operating
B.
Noncapital financing
C.
Capital and related financing
D.
Investing
A

The correct answer is: B
Explanation:
Noncapital financing activities include cash receipts from grants and subsidies except (1) those specifically restricted for capital purposes and (2) those for specific activities that are considered to be operating activities of the grantor government. Operating activities include cash inflows, receipts, and payments that do not result from transactions defined as capital and related financing, noncapital financing, or investing activities. Capital and related financing activities include acquiring and disposing of capital assets used in providing services or producing goods; borrowing money for acquiring, constructing, or improving capital assets and repaying the amounts borrowed, including interest; and paying for capital assets obtained from vendors on credit. Investing activities include making and collecting loans (except program loans) and acquiring and disposing of debt or equity instruments.

How well did you know this?
1
Not at all
2
3
4
5
Perfectly
67
Q

Which of the following characteristics trigger unique accounting standards for governmental entities?

A.
Governmental ownership of utilities such as water, sewer, or electric services
B.
Democratic structure for state and local governmental entities
C.
Fund accounting and budgetary reporting
D.
Profit motive in governmental funds
A

The correct answer is: C
Explanation:
Fund accounting and budgetary reporting are key elements for demonstrating accountability for public resources and provide cornerstones for governmental financial reporting. They are the characteristics that trigger unique accounting standards for governmental entities.

How well did you know this?
1
Not at all
2
3
4
5
Perfectly
68
Q

Which of the following is discussed in GASB Concepts Statement No. 1 as a financial reporting objective for governmental financial statements?

A.
Assisting users in understanding funding objectives for public retirement systems and the potential tax burden on future taxpayers
B.
Assisting users in identifying management’s policies for selection of critical programs and services to provide and the annual tax burden to finance those programs
C.
Assisting users in assessing the operating results, level of services provided, and the government’s ability to meet cash requirements
D.
Assisting users in determining the annual tax burden per capita for each of the major programs provided by the governmental entity

A

The correct answer is: C
Explanation:
Assisting users with an assessment of the government’s operating results, level of services provided, and the ability to meet cash requirements is a broad principle and is the second objective outlined in Concepts Statement No. 1. While assisting users in understanding funding objectives for public retirement systems and the potential tax burden on future taxpayers may be one of the significant elements for any governmental financial report, it is not specifically discussed as a reporting objective. Assisting users in identifying management’s policies for selection of critical programs and services to provide and the annual tax burden to finance those programs is an example of a specific reporting issue, not a broad principle. Assisting users in determining the annual tax burden per capita for each of the major programs provided by the governmental entity is an example of specific information that is critical to users, but not discussed as a primary broad objective.

How well did you know this?
1
Not at all
2
3
4
5
Perfectly
69
Q

Arlen City’s fiduciary funds contained the following cash balances at December 31:

Under the Forfeiture Act-cash confiscated from illegal activities;
principal can be used only for law enforcement activities $300,000
Sales taxes collected by Arlen to be distributed
to other governmental units 500,000
What amount of cash should Arlen report in its permanent funds at December 31?

A.
$0
B.
$300,000
C.
$500,000
D.
$800,000
A

The correct answer is: A
Explanation:
The cash collected under the For feiture Act, can only be used for law enforcement activities. It should be accounted for in a special revenue fund because the Act does not require the preservation of fund principal and the principal may be used for Arlen’s benefit . An agency fund is established to account for assets received by a government in its capacity as an agent for individuals, businesses, or other governments. Therefore, the sales taxes collected by Arlen to be distributed to other governments are accounted for in an agency fund.

How well did you know this?
1
Not at all
2
3
4
5
Perfectly
70
Q

For a municipal solid waste landfill, equipment and facilities included in the estimated total current cost of closure and postclosure care are reported in the statement of net position as which of the following?

A.
Current expenses
B.
Capital assets
C.
A reduction of the liability
D.
None of the above
A

The correct answer is: C
Explanation:
Equipment and facilities included in the estimated total current cost of closure and postclosure care are not reported as current expenses, but as a reduction of the liability. A liability that would be excluded from the “capital-related debt” portion of the statement of net position is the liability for closure and postclosure care associated with a municipal solid waste landfill.

How well did you know this?
1
Not at all
2
3
4
5
Perfectly
71
Q

Palm City acquired, through forfeiture as a result of nonpayment of property taxes, a parcel of land that the city intends to use as a parking lot for general governmental purposes. The total amount of taxes, liens, and other costs incurred by Palm incidental to acquiring ownership and perfecting title was $20,000. The land’s fair market value at the forfeiture date was $60,000. What amount should be reported in the governmental activities column of the governmentwide financial statements for this land?

A.
$0
B.
$20,000
C.
$60,000
D.
$80,000
A

The correct answer is: B
Explanation:
General fixed assets acquired by foreclosure are recorded at the lower of (1) the amount due for taxes, special assessments, penalties and interest, plus foreclosure costs or (2) appraised fair market value. Therefore, since the $20,000 Palm incurred for taxes, liens, and other costs incidental to acquiring ownership and perfecting title is less than the land’s $60,000 fair market value at the forfeiture date, the land should be reported in the governmental activities column of government-wide financial statements at $20,000.

How well did you know this?
1
Not at all
2
3
4
5
Perfectly
72
Q

Governments are required to present budgetary comparisons for which of the following?

A.
The general fund only
B.
The general fund and all major special revenue funds
C.
All governmental and proprietary funds
D.
None of the above
A

The correct answer is: B
Explanation:
Governmentsare required to present budgetary comparisons for the general fund and all major special revenue funds in the general purpose external financial reports, but not for proprietary funds.

How well did you know this?
1
Not at all
2
3
4
5
Perfectly
73
Q

Lily City uses a pay-as-you-go approach for funding postemployment benefits other than pensions. The city reports no other postemployment benefits (OPEB) liability at the beginning of the year. At the end of the year, Lily City reported the following information related to OPEB for the water enterprise fund:

Benefits paid	$100,000
Annual required contribution	500,000
Unfunded actuarial accrued liability	800,000What amount of expense for OPEB should Lily City’s water enterprise fund report in its fund level statements?
A.
$100,000
B.
$500,000
C.
$600,000
D.
$1,400,000
A

The correct answer is: B
Explanation:
Other postemployment benefits (OPEB) are typically in the form of health care coverage. Most entities fund their OPEB on a pay-as-you-go basis, paying only the amount equal to benefits claimed. GASB believes the pay-as-you-go approach does not reflect the true OPEB liability, and issued GASB 45 outlining OPEB reporting and disclosure requirement. The objective is to systematically and rationally allocate the present value of the projected cost of benefits over the working life of benefiting employees. This allocation is done with an actuarial valuation. The amount allocated to a given year is known as the annual required contribution (ARC), which is the basic amount of expense (i.e., $500,000). The present value of benefits earned to date by employees is known as the actuarial accrued liability. The difference between this amount and any resources placed in trust is known as the unfunded actuarial accrued liability.

How well did you know this?
1
Not at all
2
3
4
5
Perfectly
74
Q

The debt service transactions of a special assessment bond issue for which the government is not obligated in any manner should be reported in the

A.
Custodial Fund
B.
Enterprise Fund
C.
Special Revenue Fund
D.
Long-Term Debt Account Group
A

The correct answer is: A
Explanation:
The correct answer is (A).

Although the special assessment debt for which the government is not obligated in any manner is not reported as government debt, the government usually acts as a debt service agent for the special assessment district.

The government has a fiduciary responsibility to collect the special assessments and to remit the collections to the bondholders when the debt service payments come due.

However, if collections are insufficient to cover required debt service payments, the government is not obligated to pay the difference and does not intend to do so.

Thus, the government is acting in a custodial capacity with respect to the debt service transactions, and these transactions should be accounted for in a Custodial Fund.

How well did you know this?
1
Not at all
2
3
4
5
Perfectly
75
Q

King City Council will be establishing a library fund. Library fees are expected to cover 55% of the library’s annual resource requirements. King has decided that an annual determination of net income is desirable in order to maintain management control and accountability over library. What type of fund should King establish in order to meet their measurement objectives?

A.
Special revenue fund.
B.
General fund.
C.
Internal service fund.
D.
Enterprise fund.
A

The correct answer is: D
Explanation:
Enterprise funds must be used to account for a government’s business-type operations that are financed and operated like private businesses—where the government’s intent is that all costs of providing goods or services to the general public on a continuing basis are to be financed or recovered primarily through users charges. Governments are permitted to account for virtually any type of self-contained business-type activity in enterprise funds if it prefers to do business-type accounting rather than general government accounting. An enterprise fund is a proprietary fund.

How well did you know this?
1
Not at all
2
3
4
5
Perfectly
76
Q

Which of the following would be reported as program revenues on a local government’s government-wide statement of activities?

A.
Charges for services
B.
Taxes levied for a specific function
C.
Proceeds from the sale of a capital asset used for a specific function
D.
Interest revenues
A

A.
Charges for services

Explanation:
On the government-wide statement of activities, the net revenue (expense) format and net program cost format are used. Charges for services are one of the program revenues reported on a local government’s government-wide statement of activities. Taxes levied, proceeds from the sale of a capital asset, and interest revenues would fall under general revenues.

How well did you know this?
1
Not at all
2
3
4
5
Perfectly
77
Q

Economic and demographic information in the statistical section of the CAFR helps users do which of the following?

A.
Gauge the resources for the government’s revenues and demands for public services
B.
Calculate the debt burden per capita
C.
Evaluate the direct and overlapping debt burden on the available taxpayers
D.
Perform all of the above actions
A

The correct answer is: A
Explanation:
Economic and demographic information can help users gauge the economic health of a community, and thus, the demands for governmental services as well as the key factors that will impact revenues. The debt burden per capita is presented in the debt capacity tables. Overlapping debt information is presented in the debt capacity tables.

How well did you know this?
1
Not at all
2
3
4
5
Perfectly
78
Q

Cy City’s Municipal Solid Waste Landfill Enterprise Fund was established when a new landfill was opened January 3, year 1. The landfill is expected to close December 31, year 20. Cy’s year 1 expenses would include a portion of which of the year 21 expected disbursements?

Cost of a final cover to be applied to the landfill.
Cost of equipment to be installed to monitor methane gas buildup.
A.
I only.
B.
II only.
C.
Both I and II.
D.
Neither I nor II.
A

The correct answer is: C
Explanation:
In accounting for municipal solid waste landfill (MSWLF) costs, a portion of the estimated total current costs must be recognized as an expense and as a liability in each period during which the MSWLF accepts solid waste. The estimated total current costs of MSWLF closure and postclosure care should include those costs which result in disbursements near, or after, the date that the MSWLF stops accepting solid waste and during the postclosure period, including the cost of a final cover and monitoring equipment.

How well did you know this?
1
Not at all
2
3
4
5
Perfectly
79
Q

Which of the following is considered the most prevalent fiduciary fund?

A.
Custodial fund
B.
Pension trust fund
C.
Investment trust fund
D.
Private-purpose trust fund
A

The correct answer is: B
Explanation:
The most prevalent fiduciary fund is the pension trust fund, not the custodial fund, the investment trust fund, or the private-purpose trust fund.

How well did you know this?
1
Not at all
2
3
4
5
Perfectly
80
Q

Which of the following statements about the governmental statement of activities is false?

A.
The statement of activities highlights the costs of programs and the sources of revenues for those programs.
B.
Governmental services will likely show large deficits in the net revenue column.
C.
Tax revenues may be reported as program income if these resources are restricted to a specific program.
D.
Governments are allowed to allocate administrative and overhead costs from the general government category to individual programs.

A

The correct answer is: C
Explanation:
All taxes are considered general revenues, even if restricted to a specific program. GASB Statement No. 34 shifted the governmental operating statement to a program focus that first matches the costs of program revenues and the sources of revenues for those programs. The statement of activities divides the operations of the government not into traditional line items—but into program or operational categories. Governmental services will likely show large deficits in the net (expense) revenue column. This allocation is optional, but governments are allowed to allocate the administrative and overhead costs from the general government category.

How well did you know this?
1
Not at all
2
3
4
5
Perfectly
81
Q

Which type of fund is allowed to incur long-term debt through the issuance of bonds?

A.
Governmental funds
B.
Proprietary funds
C.
Both governmental and proprietary funds
D.
Neither governmental nor proprietary funds
A

The correct answer is: C
Explanation:
Both governmental and proprietary funds could issue long-term debt in the form of bonds payable. The differences would lie in the reporting of the bond proceeds and the related liabilities on the financial statements.

How well did you know this?
1
Not at all
2
3
4
5
Perfectly
82
Q

Carlson City’s fiscal year ends December 31. On August 1, the city issued a purchase order for new vehicles to be delivered at the rate of two per month beginning October 15. Twelve vehicles were delivered as scheduled and payments of $264,000 were made upon delivery. If these were the only transactions made by the city, which of the following balances would appear on the balance sheet as of December 31?

A	Encumbrances
Reserve for encumbrances	$132,000
132,000
B	Unassigned fund balance
Reserve for encumbrances	$132,000
132,000
C	Reserve for encumbrances
Unassigned fund balance	$264,000
264,000
D	Encumbrances
Reserve for encumbrances	$264,000
264,000
A

The correct answer is: B
Explanation:
The correct answer is (B).

Year-end closing entries close the budgetary accounts for open encumbrances (i.e., orders that were placed during the year but not fulfilled as of the end of the year).

Also, unreserved fund balance on the books is reduced since an encumbrance represents a legal commitment to purchase.

Between October 15 and December 31, six vehicles were delivered at the rate of two per month, while an encumbrance was created for 12 vehicles at $264,000.

Therefore, $132,000 was paid for the delivery of six vehicles.

Dr: Unassigned Fund Balance $132,000
Cr: Reserved for encumbrances $132,000
Note: The schedule was for two vehicles per month starting October 15 for a total of 12 vehicles. As all vehicles were delivered as planned, two vehicles each for October, November, and December will total to six vehicles.

How well did you know this?
1
Not at all
2
3
4
5
Perfectly
83
Q

It is inappropriate to record depreciation expense in the government-wide financial statements related to the assets in which of Kellick City’s funds?

A.
Custodial fund
B.
Enterprise fund
C.
General fund
D.
Special revenue fund
A

The correct answer is: A
Explanation:
The correct answer is (A)

Custodial funds should be used to report resources held by the reporting government in a purely custodial capacity (assets equal liabilities).

These funds typically involve only the receipt, temporary investment, and remittance of fiduciary resources to individuals, private organizations, or other governments.

Custodial funds generally have neither capital assets nor expenses, and as such would not have depreciation expense

How well did you know this?
1
Not at all
2
3
4
5
Perfectly
84
Q

The following information pertains to property taxes levied by Oak City for the calendar year 1:

Collections during year 1 $500,000
Expected collections during the first 60 days of year 2 100,000
Expected collections during the balance of year 2 60,000
Expected collections during January year 3 30,000
Estimated to be uncollectible 10,000
Total levy $700,000What amount should Oak report for year 1 net property tax revenues?
A.
$700,000
B.
$690,000
C.
$600,000
D.
$500,000

A

The correct answer is: C
Explanation:
Governmental funds use the modified accrual basis of accounting, under which revenues susceptible to accrual (e.g., property taxes) are recognized when they become measurable and available for use. ‘Available for use’ means that the revenues will be collected within the current period or collected early enough in the next period (i.e., within 60 days or so) to be used to pay for expenditures incurred in the current period. Therefore, for year 1, Oak should report property tax revenues of $600,000, the sum of the of property taxes levied and collected in year 1 and the expected property tax collections during the first 60 days of year 2.

How well did you know this?
1
Not at all
2
3
4
5
Perfectly
85
Q

Polk County’s solid waste landfill operation is accounted for in a governmental fund. Polk used available cash to purchase equipment that is included in the estimated current cost of closure and post-closure care of this operation. How would this purchase affect the long-term asset and the long-term liability amounts in Polk’s general fund?

Asset	Liability
A	Increase	Decrease
B	Increase	No effect
C	No effect	No effect
D	No effect	Decrease
A

The correct answer is: D
Explanation:
The correct answer is (D).

A MSWLF (Municipal Solid Waste Landfill) is accounted for in a government-type fund and should accrue the cost of assets required for closure and post-closure on an annual basis with a corresponding liability. When the available cash is used to purchase equipment, it has no effect on the asset but the liability would be reduced.

Equipment, facilities, services, and final expenses included in the estimated total current cost of closure and post-closure care should be reported as a reduction of the reported liability for closure and post-closure care when they are acquired.

How well did you know this?
1
Not at all
2
3
4
5
Perfectly
86
Q

Which of the following funds of a local government would report transfers to other funds as an other financing use?

A.
Enterprise
B.
Internal service
C.
Pension trust
D.
General
A

The correct answer is: D
Explanation:
The correct answer is (D).

Interfund transactions appear in the operating statements of the affected funds. In governmental funds, transfers are reported as Other Financing Sources (Uses). In proprietary funds, transfers should be reported as Transfers In (Out). Only the general fund is a governmental fund and would report transfers to other funds as an other financing use. The enterprise and internal service funds are proprietary funds that would report transfers to other funds as transfers out. The pension trust fund is a fiduciary fund that is accounted for in essentially the same manner as proprietary funds.

How well did you know this?
1
Not at all
2
3
4
5
Perfectly
87
Q

Which of the following is NOT included on the face of the government-wide statement of net position?

A.
Governmental activities
B.
General fund
C.
Business-type activities
D.
Component unit
A

The correct answer is: B
Explanation:
The three primary columns shown on the face of the government-wide statement of net position are governmental activities, business-type activities, and component units if applicable. The balances for the general fund would be shown on the balance sheet of the governmental fund

How well did you know this?
1
Not at all
2
3
4
5
Perfectly
88
Q

Baker City is planning to lease some trucks. Determine the lease term from the information given below:

Non-cancellable period of the lease: 5 years
Renewal periods not certain to be exercised: 2 years
Periods covered under a termination option: 1 year
Non-termination periods controlled by the lessor: 2 years
It is to be assumed that the city won’t exercise the termination option.

A.
5 years
B.
10 years
C.
8 years
D.
7 years
A

The correct answer is: C
Explanation:
The correct answer is (C).

For Governmental Accounting, the lease term is calculated the same way as it is in Financial Accounting under the FASB.

The lease term or tenure of the lease is calculated as follows:

Non-cancellable period: 5 years.

Renewal periods “reasonably certain” of exercise
Periods covered by a termination option reasonably certain not to be exercised: 1 Year
Renewal periods (non-termination periods) controlled by lessor: 2 Years.
Total: 8 Years.
In the given case of Baker City, all periods are included except for the renewal period which is not certain to be exercised.

How well did you know this?
1
Not at all
2
3
4
5
Perfectly
89
Q
Which of the following is the paramount objective of financial reporting by state and local governments?
A
Reliability
B
Consistency
C
Comparability
D
Accountability
A

D

Explanation:
Governments are focused on programs and services that will improve the quality of life in their communities. Governmental entities must provide accountability for how they raised resources for those programs and explain how they managed and safeguarded those public resources.

How well did you know this?
1
Not at all
2
3
4
5
Perfectly
90
Q

Encumbrances would not appear in which fund?

A
Capital projects.
B
Special revenue.
C
General.
D
Enterprise.
A

D

Explanation:
The encumbrance system is used in governmental funds (general, special revenue, and capital pro­jects funds) to prevent over-expenditure and to demonstrate compliance with legal requirements.The enterprise fund is a proprietary fund and does not use the encumbrance system.Option (a), (b) and (c) are incorrect because these funds can have encumbrances.

How well did you know this?
1
Not at all
2
3
4
5
Perfectly
91
Q

Oak County incurred the following expenditures in issuing long-term bonds:

Issue cost $ 400,000
Debt insurance 90,000
When Oak establishes the accounting for operating debt service, what amount should be deferred and amortized over the life of the bonds?

A
$90,000
B
$0
C
$400,000
D
$490,000
A

Explanation:
The correct answer is (B).

The general long-term debt of a state or local government is reported:

Government-wide financial statements: Accrual basis of accounting.
Governmental Funds (Debt Service Funds): Modified Accrual basis of accounting.
In Government Funds (Debt Service Funds) under Modified Accrual basis of accounting costs are recognized using the expenditure principle, and recorded in the period the obligation to pay arises, irrespective of when the goods/services are being actually used by the government entity.

Accordingly, the modified accrual basis does not allow for amortization of issue costs and debt insurance premiums, which must be recognized in full when the liability is incurred.

The amount deferred and amortized over the life of the bonds is $0.

$490,000 is expensed in the period incurred.

How well did you know this?
1
Not at all
2
3
4
5
Perfectly
92
Q

Dale Town’s public school system is administered by a separately elected board of education. The board of education is not organized as a separate legal entity and does not have the power to levy taxes or issue bonds. Dale’s town council approves the school system’s budget. Where should Dale report the public school system in its government-wide information?

A
Within the component units column
B
Within the governmental activities column
C
In the notes to the financial statements
D
In the required supplementary information
A

Explanation:
The correct answer is (B).

The government-wide statements should display information about the reporting government as a whole, except for its fiduciary activities. Governmental-Wide Financial Statement’s, Statement of Net Position has four columns as follows:

Governmental activities: Presents consolidated results of all the Governmental Funds and the Internal Service Funds including all component units subject to blended presentation
Business-type activities: Presents consolidated results of all Enterprise Funds
Total: Presents a total of Governmental Activities & Business-type Activities
Component units: Presents results of all the component units subject to discrete presentation
Component units are legally separate organizations for which elected officials of the primary government are financially accountable.

A unit classifies for a discreet presentation (within the component unit’s column) when it meets the following criteria or else it is presented as a blended component within the Governmental activities column.

Services provided by the government are not primary to the government
Management of the unit consists of separately elected officials i.e. a separately elected governing board.
Separate Legal Entity
Fiscally Independent i.e. power to determine its own budget, levy taxes, and issue bonds.
Dale Town’s public-school system has separately elected governing board but it is not a separate legal entity and does not have the power to levy taxes or issue bonds (i.e. not fiscally independent) and hence it fails the criteria for discreet presentation and will be presented as a blended component within Governmental activities column.

How well did you know this?
1
Not at all
2
3
4
5
Perfectly
93
Q

In governmental accounting, a fund is

The basic accounting unit.
Used to assist in ensuring fiscal compliance.
A
I only
B
II only
C
Both I and II
D
Neither I nor II
A

C

Explanation:
A specific governmental unit is not accounted for through a single accounting entity. Instead, the accounts of a government are divided into several funds. A fund is a fiscal and accounting entity with a self-balancing set of accounts recording cash and other financial resources, together with all related liabilities and residual equities and balances, and changes therein, which are segregated for the purpose of carrying on spe­cific activities or attaining certain objectives in accordance with special regulations, restrictions, or limitations.

How well did you know this?
1
Not at all
2
3
4
5
Perfectly
94
Q

As described in GASB Concepts Statement No. 3, the distinction between required supplementary information (RSI) and other supplementary information (OSI) is generally based on which of the following?
A
Whether the GASB has specified the requirement to present the information
B
Whether the government has decided to present the information
C
The public’s demand for information from the government
D
None of the above

A

A

Explanation:
The distinction between RSI and OSI is generally based on whether the GASB has specified the requirement to present the information, not the public’s demand for the information. Management can certainly add to RSI, but additional information must be presented separately so that users can consistently compare reports among governmental entities.

How well did you know this?
1
Not at all
2
3
4
5
Perfectly
95
Q

The governmental category in the fund statements includes which of the following?
A
General fund, special revenue, debt service, and enterprise funds
B
General fund, special revenue, debt service, capital projects, and permanent funds
C
General fund, special revenue, fiduciary funds, capital projects, and internal service funds
D
General fund, special revenue, debt service, and internal service funds

A

B

Explanation:
The governmental category in the fund statements summarizes those programs funded with general types of revenues (generally taxes and grants). This category includes the general fund, special revenue, debt service, capital projects, and permanent funds. Fiduciary funds are not available to support the government’s own programs. Proprietary funds include: enterprise funds and internal service funds.

How well did you know this?
1
Not at all
2
3
4
5
Perfectly
96
Q

Cedar City issues $1,000,000, 6% revenue bonds were issued at par on April 1, to build a new water line for the water enterprise fund. Interest is payable every six months. What amount of interest expense should be reported for the year ended December 31?

A
$0
B
$30,000
C
$45,000
D
$60,000
A

C

Explanation:
Since this is an enterprise fund, accrual accounting applies. Interest expense for the year includes the $30,000 ($1,000,000 × 6% × 1/2 year) paid on October 1st and the $15,000 ($1,000,000 × 6% × 1/4 year) accrued expense from October 1st through December 31 for a total of $45,000.

How well did you know this?
1
Not at all
2
3
4
5
Perfectly
97
Q
Marta City's school district is a legally separate entity but two of its seven board members are also city council members and the district is financially dependent on the city. The school district should be reported as a
A
Blended unit.
B
Discrete presentation.
C
Note disclosure.
D
Primary government.
A

B

Explanation:
Component units should be presented discretely unless either (a) the components unit’s governing body is substantively the same as the governing body of the primary government, or (b) the component unit provides services almost entirely to the primary government, or almost exclusively benefits the primary government although it does not provide services directly to it.

How well did you know this?
1
Not at all
2
3
4
5
Perfectly
98
Q

Elliott Township purchased a firetruck costing $1,000,000 in its general fund on January 1 of the current year. The truck had a useful life of 10 years and the Township has a policy of taking a full year’s depreciation in the first year. How much depreciation related to the truck would the Township report on its statement of activities?

A
$1,000,000
B
$100,000
C
$10,000
D
$0
A

B
Explanation:
The statement of activities is a government-wide financial statement which uses the accrual method of accounting. The fire truck would be reported as a capital asset on the government-wide statement of net position, and $100,000 in depreciation would be reported on the statement of activities.

How well did you know this?
1
Not at all
2
3
4
5
Perfectly
99
Q

Which of the following statements is correct concerning disclosure of reverse repurchase and fixed coupon reverse repurchase agreements?
A
Related assets and liabilities should be netted.
B
Related interest cost and interest earned should be netted.
C
Credit risk related to the agreements need not be disclosed.
D
Underlying securities owned should be reported as “Investments.”

A

D

Explanation:
The underlying securities owned with regard to reverse repurchase and fixed coupon reverse repurchase agreements should be reported as “Investments.” Related assets and liabilities and interest cost and interest earned should not be netted. Credit risk related to such agreements must be disclosed.

How well did you know this?
1
Not at all
2
3
4
5
Perfectly
100
Q

Which of the following amounts are included in a general fund’s encumbrance account?

Outstanding vouchers payable amounts
Outstanding purchase order amounts
Excess of the amount of a purchase order over the actual expenditure for that order
A
I only.
B
I and III.
C
II only.
D
II and III.
A

C

Explanation:
The encumbrance system is used by governmental funds to prevent overexpenditure and to demonstrate compliance with legal requirements. When a purchase order is issued, the estimated amount of the planned expenditure is encumbered by debiting Encumbrances and crediting Fund Balance Reserved for Encumbrances. When the related invoice is received, the encumbrance entry is reversed and the actual expenditure is recorded. Thus, the balance of the Encumbrance account will always equal the outstanding purchase order amounts until the books are closed at year-end.

How well did you know this?
1
Not at all
2
3
4
5
Perfectly
101
Q

Which of the following funds should be reported as part of local government’s governmental activities column in its government-wide statements?

A
Debt service
B
Custodial
C
Private-purpose trust
D
Pension trust
A

Explanation:
The correct answer is (A).

The debt service fund should be reported as part of local government’s governmental activities column in its government-wide statements. Reporting capital assets and long-term liabilities are required in the government-wide statements. Custodial, private-purpose trust, and pension trust funds are all fiduciary funds. Fiduciary activities are not included in the government-wide statements because the assets and liabilities cannot be used to support the government’s own programs.

Option (b), (c) and (d) are incorrect because they are fiduciary funds and excluded from government-wide F/S.

How well did you know this?
1
Not at all
2
3
4
5
Perfectly
102
Q
Pine City's year end is June 30. Pine levies property taxes in January of each year for the calendar year. One-half of the levy is due in May and one-half is due in October. Property tax revenue is budgeted for the period in which payment is due. The following information pertains to Pine's property taxes for the period from July 1, year 1, to June 30, year 2:
Calendar year
Year 1	Year 2
Levy	$2,000,000	$2,400,000
Collected in:		
May	$950,000	$1,100,000
July	50,000	60,000
October	920,000	
December	80,000	The $40,000 balance due for the May, year 2, installments was expected to be collected in August, year 2. What amount should Pine recognize for property tax revenue for the year ended June 30, year 2?
A
$2,160,000
B
$2,200,000
C
$2,360,000
D
$2,400,000
A

B

Explanation:
Governmental funds use the modified accrual basis of accounting, under which, revenues are recognized when they become measurable and available for use. ‘Available for use’ means that the revenues will be collected within the current period or collected early enough in the next period (e.g., within 60 days or so) to be used to pay for expenditures incurred in the current period. Therefore, for the year ended June 30, year 2, Pine should recognize property tax revenue of $2,200,000. This amount is comprised of (1) $1,000,000 (i.e., $920,000 + $80,000) of property taxes levied in January of year1 which were collected in October and December of year 1 and (2) $1,200,000 (i.e., $1,100,000 + $60,000 + $40,000) of property taxes levied in January year 2 which were collected in May, July, and August year 2.

How well did you know this?
1
Not at all
2
3
4
5
Perfectly
103
Q

During the current year Knoxx County levied property taxes of $2,000,000, of which 1% is expected to be uncollectible. The following amounts were collected during the current year:
Prior year taxes collected within the 60 days of the current year $ 50,000
Prior year taxes collected between 60 and 90 days into current year 120,000
Current year taxes collected in the current year 1,800,000
Current year taxes collected within the first 60 days of the subsequent year 80,000What amount of property tax revenue should Knoxx County report in its entity-wide statement of activities?
A
$1,800,000
B
$1,970,000
C
$1,980,000
D
$2,000,000

A

C

Explanation:
Entity-wide statements, the government-wide statements, are done under normal accrual accounting. Only fund financial statements, done under modified accrual, would take into account any 60-day criteria. Under normal accrual accounting the amount of property tax revenue reported would be the $2,000,000 property taxes levied less the 1% ($20,000) for estimated uncollectible taxes for a total of $1,980,000. The entry would be:

Taxes receivable - current 2,000,000
Estimated uncollectible taxes 20,000
Revenues control 1,980,000To record property tax revenue.

How well did you know this?
1
Not at all
2
3
4
5
Perfectly
104
Q
Which of the following would be reported as program revenues on a local government's government-wide statement of activities?
A
Charges for services
B
Taxes levied for a specific function
C
Proceeds from the sale of a capital asset used for a specific function
D
Interest revenues
A

A

Explanation:
On the government-wide statement of activities, the net revenue (expense) format and net program cost format are used. Charges for services are one of the program revenues reported on a local government’s government-wide statement of activities. Taxes levied, proceeds from the sale of a capital asset, and interest revenues would fall under general revenues.

How well did you know this?
1
Not at all
2
3
4
5
Perfectly
105
Q

A city government would report each of the following categories in its government-wide statement of net position except

A
Governmental activities.
B
Business-type activities.
C
Fiduciary activities.
D
Component units.
A

Explanation:
The correct answer is (C).

In the government-wide financial statements, all government and proprietary funds are included (but not fiduciary funds) along with discreetly presented component units in the statement of net position.

(A), (B) and (D) are incorrect because these are included in the statement of net position.

How well did you know this?
1
Not at all
2
3
4
5
Perfectly
106
Q

On January 1, Fonk City approved the following general fund resources for the new fiscal period:

Property taxes	$ 5,000,000
Licenses and permits	400,000
Intergovernmental revenues	150,000
Transfers in from other funds	350,000
What amount should Fonk record as estimated revenues for the new fiscal year?
A
$5,400,000
B
$5,550,000
C
$5,750,000
D
$5,900,000
A

B

Explanation:
The correct answer is B. Estimated revenues are revenues that are expected to be available to be spent in the period.

Ref

Summary

Amount

a

Property taxes

$5,000,000

b

Licenses & Permits

$400,000

c

Inter-governmental revenues

$150,000

d

Estimated Revenue (a+b+c)

$5,550,000

Transfers in from other funds are not considered because it is an estimated other financing source, not revenue. Inter-governmental revenue is included in the calculation of estimated revenue. Transfers in from other funds of $350,000 are not included in the calculation for estimated revenue.

How well did you know this?
1
Not at all
2
3
4
5
Perfectly
107
Q

Which of the following is the measurement focus and basis of accounting for the government-wide financial statements?

Measurement Focus	Basis of accounting
A	Economic resources	Accrual
B	Economic resources	Modified accrual
C	Current financial resources	Accrual
D	Current financial resources	Modified accrual
A

A
Explanation:
(a) GASB 34 requires the economic resources measurement focus and accrual basis of accounting for all amounts in the government-wide financial statements. Option (b) and (c) are incorrect as per the above explanation. Option (d) is incorrect because it applies to governmental funds; not to government-wide F/S.

How well did you know this?
1
Not at all
2
3
4
5
Perfectly
108
Q

A city government reported a $9,000 increase in net position in the motor pool internal service fund, a $12,000 increase in net position in the water enterprise fund, and a $7,000 increase in the employee pension fund. The motor pool internal service fund provides service primarily to the police department. What amount should the city report as the change in net position for business-type activities in its statement of activities?

A
$9,000
B
$12,000
C
$21,000
D
$28,000
A

Explanation:
The correct answer is (B).

Business type activities that are normally financed through user charges and reports the consolidated results of all enterprise funds. It does not include internal service funds, which are accounted for in governmental activities. The employee pension fund is a fiduciary fund and not included in business-type activities. The $12,000 increase in net position from Water enterprise is included.

(A) is incorrect because the motor pool internal service fund is excluded from business-type activities.

(C) is incorrect because the calculation also includes motor pool internal service fund which should be excluded.

(D) is incorrect because the motor pool Internal service fund and employee pension trust fund should be excluded from business-type activities.

How well did you know this?
1
Not at all
2
3
4
5
Perfectly
109
Q

Lake County received the following proceeds that are legally restricted to expenditure for specified purposes:

Levies on affected property owners to install sidewalks $500,000
Gasoline taxes to finance road repairs 900,000
What amount should be accounted for in Lake’s special revenue funds?

A
$1,400,000
B
$900,000
C
$500,000
D
$0
A

B

Explanation:
Special revenue fund sources includes fees, grants, specific taxes, and other earmarked revenue sources. The gasoline taxes to finance road repairs for $900,000 would be a specific tax, accounted under special revenue funds. Levies on affected property owners to install sidewalks is usually accounted for under the agency fund. It may also be accounted for in a debt service fund, for example: the levies are used to repay a bond that was issued to install the sidewalks. Options (a), (c) and (d) are incorrect based on the above explanation.

How well did you know this?
1
Not at all
2
3
4
5
Perfectly
110
Q

Grove Township issued $50,000 of bond anticipation notes at face amount in the current year and placed the proceeds into its capital projects fund.All legal steps were taken to refinance the notes, but Grove was unable to consummate refinancing. In the capital projects fund, what account should be credited to record the $50,000 proceeds?

A
Other Financing Sources Control
B
Revenues Control
C
Deferred Revenues
D
Bond Anticipation Notes Payable
A

D

Explanation:
A governmental unit would issue bond anticipation notes (BANs) to provide funds to defray costs expected to be incurred before the related bonds are issued. Such notes are treated as long-term debt, even if due within one year, if (1) they are to be repaid with the proceeds of the bond issue, (2) all legal steps have been taken to refinance the notes, and (3) the intent is supported by an ability to refinance the short-term notes on a long-term basis. Since all of these criteria are not met for the bond anticipation notes in question, they are reported as a liability of the capital projects fund.

How well did you know this?
1
Not at all
2
3
4
5
Perfectly
111
Q

Which of the following funds of a local government would report transfers to other funds as another financing use?

A
Enterprise.
B
Internal service.
C
Pension trust.
D
General.
A

Explanation:
The correct answer is (D).

The general fund of a local government is used to report transfers to other funds as an “other financing use.”

Only governmental funds (general, special revenue, debt service, capital projects, and permanent) report transfers to other funds as “other financing use.”

How well did you know this?
1
Not at all
2
3
4
5
Perfectly
112
Q
Chase City uses an internal service fund for its central motor pool. The assets and liabilities account balances for this fund that are not eliminated normally should be reported in the government-wide statement of net position as
A
Governmental activities
B
Business-type activities
C
Fiduciary activities
D
Note disclosures only
A

A

Explanation:
Although internal service funds are proprietary funds, they appear in the government-wide statements as governmental activities in the new reporting model.

How well did you know this?
1
Not at all
2
3
4
5
Perfectly
113
Q

A special-purpose government is distinguished from a general purpose government by which of the following?
A
It is established by a state government.
B
It provides limited services, such as only governmental or business-type activities.
C
It does not manage any fiduciary funds.
D
It is not governed by a separately elected legislative body.

A

B

Explanation:
The distinction between general purpose and special-purpose governments is based on the entity’s programs and services, not how the government was created. Special-purpose governments only provide limited services; unlike cities, counties, states, towns or villages that provide a broad range of services. It is possible that a special purpose government may only manage fiduciary programs. Governance structure is not the distinguishing feature of special-purpose governments.

How well did you know this?
1
Not at all
2
3
4
5
Perfectly
114
Q

The water department of a city provides water to the nearby districts for a monthly fee. In year 2, it decides to take machinery on lease from one of the firms in the city. The lease term was decided to be 3 years and the ownership of the machinery would not transfer to the department at the end of the term. Given that the useful life of the machinery was 2 years and the value of the leased asset was $120,000 in the books of the water department, what would be the amortization in the books in year 2?

A
$60,000
B
$40,000
C
$120,000
D
$0
A

Explanation:
The correct answer is (A).

In leases other than a short-term lease and contracts that transfer ownership, the lessee recognizes a right-to-use asset in its books. The asset is required to be amortized over the life of the asset or the lease term, whichever is less. In the given question, the useful life of 2 years is less than the lease term of 3 years. The value of the ROU asset shall be amortized over a period of 2 years.

Annual amortization = 120,000/2 = $60,000

How well did you know this?
1
Not at all
2
3
4
5
Perfectly
115
Q

The following obligations were among those reported by Fern Village at December 31:
Vendor financing with a term of 10 months when incurred,
in connection with a capital asset acquisition that is not
part of a long-term financing plan $ 150,000
Long-term bonds for financing of capital asset acquisition 3,000,000
Bond anticipation notes due in six months, issued as part
of a long-term financing plan for capital purposes 400,000What aggregate amount should Fern report as general long-term capital debt at December 31?
A
$3,000,000
B
$3,150,000
C
$3,400,000
D
$3,550,000

A

C

Explanation:
Fern should report $3,400,000 as long-term capital debt. This amount is comprised of (1) $3,000,000 of long-term bonds for financing of capital asset acquisition and (2) $400,000 of bond anticipation notes due in six months that were issued as part of a long-term financing plan for capital purposes. Although the bond anticipation notes (BANs) are due in six months, the intent is to refinance the bond anticipation notes on a long-term basis. The vendor financing is not reported as long-term capital debt because it is not part of a long-term financing plan.

How well did you know this?
1
Not at all
2
3
4
5
Perfectly
116
Q

Which format must an enterprise fund use to report cash flow operating activities in the statement of cash flows?
A
Indirect method, beginning with operating income.
B
Indirect method, beginning with change in net position.
C
Direct method.
D
Either direct or indirect method.

A

C

Explanation:
An enterprise fund is one of the governmental proprietary funds. Governments should present a statement of cash flows for proprietary funds based on the provisions of GASB statements. The GASB states the direct method of presenting cash flows from operating activities (including a reconciliation of operating cash flows to operating income) should be used.

How well did you know this?
1
Not at all
2
3
4
5
Perfectly
117
Q

Which event(s) should be included in a statement of cash flows for a governmental entity?

Cash inflow from issuing bonds to finance city hall construction.
Cash outflow from a city utility representing payments in lieu of property taxes.
A
I only
B
II only
C
Both I and II
D
Neither I nor II
A

Explanation:
The correct answer is (B).

Bonds issued to finance city hall construction is recorded in its Capital Projects Fund, which is a Governmental Fund. Governmental Funds are accounted on a modified accrual basis and a Statement of Cash Flows is not presented for Governmental Funds.

Under governmental accounting, a Statement of Cash Flows is prepared only for Proprietary Funds.

A cash flows statement includes receipts and payments resulting only from the Proprietary Fund’s activities. The payments in lieu of property taxes are from an operating activity of an Enterprise Fund (Proprietary).

How well did you know this?
1
Not at all
2
3
4
5
Perfectly
118
Q
Nox City reported a $25,000 net increase in the fund balances for total governmental funds. Nox also reported an increase in net position for the following:
Motor pool internal service fund	$9,000
Water enterprise fund	12,000
Employee pension fund	7,000The motor pool internal service fund provides service to the general fund departments. What amount should Nox report as the change in net position for governmental activities?
A
$25,000
B
$34,000
C
$41,000
D
$46,000
A

Explanation:
The GWS governmental activities column includes the governmental funds plus most internal service funds and capital assets not specific to business-type activities or fiduciary funds. $25,000 + $9,000 = $34,000. The water enterprise fund is a business-type activity. The employee pension fund is a fiduciary fund; it doesn’t appear in the GWS.

How well did you know this?
1
Not at all
2
3
4
5
Perfectly
119
Q

Which fund would report the amount of bonds payable that are due within one year?

A
Governmental funds
B
Proprietary funds
C
Both governmental and proprietary funds
D
Neither governmental nor proprietary funds
A

B

Explanation:
Proprietary funds report the current amounts due for any bonds outstanding on its statement of net position. At the fund level, governmental funds do not report the current portions of principal or interest due.

How well did you know this?
1
Not at all
2
3
4
5
Perfectly
120
Q

Which of the following is a minimum required report for the basic financial statements of a government entity?

A
Fund financial statements.
B
Management's discussion and analysis.
C
Required supplementary information.
D
Budgetary comparison schedules.
A

Explanation:
The correct answer is (A).

The various elements of the basic financial statements of a government entity are:

Government-Wide Financial Statements
Fund Financial Statements
Notes to the Financial Statements
Hence, out of the given options, fund financial statements qualifies as the minimum required report for the basic financial statements of a government entity.

How well did you know this?
1
Not at all
2
3
4
5
Perfectly
121
Q
Programs funded with taxes, grant revenues, and other nonexchange resources are reported in the government-wide statements as which of the following?
A
Social service activities
B
Intergovernmental activities
C
Governmental activities
D
Business-type activities
A

C

Explanation:
Governmental activities include those programs funded with taxes, grant revenues, and other nonexchange resources (not social service activities or intergovernmental activities). Business-type activities are financed in whole or in part by fees charged to external parties for goods and services. Enterprise funds are used to account for these programs.

How well did you know this?
1
Not at all
2
3
4
5
Perfectly
122
Q

For which of the following funds do operating transfers affect the results of operations?

Governmental funds	Proprietary funds
A	No	No
B	No	Yes
C	Yes	Yes
D	Yes	No
A

C

Explanation:
Operating transfers should be recorded in Other Financing Sources Uses or Other Financing accounts and reported after revenues and expenditures or expenses, but before determining the results of operations in the operating statement.

How well did you know this?
1
Not at all
2
3
4
5
Perfectly
123
Q

Eureka City should issue a statement of cash flows for which of the following funds?

Eureka City Hall
Capital Projects Fund	Eureka Water
Enterprise Fund
A	No	Yes
B	No	No
C	Yes	No
D	Yes	Yes
A

A

Explanation:
The capital projects fund is a governmental fund, whereas the enterprise fund is a proprietary fund. A statement of cash flows should be presented for all proprietary funds, but not for governmental funds.

How well did you know this?
1
Not at all
2
3
4
5
Perfectly
124
Q

The following information for the year ended June 30 pertains to a proprietary fund established by Glen Village in connection with Glen’s public parking facilities:

Receipts from users of parking facilities $600,000
Expenditures: Parking meters 410,000
Expenditures: Salaries and other cash expenses 96,000
Depreciation of parking meters 94,000
For the year ended June 30, this proprietary fund should report net income of

A
$0
B
$94,000
C
$96,000
D
$410,000
A

D

Explanation:
Proprietary funds account for their fixed assets in the same manner as commercial enterprises; therefore, the expenditure for the parking meters should be recorded in the fund’s fixed asset accounts. Enterprise funds and internal service funds are the two types of proprietary funds. The fund in question is an enterprise fund because it is a self-supporting fund which provides goods and/or services to the general public.

Parking meter is an so its a fixed asset
Net Income = 600k -96k-94k
=410k

How well did you know this?
1
Not at all
2
3
4
5
Perfectly
125
Q
A local government has the following liabilities on its adjusted trial balance at the end of the current year. What amount would the government report on its balance sheet as total liabilities?
$50,000 accrued payroll
$100,000 bonds payable – current portion
$900,000 bonds payable – long-term portion
$25,000 accounts payable
A
$1,075,000
B
$175,000
C
$100,000
D
$75,000
A

D

Explanation:
The balance sheet is a fund level financial statement and thus uses the modified accrual method of accounting. This means that the $1,000,000 related to the bonds will not be reported, and the answer is the $50,000 in accrued payroll and the $25,000 in accounts payable.

How well did you know this?
1
Not at all
2
3
4
5
Perfectly
126
Q
Land and other real estate held as investments by endowments in a government’s permanent fund should be reported at
A
Historical cost
B
The lower of cost and net realizable value
C
Fair value
D
Fair value less costs of disposal
A

C

Explanation:
Endowments are donations and donations are recorded as revenue at fair value, regardless of their cost to the donor, and reported at fair value at statement dates.

How well did you know this?
1
Not at all
2
3
4
5
Perfectly
127
Q
Property taxes and fines represent which of the following classes of nonexchange transactions for governmental units?
A
Derived tax revenues
B
Imposed nonexchange revenues
C
Government-mandated nonexchange transactions
D
Voluntary nonexchange transactions
A

B

Explanation:
Imposed nonexchange revenues are assessments on non-governmental entities, and include property taxes and fines or forfeitures.

How well did you know this?
1
Not at all
2
3
4
5
Perfectly
128
Q

Which of the following can be reported in a capital projects fund?
A
Bond proceeds for sewer treatment plant upgrades
B
Committed resources to expand water distribution system
C
Restricted, committed, or assigned resources for governmental capital outlays
D
Assigned resources for landfill postclosure care

A

C

Explanation:
Restricted, committed, or assigned resources for governmental capital outlays are reported in a capital projects fund. Sewer services, water utilities, and landfills are usually reported in an enterprise fund because these services are funded with user fees. Enterprise fund capital assets are not reported in a capital projects fund.

How well did you know this?
1
Not at all
2
3
4
5
Perfectly
129
Q

Tang City received land from a donor who stipulated that the land must remain intact, but any income generated from the property may be used for general government services. In which fund should Tang City record the donated land?

A
Special revenue.
B
Permanent.
C
Private-purpose trust.
D
Agency.
A

Explanation:
The correct answer is (B).

The Permanent Fund accounts and reports assets whose principal is restricted and may not be spent, but must be invested on a permanent basis. Since land is restricted by the donor and income generated from the land can be spent, Tang city would record in permanent fund.

Option (a) is incorrect because special revenue fund accounts for and reports specific revenues from earmarked sources that are restricted or committed to be used to finance designated activities other than capital projects and debt service.

Option (c) is incorrect because private-purpose trust fund accounts for resources that are being held for benefit of private persons, organizations or other governments.

Option (d) is incorrect because agency funds accounts for collected amounts that must be transferred to other funds or outsiders.

How well did you know this?
1
Not at all
2
3
4
5
Perfectly
130
Q

The measurement focus of governmental-type funds is on the determination of
Flow of financial resources Financial position
A Yes No
B No Yes
C No No
D Yes Yes

A

D

Explanation:
The governmental fund measurement focus is on determination of financial position and changes in financial position, rather than on net income determination. In governmental funds, the primary emphasis is on the flow of financial resources.

How well did you know this?
1
Not at all
2
3
4
5
Perfectly
131
Q
The following are Boa City's fixed assets:
Fixed assets used in enterprise fund activities	$1,000,000
Infrastructure assets	9,000,000
All other fixed assets	1,800,000What aggregate amount should Boa report in the governmental activities column of the government-wide financial statements?
A
$ 9,000,000
B
$ 10,000,000
C
$10,800,000
D
$11,800,000
A

C

Explanation:
The government-wide financial statements (GWS) reports all general government capital assets (fixed assets plus infrastructure) in the governmental activities column ($9,000 + $1,800 = $10,800). Enterprise funds appear in the business-type activities column in the GWS.

How well did you know this?
1
Not at all
2
3
4
5
Perfectly
132
Q

Users need information on how the government applies restricted resources when they have a choice between using restricted or unrestricted resources to evaluate which of the following?
A
Whether the government made improper payments from the restricted and unrestricted resources
B
Whether the government complied with laws and regulations associated with federal funds
C
How ending balances may be applied to continuation of programs and services in future periods
D
None of the above

A

C

Explanation:
Governments rely on grants and other types of contributions to fund a variety of programs. The note disclosing how the government applies restricted resources when they have a choice between using restricted versus unrestricted resources provides users with information to evaluate how ending balances may be applied to continuation of programs and services in future periods (not whether the government made improper payments or whether the government complied with laws and regulations).

How well did you know this?
1
Not at all
2
3
4
5
Perfectly
133
Q

The mission of the Governmental Accounting Standards Board (GASB) includes which of the following?
A
Providing a framework to educate users and support public policies, invest-ments, and management by those users
B
Establishing standards to identify appropriate performance measures and criteria
C
Outlining best practices for administration of governmental budgets
D
Providing accounting and reporting standards for federal government agencies

A

A

Explanation:
The GASB’s mission includes research and development of standards that will improve accounting and financial reporting for state and local governments. The GASB’s mission includes providing a framework to educate users and support public policies, investments, and management by those users.

How well did you know this?
1
Not at all
2
3
4
5
Perfectly
134
Q

A county’s balances in the general fund included the following:

Appropriations	$435,000
Encumbrances	18,000
Expenditures	164,000
Vouchers payable	23,000
What is the remaining amount available for use by the county?
A
$230,000
B
$248,000
C
$253,000
D
$271,000
A

C

Explanation:
Funds available for use = Appropriations (budgeted) - Encumbrances - Expenditures ( $435,000 - $18,000 - $164,000 ) = $253,000. Option (a) is incorrect because vouchers payable is excluded in available for funds calculations. Option (b) is incorrect because encumbrances are not excluded, instead vouchers payable are excluded from appropriations. Option (d) is incorrect because encumbrances should be excluded from appropriations to arrive at funds available.

How well did you know this?
1
Not at all
2
3
4
5
Perfectly
135
Q

Lys City reports a compensated absences liability in its combined balance sheet. The salary rate used to calculate the liability should normally be the rate in effect
A
When the unpaid compensated absences were earned.
B
When the compensated absences were earned or are to be paid, or at the balance sheet date, whichever results in the lowest amount.
C
At the balance sheet date.
D
When the compensated absences are to be paid.

A

C

Explanation:
The compensated absences liability ordinarily is measured using the pay rates in effect at the balance sheet date.

Liabilities for compensated absences should be inventoried at the end of each accounting period and adjusted to current salary costs.

How well did you know this?
1
Not at all
2
3
4
5
Perfectly
136
Q

Which of the following funds of a governmental unit uses the same basis of accounting as the special revenue fund?

A
Internal service.
B
Capital projects.
C
Nonexpendable trust.
D
Enterprise.
A

Explanation:
The correct answer is (B)
General Funds, Special Revenue Funds, Capital Projects Fund, Debt Service Fund, and Permanent Fund use modified accrual accounting.

Under the modified accrual basis of accounting, revenue is recognized in the period it is measurable and available to spend. Costs are recorded in the period that the obligation to pay them arises, whether this is before or after the period in which the government is actually using the assets or services.

The Capital Projects Fund uses the same basis of accounting as the Special Revenue Fund.

How well did you know this?
1
Not at all
2
3
4
5
Perfectly
137
Q

Which basis of accounting is required for a city’s government-wide financial statements?

A
Cash.
B
Modified Cash.
C
Modified Accrual.
D
Accrual.
A

Explanation:
The correct answer is (D).

A city’s government-wide financial statements are required to be completed on the accrual basis of accounting.

How well did you know this?
1
Not at all
2
3
4
5
Perfectly
138
Q

Which of the following is not within the scope of the GASB mission for standard setting?
A
Conceptual framework for accounting and reporting standards
B
Goals and objectives of state and local governmental services
C
External financial reporting
D
Assist users in assessing the level of services that can be provided by governmental entities

A

B

Explanation:
Goals and objectives for governmental programs and services should be established by management, elected officials, and the government’s stake-holders.

How well did you know this?
1
Not at all
2
3
4
5
Perfectly
139
Q

Which of the following is the purpose of using major fund reporting in the fund statements?
A
To highlight special revenue funds
B
To distinguish special revenue funds from debt service funds
C
To report the government’s largest funds rather than fund types
D
To provide more details about governmental funds

A

A

Explanation:
GASB Statement No. 34 shifted away from fund type reporting to highlight the government’s larger funds. Users do not have much interest in fund types, but they are focused on the government’s larger funds. Major fund reporting concentrates on fewer details and combines the government’s smaller funds to simplify reporting and focus on significant activities. While the determination of major funds is based on a mathematical relationship, management does have the ability to include other significant funds.

How well did you know this?
1
Not at all
2
3
4
5
Perfectly
140
Q
The basis of accounting for a capital projects fund is the
A
Cash basis.
B
Accrual basis.
C
Modified cash basis.
D
Modified accrual basis.
A

D

Explanation:
The modified accrual basis is the appropriate basis of accounting for governmental funds (i.e., General, Special Revenue, Capital Projects, Permanent, and Debt Service Funds).

How well did you know this?
1
Not at all
2
3
4
5
Perfectly
141
Q
Which of the following bases of accounting should a government use for its proprietary funds in measuring financial position and operating results?
Modified accrual basis	Accrual basis
A	No	Yes
B	No	No
C	Yes	Yes
D	Yes	No
A

A

Explanation:
The appropriate basis of accounting for proprietary and fiduciary funds is the accrual basis, under which revenues are recognized in the accounting period in which they are earned and become measurable.

142
Q
Which basis of accounting is required for a city’s government-wide financial statements?
A
Cash
B
Modified cash
C
Modified accrual
D
Accrual
A

D

Explanation:
GASB 34 requires the economic resources measurement focus and accrual basis of accounting for all amounts in the government-wide financial statements.

143
Q

Which of the following would be reported using the full accrual method of accounting?

A
Governmental activities
B
Business-type activities
C
Neither governmental activities nor business-type activities
D
Both governmental activities and business-type activities
A

D

Explanation:
Both governmental activities and business-type activities are included on the face of the government-wide financial statements. The government-wide financial statements use the full accrual method of accounting, which means that D is the correct answer. All amounts on the government-wide financial statements are reported using the accrual method.

144
Q
Valley Town's public school system is administered by a separately elected board of education. The board of education is not organized as a separate legal entity and does not have the power to levy taxes or issue bonds. Valley's city council approves the school system's budget. How should Valley report the public school system's annual financial results?
Discrete presentation	Blended
A	Yes	Yes
B	Yes	No
C	No	Yes
D	No	No
A

C

Explanation:
GASB 39 provides additional guidance to determine whether certain entities for which a primary government is not accountable financially must be reported as component units based on the nature and significance of their relationship with the primary government. Legally separate, tax-exempt entities that meet all three criteria must be presented discretely as component units: (1) economic resources received or held by the separate entity are totally (or nearly totally) for the direct benefit of the primary government, its component units, or its constituents; (2) the primary government, or its component units, is entitled to a majority of the economic resources received or held by the separate entity; and (3) the economic resources received or held by an individual entity that the specific primary government, or its component units, is entitled to, or has the ability to otherwise access, are significant to that primary government.

145
Q

Berry Township has adopted GASB Statement No. 34, Basic Financial Statements and Management’s Discussion and Analysis for State and Local Governments. Berry’s eligible infrastructure assets are exempt from depreciation if the modified approach is used. Which of the following are requirements of the modified approach?

The entity performs condition assessments of eligible assets and summarizes the results using a measurement scale.
The entity annually estimates the amount to preserve the eligible assets at an established and disclosed condition level.
The entity assesses asset conditions in comparison to condition levels established by the National Association of Public Works Engineers or a comparable organization.
A
I
B
I and II
C
I and III
D
I, II, and III

A

idk maybe B

Explanation:
Infrastructure assets that are part of a network or a sub-system of a network (eligible infrastructure assets) are not required to be depreciated, if the government: documents that eligible infrastructure assets are preserved approximately at (or above) a condition level established and disclosed by the government; has an up-to-date inventory of eligible infrastructure assets; consistently performs condition assessments of eligible assets at least tri-annually and summarizes the results using a measurement scale; and annually estimates the amount to preserve the eligible assets at a condition level established and disclosed by the government. .

146
Q

Which of the following is required by GASB No. 34, Basic Financial Statements and Management’s Discussion and Analysis for State and Local Governments?

Governmental activities information using the economic resources measurement focus in the government-wide financial statements.
Governmental fund information included in the statement of cash flows.
A
I only
B
II only
C
Both I and II
D
Neither I nor II
A

A

Explanation:
All funds use the economic resources measurement focus and accrual basis of accounting in the government-wide statements.Only business-type activities appear in the statement of cash flows. (The governmental funds pose problems for developing meaningful information for a government-wide cash flows statement.) .

147
Q

A government’s assets include inventory of $2 million, roads constructed for $25 million with accumulated depreciation of $10 million, and equipment acquired for $5 million with accumulated depreciation of $1 million. Its liabilities include an outstanding balance of $5 million for bonds payable issued to construct the roads and a $1 million short-term loan for inventory purchases. What amount should be reported as the net investment in capital assets in the government-wide statement of net position?

A
$26 million
B
$25 million
C
$14 million
D
$10 million
A

Explanation:
The correct answer is (C).

$14,000,000 should be reported as the net investment in capital assets in the government-wide statement of net position. Net assets on the government-wide statement of financial position are divided into - Restricted, Unrestricted and Net Investment in Capital Assets.

Assets:

Roads Constructed

$25,000,000

Accum. Depreciation

($10,000,000)

$15,000,000

Equipment

$5,000,000

Accum. Depreciation

($1,000,000)

$4,000,000

Liabilities:

Bonds Payable

$(5,000,000)

Net Investment in Capital Assets

$14,000,000

Note: Inventory is not a capital asset and therefore both the inventory and liability related to the purchase of inventory is also excluded.

148
Q

Which of the following is a unique feature of governmental financial statements?
A
Demonstrating that the government produces a “bottom line profit”
B
Highlighting the government’s earnings per share for public utilities
C
The government’s relationship with related entities
D
Identifying how public resources are acquired and used

A

D

Explanation:
Governmental financial statements provide accountability for public resources. This objective is not addressed in commercial accounting standards. The “bottom line profit” is not a major focus for the governmental operating statement. Governmental entities, even enterprise utility systems, are not financed with public stock offerings, therefore, earnings per share is not a relevant measure for these governmental entities. Financial reporting standards for both governmental and commercial accounting include provisions for reporting related organizations and disclosing the primary entity’s responsibilities for these reporting entities.

149
Q

Which of the following is the primary objective for accounting principles for internal service funds?
A
Centralized management of services and cost recovery for those operations
B
Recovery of general capital assets
C
Issuance of revenue bonds to fund capital expansions and improvements
D
Accumulation of resources to repay bonded indebtedness

A

A

Explanation:
As the name implies, internal service funds provide a centralized management of shared services and allocate the costs of those services across the departments or agencies who participate. General capital assets are not included as a cost element in internal service funds, only the assets used directly in the operations. Enterprise funds are more likely to issue revenue debt. Accumulation of resources to repay bonded indebtedness describes a governmental debt service fund.

150
Q

The billings for transportation services provided to other governmental units are recorded by the internal service fund as

A
Transportation appropriations.
B
Operating revenues.
C
Interfund exchanges.
D
Intergovernmental transfers.
A

B

Explanation:
Internal service funds are accounted for in a similar manner to enterprise funds. A revenues or billings to others account is used for services provided to other departments or other governments. Appropriations accounts are budgetary accounts. Interfund exchanges would be between departments in the same governmental unit. Intergovernmental transfers are used when the same entity is doing the accounting for both governments.

151
Q

The following information pertains to Grove City’s interfund receivables and payables at December 31:

Due to special revenue fund from general fund $10,000
Due to custodial fund from special revenue fund 4,000
In Grove’s special revenue fund balance sheet at December 31, how should these interfund amounts
be reported?

A
As an asset of $6,000.
B
As a liability of $6,000.
C
As an asset of $4,000 and a liability of $10,000.
D
As an asset of $10,000 and a liability of $4,000.
A

D

Explanation:
The interfund receivables and payables of a governmental fund should not be netted. Therefore, the special revenue fund should report an asset of $10,000 for the amount due from the general fund and a liability of $4,000 for the amount due to the custodial fund.

152
Q
A major exception to the general rule of expenditure accrual for governmental units relates to unmatured
Principal of general
long-term debt	Interest on general
long-term debt
A	Yes	Yes
B	Yes	No
C	No	Yes
D	No	No
A

A

Explanation:
A major exception to the general rule of expenditure accrual relates to unmatured principal and interest on general long-term debt. This deals with the criterion related to the expenditure recognition on debt known as the “when due” criterion. Entities that budget cash outflows for debt when they legally become due include budget appropriations for debt in the year in which the cash outflow occurs. Because the financial flow of funds to make payment has not been budgeted for, interest and principal payments are not subject to accrual.

153
Q

Which of the following local government funds uses the accrual basis of accounting?

A
Enterprise
B
Debt service
C
Capital projects
D
Special revenue
A

A

Explanation:
The basis of accounting used depends on the nature of the fund. The modified accrual basis is used in the governmental fund type statements (i.e., general, special revenue, capital projects, and debt service funds). The accrual basis is used in proprietary fund statements (i.e., internal service and enterprise funds), where revenues and expenses are recorded and net income (loss) is reported. The accrual basis is also used in fiduciary fund statements.Option (b), (c) and (d) are incorrect because these funds are part of governmental funds which uses modified accrual accounting.

154
Q

Land and other real estate held as investments by endowments in a government’s permanent fund should be reported at?

A
Historical cost.
B
The lower of cost and net realizable value.
C
Fair value.
D
Fair value less costs of disposal.
A

Explanation:
The correct answer is (C).

Land and other real estate held as investments by endowments should always be reported at fair value. And changes in fair value during the period should be reported as investment income/loss.

(A) and (B) are incorrect as per the above explanation.

(D) is incorrect because the cost of disposal is not required to be deducted from fair value.

155
Q

An enterprise fund would be used when the governing body requires that

Accounting for the financing of an agency’s services to other government departments be on a cost-reimbursement basis.
User charges cover the costs of general public services.
Net income information be provided for an activity.
A
I only
B
I and II
C
I and III
D
II and III

A

D

Explanation:
II and III. An Enterprise Fund is a Proprietary Fund. Its measurement focus is on income determination and its customers are the general public.

“I” is incorrect because services provided by one governmental unit to other governmental units on a cost-reimbursement basis are accounted for in an Internal Service Fund.

156
Q

The debt service transactions of a special assessment bond issue for which the government is not obligated in any manner should be reported in the

A
Custodial Fund
B
Enterprise Fund
C
Special Revenue Fund
D
Long-Term Debt Account Group
A

Explanation:
The correct answer is (A).

Although the special assessment debt for which the government is not obligated in any manner is not reported as government debt, the government usually acts as a debt service agent for the special assessment district.

The government has a fiduciary responsibility to collect the special assessments and to remit the collections to the bondholders when the debt service payments come due.

However, if collections are insufficient to cover required debt service payments, the government is not obligated to pay the difference and does not intend to do so.

Thus, the government is acting in a custodial capacity with respect to the debt service transactions, and these transactions should be accounted for in a Custodial Fund.

157
Q

Fish Road property owners in Sea County are responsible for special assessment debt that arose from a storm sewer project. If the property owners default, Sea has no obligation regarding debt service, although it does bill property owners for assessments and uses the money it collects to pay debt holders. What fund type should Sea use to account for these collection and servicing activities?

A
Custodial
B
Debt service
C
Expendable trust funds
D
Capital projects
A

Explanation:
The correct answer is Option (A).

Sea is acting as a custodian of the money collected and the Custodial fund is used to account for the custodial activities of the governmental unit.

The debt service fund is used to account for the repayment of the general long-term debt.

The capital projects fund accounts for the project itself, not the repayment of debt.

158
Q

What is the major difference between an exchange transaction and a non-exchange transaction for governmental units?
A
The relationship between the amount of value given and received.
B
Time requirements and whether the transaction is required by law.
C
Purpose restrictions placed upon fund balances.
D
Whether resources acquired can be further exchanged.

A

A

Explanation:
There are various types of non-exchange transactions for governmental units; imposed non-exchange transactions, government-mandated non-exchange transactions, and voluntary non-exchange transactions. The difference between an exchange transaction and a non-exchange transaction is the relationship between the amount of value given and received. In an exchange transaction the presumption is the amount of value given and received is the same. A nonexchange transaction does not involve an exchange of equal value.

159
Q
Which of the following distinguishes the use of a special revenue fund compared to a private-purpose trust fund?
A
Types of revenues
B
Types of program costs
C
Intended beneficiaries
D
Use of cash basis accounting
A

C

Explanation:
When beneficiaries are the general public, the resources are accounted for and reported in a special revenue fund. When there are specific third-party beneficiaries, a private-purpose trust is more appropriate. Program costs do not affect the type of fund used for governmental activities. Cash basis of accounting is not a factor for determining the type of fund used for a specific activity.

160
Q
Gem City's internal service fund received a residual equity transfer of $50,000 cash from the general fund. This $50,000 transfer should be reported in Gem's internal service fund as a credit to
A
Revenues.
B
Other Financing Sources.
C
Transfers In.
D
Contributed Capital.
A

C

Explanation:
Internal Service Fund journal entry: 
dr. Cash, 
           cr. Transfer-in, and 
General Fund journal entry: 
dr. Transfer out, 
             cr. Cash

Interfund transfers are flows of assets (such as cash or goods) without equivalent flows of assets in return and without a requirement for repayment. This category includes payments in lieu of taxes that are not payments for, and are not reasonably equivalent in value to, services provided. In governmental funds, transfers should be reported as other financing uses in the funds making transfers and as other financing sources in the funds receiving transfers. In proprietary funds, transfers should be reported after nonoperating revenues and expenses as transfers in or transfers out

161
Q

Which of the following is included in the mission of the GASB?
A
Funding policies for public pension funds
B
Performance measurement standards
C
Budgetary strategies for public entities
D
Educate the public, about governmental financial issues

A

D

Explanation:
The GASB has a broad mission to develop standards that provide useful information for users of financial reports as well as educating the public about governmental financial issues. Management, not GASB, determines the funding policies for public pension plans. Performance measurement standards are not within the scope of governmental accounting and reporting guidance. Management, not GASB, selects the appropriate budget strategy for the governmental organization.

162
Q
Financial statements for which fund type generally report net position?
A
Capital projects.
B
Expendable pension trust.
C
Special revenue.
D
Enterprise
A

Explanation:
The enterprise fund generally reports net position. The capital projects and special revenue funds are governmental-type funds, and thus report a fund balance. Expendable trusts (under the outgoing reporting model) use fund balance accounts also.

163
Q

Seaside County collects property taxes levied within its boundaries and receives a 1% fee for administering these collections on behalf of the municipalities
located in the county. In the year, Seaside collected $1,000,000 for its municipalities and remitted $990,000 to them after deducting fees of $10,000. In the
initial recording of the 1% fee, Seaside’s custodial fund should credit

A
Fund balance-agency fund, $10,000.
B
Fees earned-agency fund, $10,000.
C
Due to Seaside County general fund, $10,000.
D
Revenues control, $10,000.
A

C

Explanation:
Custodial funds do not have revenues or a fund balance. Seaside County’s tax custodial fund should record all levies as follows:

Taxes Receivable for Other Units XX
Due to Other Funds and Units XX

The fund should record the taxes collected for the county’s municipalities during the period as follows:

Cash 1,000,000
Taxes Receivable for Other Units 1,000,000

Upon the collection of the taxes, the fund should make the following entry to record the fund’s liability for the distribution of the collections:

Due to Other Funds and Units 1,000,000
Due to Municipalities 990,000
Due to General Fund 10,000

164
Q

Fiduciary funds are accounted for using which of the following?
A
Accrual basis of accounting
B
Modified accrual basis of accounting
C
Flow of economic capital basis of accounting
D
Accrual or modified approach, whichever is most appropriate

A

A

Explanation:
All fiduciary funds are accounted for on the accrual basis in essentially the same manner as proprietary funds. Governments do not have a choice of using modified accrual basis of accounting for fiduciary funds. The modified accrual method is used for governmental funds only. The flow of economic capital is not a basis recognized for governmental accounting purposes.

165
Q
The statement of activities of the government-wide financial statements is designed primarily to provide information to assess which of the following?
A
Operational accountability.
B
Financial accountability.
C
Fiscal accountability.
D
Functional accountability.
A

A

Explanation:
The government-wide statements, which include the statement of activities, are designed to provide operational accountability by showing an economic longterm view of a government’s ability to provide services. Operational accountability includes the periodic economic cost of the services provided.

166
Q

Which of the following characteristics trigger unique accounting standards for governmental entities?
A
Governmental ownership of utilities such as water, sewer, or electric services
B
Democratic structure for state and local governmental entities
C
Fund accounting and budgetary reporting
D
Profit motive in governmental funds

A

C

Explanation:
Fund accounting and budgetary reporting are key elements for demonstrating accountability for public resources and provide cornerstones for governmental financial reporting. They are the characteristics that trigger unique accounting standards for governmental entities.

167
Q
Roy City received a gift, the principal of which is to be invested in perpetuity with the income to be used to support the local library. In which fund should this gift be recorded?
A
Permanent fund.
B
Investment trusts fund.
C
Private-purpose trusts fund.
D
Special revenue fund.
A

A

Explanation:
A permanent fund is used to account for nonexpendable resources that may be used for the government’s programs to generate and disperse money, to benefit the reporting entity or its citizens, such as the library in this question. The name of the fund comes from the purpose of the fund: a sum of equity used to permanently generate payments to maintain some financial obligation. A fund can only be classified as a permanent fund if the money is used to report the status of a restricted financial resource. The resource is restricted in the sense that only earnings from the resource are used and not the principal.

168
Q

Which of the following describes the uses of resources in the fiduciary funds?
A
By the government for any program it chooses
B
For the purposes designated by the trust agreement or law, or returned to the owners
C
Only at the end of the year
D
To pay for pension funds only

A

Explanation:
When acting in a fiduciary capacity, the government can spend the fiduciary funds only for specific purposes as spelled out in the law or trust agreement or returned to the owners. Fiduciary funds are restricted for use for specific purposes as described by the trust agreement or law. Fiduciary fund transactions occur all year, not just at the end of the year. Pension funds aren’t the only kind of fiduciary funds.

169
Q
On January 2, City of Walton issued $500,000, 10-year, 7% general obligation bonds. Interest is payable annually, beginning January 2 of the following year. What amount of bond interest is Walton required to report in the statement of revenue, expenditures, and changes in fund balance of its governmental funds at the close of this fiscal year, September 30?
A
$0
B
$17,500
C
$26,250
D
$35,000
A

A

Explanation:
Interest on general long-term debt, usually accounted for in debt service funds, normally are recorded as an expenditure on its due date rather than being accrued prior to its due date.

Only the interest cost actually paid in the period is included in interest expense. Accordingly, this answer is correct. Since no interest was paid during the year, interest expense is $0.

170
Q

On March 1, Wag City issued $1,000,000, ten-year, 6% general obligation bonds at par with no bond issue costs. The bonds pay interest September 1 and March 1. What amount of interest expense and bond interest payable should Wag report in its government-wide financial statements at the close of the fiscal year on December 31?
A
Interest expense, $50,000; interest payable, $20,000
B
Interest expense, $50,000; interest payable, $0
C
Interest expense, $60,000; interest payable, $10,000
D
Interest expense, $30,000; interest payable, $0

A

A

Explanation:
The government-wide financial statements use normal accrual accounting. The bond interest should be the $1,000,000 times 0.03 (because the 6% interest is paid every 6 months) for a total of $30,000 on each due date. The interest expense would be the $30,000 for the period March 1 to September 1 plus $20,000 for the four month period September 1 to December 31 ($30,000 × 4/6 = $20,000) for a total of $50,000. Because the $20,000 for the last four months will not be paid until the next March 1, it would be the interest payable at the close of the fiscal year on December 31.

171
Q
South City School District has a separate elected governing body that administers the public school system. The district's budget is subject to the approval of the city council. The district's financial activity should be reported in the City's financial statements by
A
Blending only.
B
Discrete presentation.
C
Inclusion as a footnote only.
D
Either blending or inclusion as a footnote.
A

B

Explanation:
Component units should be presented discretely unless either (a) the components unit’s governing body is substantively the same as the governing body of the primary government, or (b) the component unit provides services almost entirely to the primary government, or almost exclusively benefits the primary government although it does not provide services directly to it

172
Q

If individual component units are consolidated on combined governmental statements, the data for the individual units must be presented in which of the
following?

A
Combining statements or in separately issued reports
B
Consolidated statements or in separately issued reports
C
The notes to the financial statements or in combining statements
D
The notes to the financial statements or in separately issued reports

A

C

Explanation:
If the individual component units are consolidated in any fashion on the combined statements, the data for individual units must be presented in either the
notes to the financial statements or in combining statements in the basic financial statements (but not in consolidated statements or a separately issued report).

173
Q

A government has the following liabilities at the end of the year:

General obligation bonds

$1,500,000

Compensated Absences

$120,000

Salaries payable

$40,000

What amount of liabilities should be reported in the governmental activities column of the government-wide statement of net position?

A
$40,000
B
$160,000
C
$1,540,000
D
$1,660,000
A

Explanation:
The correct answer is (D).

Government-wide financial statements are the second section of the government financial report. Prepared on the economic resources measurement focus and accrual basis of accounting. The amount of liabilities reported in the governmental activities column of the government-wide statement of net position is $1,660,000.

Summary

Amount

General obligation bonds

$1,500,000

Compensated Absences

$120,000

Salaries payable

$40,000

Total Liabilities

$1,660,000

174
Q

Several disclosures in the Significant Accounting Policies section of governmental financial statements describe which of the following?
A
How the government complied with applicable laws and regulations
B
Why the government exceeded its statutory budgetary authority
C
Accounting policies for recognizing revenues and expenditures
D
Why the notes are not necessary for a complete understanding of the government’s financial statements

A

C

Explanation:
The Significant Accounting Policies section includes a number of explanations about what policies the government has adopted for recognition, such as how revenues are recognized, asset capitalization, program revenues, and the government’s definition of operating transactions. Disclosures do not describe how the government complied with applicable laws and regulations, why the government exceeded its statutory budgetary authority, or why the notes are necessary.

175
Q

Which of the following transactions is an expenditure of a governmental unit’s general fund?

A
Contribution of enterprise fund capital by the general fund
B
Operating subsidy transfer from the general fund to an enterprise fund
C
Routine employer contributions from the general fund to a pension trust fund
D
Transfer from the general fund to a capital projects fund

A

C

Explanation:
General funds use modified accrual accounting. Both capital purchases and operating expenditures are considered spending of funds and are treated as current year expenditure. Routine employer contribution to a pension trust fund is an expenditure. Option (a) is incorrect because contribution of enterprise fund capital by the general fund is not an expenditure. Option (b) is incorrect because operating transfers are reported as other financing uses under modified accrual accounting. Option (d) is incorrect because transfers out is treated as other financing uses and not expenditure

176
Q

Zephyr City has adopted GASB Statement No. 34, Basic Financial Statements and Management’s Discussion and Analysis for State and Local Governments. Which activities and basis of accounting must appear in Zephyr’s statement(s) of cash flows?

Business-type activities on the accrual basis in the fund financial statements
Government-type activities on the modified accrual basis in the fund financial statements
Business-type and government-type activities on the accrual basis in the government-wide financial statements
A
I only
B
II only
C
III only
D
I and III only

A

A

Explanation:
Governmental-type funds pose problems for developing a meaningful government-wide cash flows statement. Only funds for business-type activities are required in the statement of cash flows.

177
Q

The budget for the City of Goodville for the year ending December 31 was adopted and recorded on January 2 of the same year. After recording the budget, the accounting records showed a debit balance of $50,000 in the Budgetary Fund Balance account. What does this indicate?

A
Appropriations are $50,000 greater than estimated revenues
B
Estimated revenues are $50,000 greater than appropriations
C
Appropriations are $50,000 greater than revenues
D
Revenues are $50,000 greater than appropriations

A

Explanation:
The correct answer is (A).

The accounting records showed a debit balance of $50,000 in the Budgetary Fund Balance account. This means that the appropriations are $50,000 greater than estimated revenues. When Goodville adopted its budget, estimated revenues are debited and appropriations are credited. A debit or credit to the budgetary fund balance is used when the budget does not balance. The budgetary fund balance is debited ($50,000) when appropriations are greater than estimated revenues. See the example below.

Estimated Revenue Control $40,000
Budgetary Fund Balance $50,000
Appropriations Control $90,000

178
Q

GASB Statement No. 34, Basic Financial Statements and Management’s Discussion and Analysis for State and Local Governments, requires presentation of a budgetary comparison schedule in required supplementary information. Which of the following must this schedule include?

Actual inflows, outflows, and balances, along with a final budget
Original budget
A separate column to report the variances between the final budget and actual amounts
A
I only
B
I and II only
C
I and III only
D
I, II, and III
A

B

Explanation:
The budgetary comparison schedule includes the original and final budgets as well as actual inflows, outflows, and balances, stated on the government’s budgetary basis of accounting, with a reconciliation between the budgetary and GAAP information. The schedule may have the same format as the budget documents or the statement of revenues, expenditures, and changes in fund balances. A column reporting the variances between the final budget and actual amounts is encouraged, but not required.

179
Q
Which of the following is one of the three standard sections of a governmental comprehensive annual financial report?
A
Investment.
B
Actuarial.
C
Statistical.
D
Single audit.
A

Explanation:
The comprehensive annual financial report contains the following sections: Introduction Section; Financial Section; and the Statistical Section.

180
Q

The City of Curtain had the following interfund transactions during the month of May:

Billing by the internal service fund to a department financed by the general fund, for services rendered in the amount of $5,000.

Transfer of $200,000 from the general fund to establish a new enterprise fund.

Routine transfer of $50,000 from the general fund to the debt service fund. What was the total reciprocal interfund activity for Curtain during May?

A
$5,000
B
$55,000
C
$200,000
D
$255,000
A

A

Explanation:
Quasi-external transactions payment from one fund to another for goods or services that are being provided. These are reciprocal transactions, involving an earnings process, unlike the operating and residual equity transfers, in which transfer is made without any benefit received by the transferor. The only transaction which is the billing by the internal service fund for services rendered for $5,000 is the reciprocal inter-fund activity. Option (b) is incorrect because routine transfer is an operating transfer and not reciprocal transaction. Option (c) is incorrect because there is no benefit received in return, it is a transfer to set up new fund. Option (d) is incorrect because operating transfer and transfer to set up new fund are both non-reciprocal transfers, there is no benefit received in return.

181
Q

Revenues are recorded in governmental funds when they are both measurable and available for use. Which of the following time periods is typically used when determining if a revenue is available for use?

A
30 days after period end
B
45 days after period end
C
60 days after period end
D
90 days after period end
A

C

Explanation:
The rule of thumb for measuring available revenues in the modified accrual method of accounting is to use revenues received within 60 days after the end of the reporting period.

182
Q

Elm City contributes to and administers a single-employer defined benefit pension plan on behalf of its covered employees. The plan is accounted for in a pension trust fund. Actuarially determined employer contribution requirements and contributions actually made for the past three years, along with the
percentage of annual covered payroll, were as follows:

Contribution made Actuarial requirement
Amount Percent Amount Percent

Year 3 $11,000 26 $11,000 26
Year 2 5,000 12 10,000 24
Year 1 None None 8,000 20
What account should be credited in the pension trust fund to record the year 3 employer contribution of $11,000?

A
Revenues control
B
Other financing sources control
C
Due from special revenue fund
D
Pension benefit obligation
A

A

Explanation:
All pension trust fund contributions and earnings are accounted for as fund revenues. The pension trust fund makes the following entry to record employer contributions:

Cash 11,000
Revenues control—employer contribution 11,000

183
Q

Which of the following information is needed to prepare the budgetary comparison schedules for a local government?
A
Original budget
B
Explanation of variances
C
Description of the local government’s budgeting process
D
Computation of variances from budget to actual

A

A

Explanation:
A budgetary comparison schedule is presented on the budgetary basis of accounting for the general fund and for each major special revenue fund that has a legally adopted annual budget. The schedule presents both the original and final budget, as well as actual inflows, outflows, and balances. A local government would need the original budget to prepare any budgetary comparison schedules. An explanation of variances would come after the computation of variances from budget to actual which comes after the budgetary comparison schedule is prepared. A description of the local government’s budgeting process is not needed to have a budget and produce a budgetary comparison schedule.

184
Q

Which of the following items is recognized for governmental activities in the government-wide statement of activities and not the statement of revenues, expenditures, and changes in fund balance for governmental funds?

A
Transfers between governmental funds
B
Property tax revenue for an amount deferred because it was not available
C
A state grant awarded and received for road repairs that were completed this fiscal year
D
Salaries payable at the end of the current year that will be paid at the beginning of the subsequent year

A

Explanation:
The correct answer is (B).

Property tax revenue for an amount deferred because it was not available should only be included for governmental activities in the government-wide statement of activities. Property tax revenue that is not currently available is not reported at the governmental fund level but is reported in the government-wide statement. This would be a reconciliation item between the governmental fund balance and the government-wide statement of net position.

185
Q

Smith City levied property taxes totaling $1,000,000 during the current year, of which $150,000 was collected within 60 days after year end. Smith City expects to collect 99% of the total levy. What amount should Smith City record as property tax revenue on its statement of activities?

A
$1,000,000
B
$990,000
C
$850,000
D
$150,000
A

B

Explanation:
The statement of activities is part of the government-wide financial statements, meaning that the revenue reported would be calculated using the accrual method. For this reason, the amount collected within 60 days does not apply to this question. Smith City will report $990,000, which is the 99% of the levy that it expects to collect.

186
Q

For governmental accounting, after determining whether a component unit is legally separate, the next step is to determine which of the following?
A
Whether or not the primary government elects a voting majority of the unit’s board
B
Whether or not the primary government appoints a voting majority of the unit’s board
C
Whether or not the voters of the jurisdiction elect the unit’s board
D
Whether or not the voters of the jurisdiction appoint the unit’s board

A

B

Explanation:
For those potential component units that are legally separate, the next step is to determine whether or not the primary government (not the voters) appoints (not elects) a voting majority of the unit’s board.

187
Q

Which of the following is a required part of a local government’s management’s discussion and analysis (MD&A) as part of its financial statements?

A
The MD&A should be presented with other required supplementary information.
B
The MD&A should compare current-year results to the prior year with emphasis on the current year.
C
The MD&A should include an analysis for each fund.
D
The MD&A should present condensed financial information from the fund financial statements.

A

Explanation:
The correct answer is (B).

Management discussion and analysis (MD&A) is the portion of a public company’s annual report in which management addresses the company’s performance over the previous twelve months. It is a major part of the financial statement, not just supplementary material. MD&A covers major points but does not go into details for each fund. MD&A is not for the purpose of outlining condensed financial information from the fund financial statements. As part of MD&A, there is a requirement to compare current-year results to the prior year with an emphasis on the current year.

(A) is incorrect because MD&A is presented separately and RSI is other than MD&A.

(C) is incorrect because MD&A includes analysis of overall financial position and results of operation including reasons for changes from the prior year; not for each fund.

(D) is incorrect because MD&A should present condensed financial information derived from government-wide financial statements, not from fund financial statements.

188
Q

Elliott Township purchased a firetruck costing $1,000,000 in its general fund on January 1 of the current year. The truck had a useful life of 10 years and the Township has a policy of taking a full year’s depreciation in the first year. How much depreciation related to the truck would the Township report on its statement of revenues, expenditures, and changes in fund balance?

A
$1,000,000
B
$100,000
C
$10,000
D
$0
A

D

Explanation:
The statement of revenues, expenditures, and changes in fund balance is a fund-level financial statement for the governmental funds and uses the modified accrual method of accounting. The $1,000,000 for the firetruck would have been reported as an expenditure, and the truck itself would not have been capitalized at the fund level. Therefore, no depreciation would be reported on the statement of revenues, expenditures, and changes in fund balance.

189
Q

Which of the following are required financial statements for fiduciary funds?
A
Statement of fiduciary net position and statement of revenues, expenditures, and changes in net position
B
Statement of fiduciary net position and statement of changes in fiduciary net position
C
Balance sheet and statement of revenues, expenditures, and changes in net position
D
Balance sheet, operating statement, and statement of cash flows

A

B

Explanation:
Fiduciary funds do not use the major fund reporting outlined for govern-mental and proprietary funds. The fiduciary fund statements are focused on the fund type and the specific reporting requirements for each type. Fiduciary funds should present the statement of fiduciary net position and the statement of changes in fiduciary net position in accordance with the guidance for each type of fund.

190
Q

Where does GASB Statement No. 34, Basic Financial Statements and Management’s Discussion and Analysis for State and Local Governments, require fund financial statements for major funds to be presented?
A
Before the notes to the financial statements
B
In the notes to the financial statements
C
After the notes to the financial statements, before other required supplementary information
D
After the notes to the financial statements and required supplementary information (RSI)

A

A

Explanation:
The major fund financial statements are presented before the notes to the financial statements, as part of the basic financial statements, not RSI. The basic financial statements should be preceded by MD&A, which is required supplementary information. The basic financial statements should include (1) the government-wide statements that display information about the reporting government as a whole, except for its fiduciary activities, (2) fund financial statements for the primary government’s governmental, proprietary, and fiduciary funds should be presented after the government-wide statements, and notes to the financial statements. Except for MD&A, required supplementary information, including the required budgetary comparison information, should be presented immediately following the notes to the financial statements.

191
Q

For the budgetary year, Maple City’s general fund expects the following inflows of resources:

Property taxes, licenses, and fines $9,000,000
Proceeds of debt issue 5,000,000
Interfund transfers for debt service 1,000,000
In the budgetary entry, what amount should Maple record for estimated revenues?

A
$9,000,000
B
$10,000,000
C
$14,000,000
D
$15,000,000
A

A

Explanation:
The general fund records the expected $9,000,000 of inflows of resources for property taxes, licenses, and fines as estimated revenues in the entry to record the adoption of the budget. In this same entry, the expected inflows of resources from the proceeds of the debt issue and the interfund transfers for debt service are recorded as other financing sources.

192
Q
Elm City issued a purchase order for supplies with an estimated cost of $5,000. When the supplies were received, the accompanying invoice indicated an actual price of $4,950. What amount should Elm debit (credit) to the reserve for encumbrances after the supplies and invoice were received?
A
$ (50)
B
$ 50
C
$4,950
D
$5,000
A

D

Explanation:
When the purchase order for the supplies was issued, the following entry was made:
Encumbrances control (estimated cost of supplies) 5,000
Reserve for encumbrances 5,000
Upon receipt of the supplies and invoice, the following entries were made:
Reserve for encumbrances (reverse original entry) 5,000
Encumbrances control 5,000
Expenditures (actual cost) 4,950
Vouchers payable 4,950

193
Q

Which of the following funds of a governmental unit records depreciation?

A
Capital projects fund
B
Debt service fund
C
Internal service fund
D
Special revenue fund
A

Explanation:
Internal Service fund is part of the proprietary funds which use the accrual basis of accounting with economic resources approach. Statement of revenue, expenses, and changes in net position:

Operating revenues - Operating expenses +/- Non operating revenues (expenses) = Change in net position
Depreciate capital assets
Options (a), (b) and (d) are incorrect because governmental funds include Special Revenue Fund, Capital Projects Fund, and Debt Service Fund use modified accrual accounting with financial resources approach

194
Q

Which of the following characteristics describe a multiple plan management of a PERS?
A
Assets accumulated for benefit payments are available to pay benefits for any of the plan members.
B
Assets are accumulated in separate reserves restricted for benefits of specific classes or groups of employees.
C
The government serves as a custodian for the accumulated assets and distributes funds based on instructions from member governments.
D
Assets are not accessible to creditors of member governments.

A

B

Explanation:
The PERS is responsible for multiple plans when assets are accumulated in separate reserves restricted for benefits of specific classes or groups of employees. Assets being accumulated for benefit payments available to pay benefits for any of the plan members is a characteristic of the single plan. The government serving as a custodian for the accumulated assets and distributing funds based on instructions from member governments describes an agency fund. Creditor access is not a distinguishing feature for pension trust plans.

195
Q

River City has adopted GASB Statement No. 34, Basic Financial Statements and Management’s Discussion and Analysis for State and Local Governments. River has a defined contribution pension plan. How should River report the pension plan in its financial statements?
A
Within the component units column of its government-wide financial statements
B
Within the fiduciary column of its government-wide financial statements
C
Within its fund financial statements
D
Within the governmental activities column of its government-wide financial statements

A

B

Explanation:
The government-wide statements should display information about the reporting government as a whole, except for its fiduciary activities. A pension plan is a fiduciary activity, and thus a pension trust fund is reported in the fund financial statements, but not the government-wide statements. Fiduciary statements should include financial information for fiduciary funds and similar component units.

196
Q

Fiduciary funds are accounted for using which of the following?
A
Accrual basis of accounting
B
Modified accrual basis of accounting
C
Flow of economic capital basis of accounting
D
Accrual or modified approach, whichever is most appropriate

A

Explanation:

Fiduciary funds are accounted for using the accrual basis of accounting.

197
Q

The encumbrance account of a governmental unit is debited when

A
The budget is recorded.
B
A purchase order is approved.
C
Goods are received.
D
A voucher payable is recorded.
A

B

Explanation:
The following entry is made when a purchase order for $50 is approved:

Encumbrances (estimated cost) 50
Budgetary Fund balance reserve for encumbrances 50

The following entries would be made to record the receipt of the related goods for $55 and the vouchers payable:

Budgetary Fund balance reserved for encumbrances 50
Encumbrances (estimated cost) 50
Expenditures (actual cost) 55
Vouchers payable 55

198
Q

In the General Fund Statement of Revenues, Expenditures, and Changes in Fund Balances, which of the following has an effect on the excess of revenues over expenditures?

A
Purchase of fixed assets
B
Payment to a debt-service fund
C
Special items
D
Proceeds from the sale of capital assets
A

Explanation:
The correct answer is (A).

The General Fund uses modified accrual accounting, in which the purchase of a fixed asset is an expenditure, which would affect the excess of revenue over expenditure.

Option (B) is incorrect because transfers out are reported as other financing uses which are presented below the excess of revenues (deficiency) over expenditure.

Option (C) is incorrect because special items are presented below the excess of revenues (deficiency) over expenditure.

Option (D) is incorrect because proceeds from the sale are reported in special items, which is presented below the excess of revenues (deficiency) over expenditure.

199
Q

Which of the following funds would be reported as a fiduciary fund in Pine City’s financial statements?

A
Special revenue
B
Permanent
C
Private-purpose trust
D
Internal service
A

Explanation:
The correct answer is (C).

Fiduciary Funds include:

Pension Trust Fund
Investment Trust Fund
Private Purpose Trust Fund 
Custodial Fund
Option (a) and (b) are incorrect because these are part of governmental funds. Option (d) is incorrect because the internal service fund is part of proprietary funds.
200
Q

One feature of state and local government accounting and financial reporting is that fixed assets used for general government activities
A
Often are not expected to contribute to the generation of revenues.
B
Do not depreciate as a result of such use.
C
Are acquired only when direct contribution to revenues is expected.
D
Should not be maintained at the same level as those of businesses so that current financial resources can be used for other government services.

A

A

Explanation:
Governmental entities invest large amounts of resources in non-revenue producing capital assets such as government office buildings, highways, bridges, and sidewalks.

201
Q

Dale City is accumulating financial resources that are legally restricted to payments of general long-term debt principal and interest maturing in future years. At December 31, $5,000,000 has been accumulated for principal payments and $300,000 has been accumulated for interest payments. These restricted funds should be accounted for in the

Debt service fund	General fund
A	$ 0	$5,300,000
B	$ 300,000	$5,000,000
C	$5,000,000	$ 300,000
D	$5,300,000	$ 0
A

D

Explanation:
The debt service fund accounts for and reports the resources that are restricted, committed or assigned for this long-term debt purpose. This fund is responsible for accumulating and making both interest and principal payments on the tax supported debts of the government fund. Thus, both the accumulated principal and the interest payments will be accounted for in the debt service fund for $5,300,000. Only activities not accounted for in other funds will be reported in the general fund. Options (a), (b) and (c) are incorrect based on the above explanation.

202
Q

What is the measurement focus and the basis of accounting for the government-wide financial statements?
Measurement focus Basis of accounting
A Current financial resources Modified accrual
B Economic resources Modified accrual
C Current financial resources Accrual
D Economic resources Accrual

A

D

Explanation:
Government-wide financial statements aggregate information for all governmental and business-type activities. GASB 34 requires an economic resources measurement focus and accrual basis of accounting for all amounts in the government-wide financial statements.

203
Q

In the current year, Mentor Town received $4,000,000 of bond proceeds to be used for capital projects. Of this amount, $1,000,000 was expended in this year. Expenditures for the $3,000,000 balance were expected to be incurred in the following year. These bonds proceeds should be recorded in capital projects funds for

A
$4,000,000 in the current year
B
$4,000,000 in the following year
C
$1,000,000 in the current year and $3,000,000 in the following year
D
$1,000,000 in the current year and $3,000,000 in the general fund the following year
A

A

Explanation:
Debt proceeds should be recognized by a capital projects fund at the time the debt is incurred, rather than the time the debt is authorized or when the proceeds are expended. Debt proceeds should be reported in the capital projects fund as Other Financing Sources rather than as Revenues. The entry in the capital projects fund to record the issuance of the bonds is as follows:

Cash 4,000,000
Other Financing Sources—Bond Proceeds 4,000,000

204
Q
Ridge Township's governing body adopted its general fund budget for the year ended July 31, comprised of estimated revenues of $100,000 and appropriations of $80,000. Ridge formally integrates its budget into the accounting records. To record the appropriations of $80,000, Ridge should
A
Credit appropriations control.
B
Debit appropriations control.
C
Credit estimated expenditures control.
D
Debit estimated expenditures control.
A

A

Explanation:
The journal entry to record the adoption of the budget is as follows:

Estimated Revenues Control 100,000
Appropriations Control 80,000
Budgetary Fund Balance–Unreserved 20,000

205
Q

Seaview City received a pass-through grant from the state. The money is to be distributed to families who, as determined by the state, are eligible for summer camp scholarships for their children. Seaview does not have administrative or direct financial involvement in the program. In which fund should Seaview record the grant?

A
Custodial
B
General
C
Internal service
D
Enterprise
A

Explanation:
The correct answer is (A).

Custodial funds account for collected amounts that must be transferred to other funds or outsiders. Seaview city received a pass-through grant from the state to distribute scholarship to the eligible children for the summer-camp and holding the cash temporarily and is not obligated. Thus, this is reported in the Custodial fund.

206
Q

Where does GASB Statement No. 34, Basic Financial Statements and Management’s Discussion and Analysis for State and Local Governments, require management’s discussion and analysis (MD&A) to be presented?
A
Before the financial statements
B
Before the notes to the financial statements, but after the financial statements
C
In the notes to the financial statements
D
After the notes to the financial statements, before other required supplementary information

A

A maybe idk lol

Explanation:
GASB 34 (para. 8) states, "The basic financial statements should be preceded by MD&A, which is required supplementary information. MD&A should provide an objective and easily readable analysis of the government's financial activities based on currently known facts….".
207
Q

Smith City levied property taxes totaling $1,000,000 during the current year, of which $150,000 was collected within 60 days after year end. What amount should Smith City record as deferred revenue on its statement of net position?

A
$1,000,000
B
$850,000
C
$150,000
D
$0
A

D

Explanation:
Remember that the statement of net position is a government-wide financial statement, which means that it will use the accrual method of accounting. The accrual method does not defer property tax revenues that are not received within 60 days of year end, so the answer to this question is $0.

208
Q
The estimated revenues control account of a governmental unit is debited when
A
Actual revenues are recorded.
B
Actual revenues are collected.
C
The budget is recorded.
D
The budget is closed at the end of the year.
A

C

Explanation:
The estimated revenues control account is a budgetary account. Budgetary accounts are generally opened with balances opposite of the corresponding actual account.

209
Q

Which account should Spring Township credit when it issues a purchase order for supplies?

A
Appropriations control.
B
Vouchers payable.
C
Encumbrance control.
D
Reserve for encumbrances.
A

D

Explanation:
The entry when a purchase order is issued is as follows:

Encumbrances XX
Reserve for encumbrances XX
When a corresponding invoice is received for $105, the following two entries are made:

Reserve for Encumbrances XX
Encumbrances XX
Expenditures (actual cost) YY
Vouchers Payable YY

210
Q
Park City uses encumbrance accounting and formally integrates its budget into the general fund's accounting records. Park incurred salaries and wages of $800,000 for the month of April. What account should Park debit to record this $800,000?
A
Encumbrances control
B
Salaries and wages expense control
C
Expenditures control
D
Operating funds control
A

C

Explanation:
While goods and services committed for by purchase order or contract are encumbered in governmental funds to avoid overspending appropriations, some expenditures are controlled by other means and need not be encumbered. Salaries and wages are set by contract and controlled by established payroll procedures and are not encumbered. Salaries and wages expense would not be appropriate for a governmental-type fund. Salaries and wages should be recorded as follows:

Expenditures control 800,000
Vouchers payable 800,000

211
Q

Which of the following characteristics describe the financial resources for capital projects funds?
A
Revenues based on rates that will recover the cost of capital assets required to deliver services
B
Bond issue proceeds, grants, or interest earnings
C
Donations restricted, such that only investment earnings can be used for maintenance
D
Prior year encumbrances

A

B

Explanation:
Resources for capital project funds may arise from bond issue proceeds, grants, or interest earnings. Revenues based on rates that will recover the costs of capital assets required to deliver services describes the approach used in enterprise fund accounting. Donations restricted such that only investment earnings can be used for maintenance would be accounted for in a permanent fund. The encumbrance system is used in governmental funds (general, special revenue, and capital projects funds) to prevent over-expenditure and to demonstrate compliance with legal requirements. Encumbrances are a budgetary measurement element, not a criterion for determining fund type.

212
Q

Which of the following funds will Zebra Town rarely report within the same activity column as the general fund in the government-wide financial statements?

A
Debt service fund
B
Enterprise fund
C
Internal service fund
D
Permanent fund
A

B

Explanation:
The governmental-type funds and internal service funds usually appear in the government activities column in the government-wide financial statements. Enterprise funds generally appear in the business-type activities column in the government-wide financial statements.

Internal service fund assets and liability balances that are not eliminated in the statement of net position should normally be reported in the governmental activities column. Although internal service funds are reported as proprietary funds, the activities accounted for in them are usually more governmental than business-type in nature. If enterprise funds are the predominant participants in an internal service fund, however, the government should report that internal service fund’s residual assets and liabilities within the business-type activities column in the statement of net position.

213
Q

Expenditures of a governmental unit for insurance extending over more than one accounting period

A
Must be accounted for as expenditures of the period of acquisition.
B
Must be accounted for as expenditures of the periods subsequent to acquisition.
C
Must be allocated between or among accounting periods.
D
May be allocated between or among accounting periods or may be accounted for as expenditures of the period of acquisition.

A

D

Explanation:
Expenditures are recorded when fund liabilities are incurred or assets are expended, except in regard to inventory items, interest on general long-term debt, and prepaids such as insurance. This is due to the emphasis on the flow of financial resources in governmental accounting.

214
Q

River City has a defined contribution pension plan. How should River report the pension plan in its financial statements?
A
Amortize any transition asset over the estimated number of years of current employees’ service.
B
Disclose in the notes to the financial statements the amount of the pension benefit obligation and the net assets available for benefits.
C
Identify in the notes to financial statements the types of employees covered and the employer’s and employees’ obligations to contribute to the fund.
D
Accrue a liability for benefits earned but not paid to fund participants.

A

C

Explanation:
Governmental employers identify in the notes to the financial statements the types of employees covered under the defined contribution pension plan and the employer’s and employees’ obligations to contribute to the fund.

215
Q

For governmental external financial reporting, the note on significant accounting policies should include descriptive comments about the purposes and scope of which of the following?

A
Other supplementary information
B
Required supplementary information
C
The statements of net position and activities
D
The statement of net position and net liabilities
A

Explanation:
The correct answer is (C).

The note on significant accounting policies should include descriptive comments about the purposes and scope of the statements of net position and activities. The note on significant accounting policies will only focus on the government-wide statements and does not include comments about other or required supplementary information. There is no statement of net liabilities.

216
Q
Lily City uses a pay-as-you-go approach for funding postemployment benefits other than pensions. The city reports no other postemployment benefits (OPEB) liability at the beginning of the year. At the end of the year, Lily City reported the following information related to OPEB for the water enterprise fund:
Benefits paid	$100,000
Annual required contribution	500,000
Unfunded actuarial accrued liability	800,000What amount of expense for OPEB should Lily City’s water enterprise fund report in its fund level statements?
A
$100,000
B
$500,000
C
$600,000
D
$1,400,000
A

B

Explanation:
Other postemployment benefits (OPEB) are typically in the form of health care coverage. Most entities fund their OPEB on a pay-as-you-go basis, paying only the amount equal to benefits claimed. GASB believes the pay-as-you-go approach does not reflect the true OPEB liability, and issued GASB 45 outlining OPEB reporting and disclosure requirement. The objective is to systematically and rationally allocate the present value of the projected cost of benefits over the working life of benefiting employees. This allocation is done with an actuarial valuation. The amount allocated to a given year is known as the annual required contribution (ARC), which is the basic amount of expense (i.e., $500,000). The present value of benefits earned to date by employees is known as the actuarial accrued liability. The difference between this amount and any resources placed in trust is known as the unfunded actuarial accrued liability.

217
Q

The following information pertains to certain money held by Blair County at December 31 that are legally restricted to expenditures for specified purposes:

Proceeds of long-term debt to be used for a
major capital project $90,000
Proceeds of short-term notes to be used for
advances to permanent trust funds 8,000
What amount of this restricted money should Blair account for in special revenue funds?

A
$0
B
$8,000
C
$90,000
D
$98,000
A

A

Explanation:
Special revenue funds are used to account for financial resources that are restricted by law or by contractual agreement to specific purposes other than for permanent funds or major capital projects. Thus, neither the $8,000 of proceeds of the short-term notes to be used for advances to permanent fund nor the $90,000 of proceeds on long-term debt to be used for a major capital project should be accounted for in a spe­cial revenue fund.

218
Q

Hunt Community Development Agency (HCDA), a financially independent authority, provides loans to commercial businesses operating in Hunt County. This year, HCDA made loans totaling $500,000. How should HCDA classify the disbursements of loans on the cash flow statement?

A
Operating activities.
B
Non-capital financing activities.
C
Capital and related financing activities.
D
Investing activities.
A

Explanation:
The correct answer is (A).

Hunt Community Development Agency provides loans as its operating activity, not as financing or investing activities.

As defined under Para 19 of GASB 9, these would be program loans that are made and collected as a part of a government program.

The operational purpose of this entity is to disburse loans. The loan disbursement of $500,000 would be reported under Operating Activities in the Statement of Cash Flows.

219
Q
Governmental budgetary comparisons may be presented as required supplementary information (RSI) or which of the following?
A
As a reconciling item in the statement of net assets
B
As supplementary information
C
In the notes to the financial statements
D
In the basic financial statements
A

Explanation:
Governments are encouraged to present budgetary comparisons as part of required supplementary information (RSI). There was significant discussion about whether these reports should be part of the basic financial statements, in part to ensure that audit coverage provided additional credibility for the reports. The GASB allowed governments to elect to present budgetary comparisons as statements within the basic financial statements, but not as part of other supplementary information or notes to the financial statements.

220
Q

How should a city’s general fund report the acquisition of a new police car in its governmental fund statement of revenues, expenditures and changes in fund balances?

A
Noncurrent asset
B
Expenditure
C
Expense
D
Property, plant and equipment
A

Explanation:
Both capital purchases and operating expenditures are considered spending of funds and are treated as current year expenditure.

        Dr: Expenditure              XXXX

Cr: Voucher Payable XXXX

(Government fund purchases police cars).

Option (a) is incorrect because purchase of police cars is a current year expenditure not a non-current asset. Option (c) is incorrect because purchase of police cars is a current year expenditure not expense. Option (d) is incorrect because purchase of police cars is a current year expenditure not PP&E which are non-current assets.

221
Q
The budgetary fund balance reserved for encumbrances account of a governmental-type fund is increased when
A
The budget is recorded.
B
Appropriations are recorded.
C
Supplies previously ordered are received.
D
A purchase order is approved.
A

Explanation:
The budgetary fund balance reserved for encumbrances is increased when a purchase order is approved. The budgetary fund balance reserved for encumbrances is not affected when the budget (along with the related appropriations) is recorded. Upon receipt of the item previously ordered, The budgetary fund balance reserved for encumbrances is decreased. The following entry is made when a purchase order of $50 is approved:

Encumbrances XX
Budgetary fund balance reserved for encumbrances XX

222
Q
The \_\_\_\_\_\_\_\_\_\_ basis of accounting provides the most direct link to the budgeting process.
A
Accrual
B
Modified accrual
C
Cash
D
Modified cash
A

Explanation:
The modified accrual (not accrual) basis of accounting provides the most direct link to the budgeting process and highlights that current focus that still drives the planning process in most governments.

223
Q

What basis of accounting should be used when preparing a governmental funds statement of revenues, expenditures, and changes in fund balance?

A
Accrual basis of accounting
B
Modified accrual basis of accounting
C
Modified cash basis of accounting
D
Cash basis of accounting
A

Explanation:
The correct answer is (B).

The modified accrual basis of accounting should be used when preparing a governmental funds statement of revenues, expenditures, and changes in fund balances. Under the modified accrual basis of accounting, revenue is recognized in the period it is measurable and available to spend. Costs are recorded in the period that the obligation to pay them arises, whether this is before or after the period in which the government is actually using the assets or services.

224
Q

A city government levies a tax on its citizens for improvements to roads. How should the city report the tax in its statement of activities?

A
By type of tax in general revenues.
B
By type of tax in program revenues.
C
As program-specific contributions in program revenues.
D
In special items reported separately from general revenue.
A

Explanation:
The correct answer is (A)

Revenues of the Government are reported in the Statement of Activities under the following two major headings:

Program Revenues: Includes charges levied for services, program-specific operating grants & contributions, and program-specific capital grants & contributions
General Revenues: All revenues other than ‘program revenues’
It is clear from the definition of ‘program revenues’ that taxes collected for improvement to roads do not qualify as program revenues. They are to be classified as general revenues.

(B) is incorrect because taxes are not reported in program revenues.

(C) is incorrect because taxes collected from citizens cannot be called contributions.

(D) is incorrect because ‘special items’ is meant for items which are unusual or infrequent. Tax collection isn’t an unusual or infrequent event.

225
Q

During its fiscal year ended June 30, Cliff City issued purchase orders totaling $5,000,000, which were properly charged to encumbrances at that time. Cliff received goods and related invoices at the encumbered amounts totaling $4,500,000 before year end. The remaining goods of $500,000 were not received until after year end. Cliff paid $4,200,000 of the invoices received during the year. What amount of Cliff’s encumbrances were outstanding at June 30?

A
$0
B
$300,000
C
$500,000
D
$800,000
A

Explanation:
The amount of Cliff’s encumbrances that were outstanding at June 30 is $500,000 (i.e., $5,000,000– $4,500,000). When the purchase orders were issued, the following entry was made:

Encumbrances Control (estimated cost)	
5,000,000

Reserve for Encumbrances 5,000,000
Upon receipt of the goods and related invoices, the following entries were made:

Reserve for Encumbrances

4,500,000

Encumbrances Control

4,500,000

Expenditures (actual cost)

4,200,000

Vouchers Payable

4,200,000

226
Q

Which of the following is a required part of a local government’s management’s discussion and analysis (MD&A) as part of its financial statements?
A
The MD&A should be presented with other required supplementary information.
B
The MD&A should compare current-year results to the prior year with emphasis on the current year.
C
The MD&A should include an analysis for each fund.
D
The MD&A should present condensed financial information from the fund financial statements.

A

Explanation:
The management’s discussion and analysis (MD&A) provides an analysis of the government’s financial performance for the year, provides comparisons of the current-year results to the previous year, and discusses both the positive and negative aspects of the comparison. Overall, the emphasis should be on the current year and the focus should be on the primary government. Although considered RSI, the MD&A is presented before the basic financial statements. The MD&A should present condensed financial information derived from government-wide financial statements, not fund financial statements, comparing the current year to the prior year. Although the MD&A should include an analysis of balances and transactions of individual funds, the AICPA provided answer choice b, not c, as the correct answer response.

227
Q

During the current year, Vann County’s motor pool internal service fund sold two vehicles for $5,000. The vehicles had a cost of $6,000 and a carrying value of $4,000. How should Vann County’s motor pool internal service fund report this transaction in its fund financial statements?

A
Revenue of $5,000
B
Other financing source of $5,000
C
Special item of $1,000
D
Gain of $1,000
A

Explanation:
The correct answer is (D).

The motor pool internal service fund is reported under the proprietary fund in the fund financial statements. The proprietary fund uses the accrual basis of accounting which records operating revenues, operating expenses and non-operating revenues and non-operating expenses to get the change in net position. Vann County’s motor pool internal service sold two vehicles for $5,000. Both vehicles together have a cost of $6,000 and a carrying value of $4,000. Because the carrying value was $4,000 and the vehicles sold for $5,000, Vann County’s motor pool internal service fund should report a gain of $1,000.

228
Q

Jonn City entered into a 3-year lease for equipment during the year. The ownership of the asset is transferred at the end of the lease term. How should the asset obtained through the lease be reported in Jonn City’s governmentwide statement of net position?

A
General capital asset.
B
Other financing use.
C
Expenditure.
D
Not reported.
A

Explanation:
The correct answer is (A).

General capital assets are not specifically related to activities reported in proprietary or fiduciary funds.

They are associated with and generally arise from, governmental activities.

They should not be reported as assets in governmental funds but should be reported in the governmental activity’s column in the government-wide statement of net position.

229
Q

Which of the following characteristics of service efforts and accomplishments is the most difficult to report for a governmental entity?

A
Comparability
B
Timeliness
C
Consistency
D
Relevance
A

D

Explanation:
Comparability, consistency, and timeliness in SEA reporting are readily accomplished for a single governmental entity. Ensuring that reported performance yardsticks measure goals and desired effects is more complex.

230
Q

If a city government is the primary reporting entity, which of the following is an acceptable method to present component units in its combined financial
statements?

A
Consolidation.
B
Cost method.
C
Discrete presentation.
D
Government-wide presentation.
A

C

Explanation:
Government-wide statements (GWS) aggregate information for all governmental and business-type activities. GASB 34 requires an economic resources
measurement focus and accrual basis of accounting for all amounts in the GWS. There are four required columns in the GWS, one each for: governmental
activities, business-type activities, the primary government (sum of the previous two), and component units. Most component units should be included in the
financial reporting entity by discrete presentation (reported in columns separate from primary government).

231
Q

Harland County received a $2,000,000 capital grant to be equally distributed among its five municipalities. The grant is to finance the construction of capital assets. Harland had no administrative or direct financial involvement in the construction. In which fund should Harland record the receipt of cash?

A
Custodial fund
B
General fund
C
Special revenue fund
D
Private purpose trust fund
A

Explanation:
The correct answer is (A).

Custodial Funds account for collected amounts that must be transferred to other funds or outsiders. Harland County is collecting and holding the cash temporarily and not obligated, so the cash is reported in the Custodial Fund.

(B) is incorrect because the General Fund accounts for and reports any activity or function by the government unit that is not being accounted for in another fund.
(C) is incorrect because the Special Revenue Fund accounts for and reports specific revenues from earmarked sources that are restricted or committed to be used to finance designated activities other than Capital Projects and Debt Service.
(D) is incorrect because the Private Purpose Trust Fund accounts for resources that are being held for benefit of private persons, organizations or other governments

232
Q

Smith City levied property taxes totaling $1,000,000 during the current year, of which $150,000 was collected within 60 days after year-end. What amount should Smith City record as property tax revenue?

A
$850,000
B
$1,000,000
C
$150,000
D
$0
A

Explanation:
The correct answer is (B).

Property taxes are considered the enforceable legal claim and is always recognized when billed. A government can expect to collect the amount billed to taxpayers once a property tax assessment is made as these taxes are mandatory.

As a result, property taxes that have been billed usually satisfy the requirements of being measurable and available, and are accrued in advance of collection. However, an allowance for uncollectible must be established.

Dr. Property Taxes Receivable-Current $1,000,000
Cr. Property Tax Revenue $1,000,000

233
Q

Which of the following mandates the use of an enterprise fund?
A
Activity is financed with debt that is secured by a pledge of the net revenues from fees and charges of the activity.
B
User fees are charged to external users for goods and services.
C
The General Fund provides a subsidy to cover operating deficits.
D
All surplus funds are transferred to the General Fund as a return on capital.

A

A

Explanation:
GASB Statement No. 34 listed three criteria, any one of which requires the use of an enterprise fund. Activity being financed with debt that is secured by a pledge of the net revenues from fees and charges of the activity is the first of the listed criteria. User fees charged to external users for goods and services is an optional reason for using an enterprise fund. The General Fund might provide a subsidy, but this financing does not require the use of an enterprise fund. Surplus funds may be returned to the General Fund, but this transfer does not require the use of an enterprise fund.

234
Q
For governmental fund types, which item is considered the primary measurement focus?
A
Income determination
B
Flows and balances of financial resources
C
Capital maintenance
D
Cash flows and balances
A

Explanation:
Governmental fund measurement focus is on determination of financial position and changes in financial position (sources, uses, and balances of financial resources), rather than on net income determination.

235
Q
A government makes a contribution to its pension plan in the amount of $10,000 for year 1. The actuarially-determined annual required contribution for year 1 was $13,500. The pension plan paid benefits of $8,200 and refunded employee contributions of $800 for year 1. What is the pension expenditure for the general fund for year 1?
A
$ 8,200
B
$ 9,000
C
$10,000
D
$13,500
A

C

Explanation:
The annual pension cost differs from the pension expenditure/expense reported in the financial statements. The annual pension cost is the period cost of an employer’s participation in a defined benefit pension plan. The pension expenditure/expense is the amount recognized by an employer in each accounting period for contributions to a pension plan.Therefore, the $10,000 contribution represents the pension expenditure for year 1.

236
Q

Program-specific operating grants and contributions are which of the following under governmental accounting?
A
Nonexchange revenues that are restricted to the operations of specific programs or functions
B
Exchange revenues that are restricted to the operations of specific programs or functions
C
Nonexchange revenues that are not restricted to the operations of specific programs or functions
D
Exchange revenues that are not restricted to the operations of specific programs or functions

A

Explanation:
Program-specific operating grants and contributions are nonexchange revenues that are restricted to the operations of specific programs or functions.

237
Q

Governments demonstrate accountability for managing public resources by doing which of the following?
A
Using a cost-based basis of accounting for the statement of activities
B
Adjusting property tax rates to reflect the level of services used by taxpayers
C
Complying with a legally adopted budget that controls appropriations
D
Charging for services that are structured to recover all the costs of operations

A

Explanation:
Budgets play a key role in managing public funds. Governments adopt a budget with legislative approval providing a legal mandate for the expenditure of public funds.

238
Q

Financing for the renovation of Fir City’s municipal park, begun and completed during the year, came from the following sources:

Grant from state government
$400,000
Proceeds from general obligation bond issue
$500,000
Transfer from Fir’s general fund
$100,000
In its capital projects fund operating statement, Fir should report these amounts as

Revenues	Other Financing Sources
A	$1,000,000	$0
B	$ 900,000	$ 100,000
C	$ 400,000	$ 600,000
D	$0	$1,000,000
A

Explanation:
The capital projects fund reports unrestricted grants received from other governmental units as revenue. Therefore, the $400,000 grant from the state is reported as revenue in Fir’s capital projects fund’s operating statement. The capital projects fund reports long-term debt proceeds and operating transfers from other funds as other financing sources. Therefore, the $500,000 proceeds from the general obligation bond issue and the $100,000 transfer from Fir’s general fund are reported as other financing sources of $600,000 in the capital projects fund’s operating statement.

239
Q
The portion of special assessment debt maturing in 5 years, to be repaid from general resources of the government, should be reported in the
A
General fund column.
B
Governmental activities column.
C
Agency column.
D
Capital projects column.
A

B

Explanation:
General government fixed assets and long-term debt are not recorded in the governmental funds, but are shown in the governmental activities column of the governmental-wide financial statements. General long-term debt to be repaid from general resources of the government should not be reported in any fund.

240
Q

Taxes collected and held by Eldorado County for a school district would be accounted for in which of the following funds?

A
Trust
B
Custodial
C
Special revenue
D
Internal service
A

Explanation:
The correct answer is (B)

Custodial Funds are used to account for resources held by a government as an agent for individuals or other governmental units. Taxes, for example, are occasionally collected by one governmental unit on behalf of another. Until that, money is physically transferred to the proper authority, it is recorded in a Custodial Fund.

A Trust Fund accounts for assets received and held by a governmental unit acting in the capacity of trustee or custodian. The aim in Trust Fund accounting is to ensure that the money or other resources are handled in accordance with the terms of the trust agreement and/or applicable trust laws.

Special Revenue Funds are used to account for revenues that have been legally restricted as to expenditure (e.g., a state gasoline tax collected in order to maintain streets).

Internal Service Funds are used to account for any department or custodian that provides services within the government on a cost-reimbursement basis.

241
Q

Sig City used the following funds for financial reporting purposes:
General fund Capital projects fund
Internal service fund Special revenue fund
Airport enterprise fund Debt service fund
Pension trust fund How many of Sig’s funds use the accrual basis of accounting?
A
Two
B
Three
C
Four
D
Five

A

B

Explanation:
The three funds using the accrual basis of accounting are the (1) internal service fund, (2) airport enterprise fund, and (3) pension trust fund (fiduciary fund). The modified accrual basis of accounting is used by the general fund, capital projects fund, special revenue fund, and debt service fund.

242
Q

Which of the following is a difference between governmental businesses and commercial enterprises?

A
User fees structured to recover operating costs, including the costs of capital
B
Allocation of the costs of capital assets using depreciation based on estimated asset service lives
C
Mission strategy to improve quality of life for community
D
Issuance of revenue debt to finance capital expansions and improvements

A

Explanation:
The correct answer is (C).

Governmental businesses, unlike commercial enterprises, have a mission strategy to improve quality of life for community whereas private sector commercial entities are focused on generating a net profit for shareholders.

Both governmental and private sector enterprises aim to charge fees sufficient to recover costs and depreciation based on estimated service lives. Either type of entity would likely issue revenue debt to finance capital expansions.

243
Q

Dayne County’s general fund had the following disbursements during the year.

Payment of principal on long-term debt $100,000
Payments to vendors 500,000
Purchase of a computer 300,000
What amount should Dayne County report as expenditures in its governmental funds statement of revenues, expenditures, and changes in fund balances?

A
$300,000
B
$500,000
C
$800,000
D
$900,000
A

Explanation:
Expenditures differ from expenses (as defined in commercial accounting) because expenditures include—in addition to current operating expenditures that benefit the current period—capital outlays for general fixed assets and repayment of general long-term debt principal.

Payments of principal on long-term debt $100,000
Payments to vendors 500,000
Purchase of a computer 300,000
Total expenditures $900,000
Option (a) is incorrect because payment to vendors and payment of principal on long-term debt is excluded. Option (b) is incorrect because payment of principal on long-term debt and purchase of computer is excluded. Option (c) is incorrect because payment of principal on long-term debt is excluded.

244
Q

All of the following statements regarding notes to the basic financial statements of governmental entities are true except
A
The notes contain disclosures related to required supplementary information.
B
Some notes presented by governments are identical to notes presented in business financial statements.
C
Notes that are considered essential to the basic financial statements need to be presented.
D
It is acceptable to present notes in a very extensive format.

A

A

Explanation:
The notes to the basic financial statements of governmental entities do not contain disclosures related to required supplementary information. The Management’s Discussion and Analysis (MD&A) is the required supplementary information in the government’s general purpose external report and it is presented before the financial statements. .

245
Q

GASB Statement No. 38 expanded the required disclosures for interfund balances and transfers because of which of the following?
A
Users were concerned about how these transactions affected resources available in any given fund.
B
Users were concerned that illegal transfers among funds were being made and wanted to shed light on the process.
C
Governments considered the details to be essential to demonstrating budgetary compliance.
D
Governments were burying details of such transactions in explanations provided in note disclosures.

A

Maybe A idk

Explanation:
GASB Statement No. 38 expanded the disclosures required related to interfund balances and transfers. The disclosures weren’t required because governments considered the details to be essential to demonstrating budgetary compliance. Although more details could muddy the waters for users, in this case, users wanted more detail because they expressed concern about how

246
Q
A \_\_\_\_\_\_\_\_\_\_\_\_ fund is most likely to be the fund that taxpayers recognize as the central funding source for basic services.
A
General
B
Special revenue
C
Capital projects
D
Debt service
A

A

Explanation:
The general fund is the one most likely to be the fund that taxpayers recognize as the central funding source for basic services, such as police, fire, or recreation. Users seemed to have little interest in accounting designations of special revenue versus debt service and more interest in the results for a specific program.

247
Q

When a lease of a governmental unit represents the acquisition of a general fixed asset, the acquisition should be reflected as

A
An expenditure but not as an other financing source.
B
An other financing source but not as an expenditure.
C
Both an expenditure and an other financing source.
D
Neither an expenditure nor an other financing source.

A

Explanation:
The correct answer is (C).

When a lease represents the acquisition of a general capital asset, it is accounted for as a Contract that transfers ownership.

The acquisition or construction of that asset should be reflected as an expenditure and Other Financing Source, consistent with the accounting and financial reporting for general obligation bonded debt.

248
Q
Fixed assets donated to a governmental unit should be recorded
A
At the donor's carrying amount.
B
At estimated fair value when received.
C
At the lower of the donor's carrying amount or estimated fair value when received.
D
As a memorandum entry only.
A

Explanation:

Fixed assets donated to a governmental unit are recorded at fair value when received.

249
Q
The link between government-wide and governmental fund statements requires a reconciliation to convert the governmental funds to the \_\_\_\_\_\_\_\_\_\_\_\_\_\_ resources measurement focus and \_\_\_\_\_\_\_\_\_\_\_\_\_\_ basis of accounting.
A
Economic; accrual
B
Economic; modified accrual
C
Current financial; accrual
D
Current financial; modified accrual
A

A

Explanation:
The link between government-wide and governmental fund statements requires a reconciliation to convert the governmental funds to the economic resources measurement focus and accrual basis of accounting.

250
Q

Which of the following does not affect an internal service fund’s net income?

A
Depreciation expense on its fixed assets
B
Operating transfers sources
C
Residual equity transfers
D
Temporary transfers
A

D

Explanation:
An internal service fund’s net income is not affected by temporary transfers. Internal service funds report residual equity transactions as Transfers In or Out. They are reported after non operating revenues and expenses as a capital transaction. An internal service fund’s net income is affected by operating revenues and expenses, non operating revenues and expenses, and operating and residual equity transfers.

251
Q

The following proceeds received by Arbor City are legally restricted to expenditure for specified purposes:
Donation by a benefactor mandated to an expendable
trust fund to provide meals for the needy $100,000
Sales taxes to finance the maintenance of tourist
facilities in the shopping district 300,000What amount should be accounted for in Arbor’s special revenue funds?
A
$0
B
$100,000
C
$300,000
D
$400,000

A

D

Explanation:
Special revenue funds are used to account for revenues that have been legally restricted as to expenditure. The NCGA provides several examples of revenues which would fall under this heading: (1) a state gasoline tax collected in order to maintain streets, (2) proceeds from parking meters which finance the local traffic court, and (3) state juvenile rehabilitation grants used to operate and maintain juvenile rehabilitation centers. In each of these cases, a service is being provided, but the funding comes from a specific source rather than from property taxes or any other general revenues. The donation mandated to provide meals for the needy should be accounted for in an private purpose trust fund. Special revenue funds are used to account for resources held in trust by a government for a specific purpose where both the principal and any earnings may be expended for government programs. Private-purpose trust funds resources are to be expended for the benefit of individuals, organizations or other governments.

252
Q

Which of the following are the constraints that create restricted net position in governmental accounting?

A
External impositions by creditors, grantors, contributors, or laws or regulations of other governments
B
Imposition by law through constitutional provisions or enabling legislation
C
Both A. and B.
D
Neither A. nor B.
A

C

Explanation:
There are two constraints that create “restricted net assets” in the government-wide statement of net position: constraints externally imposed by creditors, grantors, contributors, or laws or regulations of other governments and constraints imposed by law through constitutional provisions or enabling legislation.

253
Q
On March 2, year 1, Finch City issued 10-year general obligation bonds at face amount, with interest payable March 1 and September 1. The proceeds were to be used to finance the construction of a civic center over the period April 1, year 1, to March 31, year 2. During the fiscal year ended June 30, year 2, no resources had been provided to the debt service fund for the payment of principal and interest. On June 30, year 2, Finch's debt service fund should include interest payable on the general obligation bonds for
A
0 months.
B
3 months.
C
4 months.
D
6 months.
A

A

Explanation:
Finch did not provide debt service fund resources during the current fiscal year for the payment of the interest on the general obligation bonds. Therefore, the interest expenditure related to the general obligation bonds should be recorded when it is due to be paid (i.e., September 1, year 2) rather than being accrued at year-end (i.e., June 30, year 2). Only if debt service fund resources had been provided during the current fiscal year for the payment of the interest, could the expenditure and related liability be recognized in the debt service fund.

254
Q
When a purchase order is released, a commitment is made by a governmental unit to buy a computer to be manufactured to specifications for use in property tax administration. This commitment should be recorded in the general fund as a (an)
A
Appropriation.
B
Encumbrance.
C
Expenditure.
D
Fixed asset.
A

B

Explanation:
Commitments made by a government are encumbrances. Appropriations are amounts budgeted to be spent. Expenditures are amounts that have been spent. A fixed assets account is debited when the property is placed in service.

255
Q
Which of the following is included in the governmental fund statements?
A
Cash flows statements
B
Component unit information
C
Reconciliation between the fund statements and the government-wide statements
D
Major fiduciary funds
A

C

Explanation:
Governmental fund financial statements present a reconciliation to the government-wide statements. Cash flows statements appear only in the fund statements for proprietary funds. Individual component unit information is presented in individual columns in the government-wide statements, after the primary government’s fund statements, or in the notes to the financial statements. Governmental fund financial statements do not include major fiduciary funds. Even fiduciary fund statements present fund types, not major funds.

256
Q
Deficits in any component of net position could indicate which of the following?
A
Liabilities exceed the value of assets
B
Assets are fully depreciated
C
Net position has deteriorated
D
Internal balances exceed available resources
A

A

Explanation:
Net position is the residual of assets and deferred outflows less liabilities and deferred inflows. A deficit in any of the three components would indicate that liabilities probably exceed assets and deferred outflows. Assets may be fully depreciated if associated debt exceeds the net value of capital assets; however, this component could have a deficit if the debt amortization period is longer than the asset depreciation schedule even before assets are fully depreciated. Net position is certainly in jeopardy if any of the components have negative balances; however, the change in net position could have improved if the deficit is lower than the previous year. Internal balances are eliminated and reclassified for the government-wide statements and should not cause deficits in components of net position.

257
Q

Nack City received a donation of a valuable painting. Nack planned to add the painting to its collection and display it in the protected exhibition area of city hall. Nack had a policy that if such donated art works were sold, the proceeds would be used to acquire other items for its collections. Which of the following would be correct regarding the donated painting?
A
Must be capitalized and depreciated.
B
Must be capitalized but not depreciated.
C
May be capitalized, but it is not required, and it must be depreciated.
D
May be capitalized, but it is not required, and depreciation is not required.

A

D

Explanation:
Governments are encouraged, but not required, to capitalize a collection of works of art or historical treasures (and all additions to that collection) whether donated or purchased that meets all of the following conditions. The collection is 1) held for public exhibition, education, or research in furtherance of public service rather than financial gain, 2) protected, kept encumbered, cared for, and preserved, and 3) subject to an organizational policy that requires the proceeds from sales of collection items to be used to acquire other items for collections. Capitalized collections or individual items that are exhaustible, such as exhibits whose useful lives are diminished by display or educational or research applications, should be depreciated over their estimated useful lives. Depreciation is not required for collections or individual items that are inexhaustible.

258
Q
Types of operating information required in the statistical section of the CAFR include which of the following?
A
The number of government employees
B
Operating indicators
C
Capital asset information
D
All of the above
A

D

Explanation:
Three types of operating information are required in the statistical section of the CAFR: number of government employees, operating indicators, and capital asset information.

259
Q

Glen County uses governmental fund accounting and is the administrator of a multiple-jurisdiction deferred compensation plan covering both its own employees and those of other governments participating in the plan. This plan is an eligible deferred compensation plan under the U.S. Internal Revenue Code and Income Tax Regulations. Glen has legal access to the plan’s $40,000,000 in assets, comprising $2,000,000 pertaining to Glen and $38,000,000 pertaining to the other participating governments. In Glen’s balance sheet, what amount should be reported in the custodial fund for plan assets and as a corresponding liability?

A
$0
B
$2,000,000
C
$38,000,000
D
$40,000,000
A

D

Explanation:
When a governmental unit is the administrator of a multiple-jurisdiction deferred compensation plan, it should report all of the plan assets and liabilities (e.g., $40,000,000) in Custodial Fund because the governmental unit is acting as an agent for both its own employees and other governments participating in the plan.

260
Q

Which of the following is considered part of the required supplementary information for general-purpose external financial reporting of a local government?

A
Fund financial statement.
B
Combining non-major fund statement.
C
Notes to the financial statements.
D
Management discussion and analysis.
A

Explanation:
The correct answer is (D).

Required Supplementary Information is the information that is mandated to be disclosed by the reporting entity. This information, though not a part of the financial statements, are considered a prerequisite for a complete understanding and overview of financial information.

One such information is Management Discussion and Analysis (MD&A), which is the assessment of the organization’s performance for the said period by the management. MD&A are a compulsory part of Required Supplementary Information for general purpose external financial reporting of a local government.

261
Q
A capital projects fund for a new city courthouse recorded a receivable of $300,000 for a state grant and a $450,000 transfer from the general fund. What amount should be reported as revenue by the capital projects fund?
A
$0
B
$300,000
C
$450,000
D
$750,000
A

B

Explanation:
The capital projects fund reports unrestricted grants received from other governmental units as revenue. Therefore, the $300,000 grant from the state is reported as revenue in the capital projects fund’s operating statement. The capital projects fund reports long-term debt proceeds and operating transfers from other funds as other financing sources. Therefore, the $450,000 transfer from the general fund is reported as other financing sources in the capital project fund’s operating statement.

262
Q

Clay City levied property taxes of $600,000 for the current year, and estimated that $25,000 would be uncollectible. Which of the following is the correct general fund journal entry to record the property tax levy?

A
Property taxes receivable–current 600,000 Property tax revenue 600,000
B
Property taxes receivable–current 575,000 Bad debt expense 25,000 Property tax revenue 600,000
C
Property taxes receivable–current 600,000 Property tax revenue 575,000 Allowance for uncollectible property taxes–current 25,000
D
Property taxes receivable–current 600,000 Bad debt expense 25,000 Property tax revenue 600,000 Allowance for uncollectible property taxes–current 25,000

A

C

Explanation:
Property taxes are considered enforceable legal claim and is always recognized when billed. Government can expect to collect the amount billed to taxpayers once a property tax assessment is made as these taxes are mandatory. As a result, property taxes that have been billed usually satisfy the requirements of being measurable and available, and are accrued in advance of collection. However, an allowance for uncollectible must be established. Option (a) is incorrect because, J/E does not provide for allowance for uncollectible for property taxes. Option (b) is incorrect because the J/E debits bad debt expense instead of crediting allowance for uncollectible account for property taxes. Option (d) is incorrect because J/E debits bad debt expense and credits revenue at gross amount instead of recording revenue net of uncollectible on property taxes.

263
Q

The budget of a governmental unit, for which the appropriations exceed the estimated revenues, was adopted and recorded in the general ledger at the beginning of the year. During the year, expenditures and encumbrances were less than appropriations; whereas revenues equaled estimated revenues. The budgetary fund balance account is
A
Credited at the beginning of the year and debited at the end of the year.
B
Credited at the beginning of the year and not changed at the end of the year.
C
Debited at the beginning of the year and credited at the end of the year.
D
Debited at the beginning of the year and not changed at the end of the year.

A

C

Explanation:
An excess of appropriations over estimated revenues calls causes a debit to the fund balance account at the beginning of the period. An excess of appropriations over expenditures and encumbrances causes a credit to the fund balance at the end of the period.

The unassigned fund balance account of the governmental unit in question is debited at the beginning of the year because appropriations exceed estimated revenues, shown in Entry (1). During the year, revenues equaled estimated revenues; therefore, the unassigned fund balance account is not affected by the revenue closing entry, as in Enrty (2). During the year, expenditures and encumbrances were less than appropriations; therefore, the unassigned fund balance account is credited for the closing entry for the appropriations, expenditures, and encumbrances, as in Entry (3).

Entry (1)
Estimated Revenues Control	XX	
Unassigned Fund Balance (to balance)	XX	
Appropriations Control		XX
Entry (2)		
Revenues Control	XX	
Estimated Revenues Control		XX
Entry (3)		
Appropriations Control	XX	
Expenditures Control		XX
Encumbrances Control		XX
Unassigned Fund Balance (to balance)		XX
264
Q

The following information pertains to Pine City’s general fund for the current year:

Appropriations	$6,500,000
Expenditures	5,000,000
Other financing sources	1,500,000
Other financing uses	2,000,000
Revenues	8,000,000
After Pine's general fund accounts were closed at the end of the year, the unassigned fund balance increased by
A
$3,000,000
B
$2,500,000
C
$1,500,000
D
$1,000,000
A

Explanation:
Appropriations is a budgetary account which will also be closed. However, since closing the budgetary accounts (which are not all given) simply reverses the
entry to record the budget, their closing has no effect on fund balance. The closing entry for the activity accounts given would increase the fund balance, as
follows:

Revenues	8,000,000	 
Other financing sources	1,500,000	 
Expenditures	 	5,000,000
Other financing uses	 	2,000,000
Unassigned fund balance (difference)	 	2,500,000
265
Q
Through an internal service fund, New County operates a centralized data processing center to provide services to New's other governmental units. This internal service fund billed New's parks and recreation fund $150,000 for data processing services. What account should New's internal service fund credit to record this $150,000 billing to the parks and recreation fund?
A
Data processing department expenses
B
Intergovernmental transfers
C
Interfund exchanges
D
Operating revenues control
A

D

Explanation:
Billings for services provided to other governmental units are recorded by the internal service fund as operating revenues.

266
Q

According to GASB 34, Basic Financial Statements and Management’s Discussion and Analysis for State and Local Governments, certain budgetary schedules are required supplementary information. What is the minimum budgetary information required to be reported in those schedules?
A
A schedule of unfavorable variances at the functional level.
B
A schedule showing the final appropriations budget and actual expenditures on a budgetary basis.
C
A schedule showing the original budget, the final appropriations budget, and actual inflows, outflows, and balances on a budgetary basis.
D
A schedule showing the proposed budget, the approved budget, the final amended budget, actual inflows and outflows on a budgetary basis, and variances between budget and actual.

A

C

Explanation:
Budgetary Comparison Schedule (BCS) is presented for the general fund and for each major special revenue fund that has a legally adopted annual budget, on the budgetary basis of accounting. The BCS presents both the original and final budget as well as actual inflows, outflows, and balances. It uses the same format, terminology, and classifications as either the budget document or a statement of revenues, expenditures, and changes in fund balances.

267
Q

Hill City’s water utility fund held the following investments in U.S. Treasury securities at June 30, year 5:
Investment Date purchased Maturity date Carrying amount
3-month T-bill 5/31, yr 5 7/31, yr 5 $ 30,000
3-year T-note 6/15, yr 5 8/31, yr 5 50,000
5-year T-note 10/1, yr 1 9/30, yr 6 100,000In the fund’s balance sheet, what amount of these investments should be reported as cash and cash equivalents at June 30, year 5?
A
$0
B
$30,000
C
$80,000
D
$180,000

A

C

Explanation:
For purposes of preparing a statement of cash flows, cash equivalents are short-term, highly liquid investments that are both (1) readily convertible into known amounts of cash and (2) so near their maturity that they present insignificant risk of changes in value because of changes in interest rates. Generally, only investments with original maturities to the reporting entity of three months or less qualify as cash equivalents. Thus, for example, both a 3-month U.S. Treasury bill and a 3-year Treasury note purchased three months from maturity qualify as cash equivalents. However, a note purchased three years ago does not become a cash equivalent when its remaining maturity is three months. Therefore, the amount of the investments that Hill should report as cash and cash equivalents at 06/30, year 5 is $80,000, the sum of the 3-month bill and the 3-year note purchased less than three months from maturity. The note with an original maturity to Hill of five years will never be reported as a cash equivalent by Hill.

CLOSE

268
Q

Which of the following is a significant issue for SEA reporting for governmental entities?

A
SEA reporting provides authorization for tax initiatives.
B
SEA reporting will help explain budgetary proposals.
C
SEA reporting helps bring performance results information to citizens to help with their assessment, program selection, and implementation.
D
SEA reporting is required by GASB accounting and reporting standards.

A

C

Explanation:
SEA reporting emphasizes performance results and is designed to help public policy-makers evaluate which programs are critical for a community.

269
Q

A state government collected income taxes of $8,000,000 for the benefit of one of its cities that imposes an income tax on its residents. The state remitted these collections periodically to the city. The state should account for the $8,000,000 in the

A
General fund.
B
Custodial funds.
C
Internal service funds.
D
Special assessment funds.
A

Explanation:
The correct answer is (B).

Custodial Funds are used to account for resources held by a government as an agent for individuals or other governmental units. Taxes, for example, are occasionally collected by one governmental unit on behalf of another. Until the money is physically transferred to the proper authority, it is recorded in a Custodial Fund.

Option (A) is incorrect because the general fund is used to account for resources that are not required to be accounted for in another fund. By accounting for the collected city income taxes in the General Fund, the state would imply that the taxes were available for general purposes.

Option (C) is incorrect because Internal service funds are used to account for services provided to other governmental units as if the service provider were a separate business.

Option (D) is incorrect because Special assessment funds are no longer used.

270
Q

Which of the following transactions should be reported as a liability in the general fund financial statements?

A
An amount that is due within one year of the balance sheet date
B
An amount to be paid from current financial resources
C
An amount set aside to pay for an unfilled contract
D
Principal on long-term debt due 90 days after the next balance sheet date

A

Explanation:
The correct answer is (B).

For general fund financial statements, we need to use modified accrual accounting as per current financial resources approach. In modified accrual accounting, a department will usually not account for long-term debt as this represents future periods and the focus is on a single period here (usually a year). Interest and principal on long-term debt are not recorded until the periods in which they must be paid. Thus, an amount to be paid from current financial resources should be reported as a liability in the general fund financial statements.

(A) is incorrect because this is applicable for current liabilities under accrual accounting.

(C) is incorrect because if it’s an unfilled contract then it should not be recorded as a liability in the current year as there is no certainty of outflow in the current year.

(D) is incorrect because liability is recorded only if they are paid in the current period and not in future periods.

271
Q

Which of the following circumstances are indicative of a public employees’ retirement system (PERS) administering a single plan?
A
Assets are held in a single investment portfolio
B
The plan document requires separate, restricted reserves for certain classes of employees
C
All assets accumulated may be legally used for payment of benefits to any beneficiary
D
The sponsoring government wants to consolidate reporting

A

C

Explanation:
The primary distinction for a single plan is the ability to legally redirect resources among any of the beneficiaries, within the confines of the plan agreement. While it is not recommended, multiple employer plans may use a single portfolio to maximize earnings on investments. A single plan could have separate reserves, but this is not one of its distinguishing characteristics. Consolidated reporting is not available to sponsoring governments.

272
Q
The following equity balances are among those maintained by Cole City:
Enterprise funds	$1,000,000
Internal service funds	400,000Cole's proprietary equity balances amount to
A
$1,400,000
B
$1,000,000
C
$ 400,000
D
$0
A

A

Explanation:
The proprietary funds consist of the enterprise funds and the internal service funds. Therefore, Cole’s proprietary equity balances amount to $1,400,000 (i.e., $1,000,000 + $400,000).

273
Q
Which of the following should be included in the introductory section of a local government’s comprehensive annual financial report?
A
Auditor's report
B
Management letter
C
Engagement letter
D
Letter of transmittal
A

D

Explanation:
The introductory section of a local government’s comprehensive annual financial report would include a title page, a table of contents, and a letter of transmittal. It may also include other material deemed appropriate by management. Issues that may be included in the transmittal letter include, but are not limited to, a profile of the government, information useful in assessing the government’s financial condition, and awards and acknowledgments. The financial section would include the auditor’s report, management’s discussion and analysis (MD&A), basic financial statements, notes, and any required supplementary information.

274
Q

A local municipality had the following account balances and information related to interest on its bonds payable. What amount of interest expenditure would be reported on the statement of revenues, expenditures, and changes in fund balance for the year ended December 31, Year 2?

$50,000 accrued interest on December 31, Year 1
$250,000 in interest payments made during Year 2
$47,000 accrued interest on December 31, Year 2
A
$253,000
B
$250,000
C
$247,000
D
$0

A

B

Explanation:
The statement of revenues, expenditures, and changes in fund balance would report the amount of cash paid in interest during the year as the amount of interest expenditures. The change in accrual would not affect the interest expenditure at the fund level. $250,000 in interest was paid during the year.

275
Q

Which of the following statements about the statistical section of the Comprehensive Annual Financial Report (CAFR) of a governmental unit is true?
A
Statistical tables may not cover more than two fiscal years.
B
Statistical tables may not include nonaccounting information.
C
The statistical section is not part of the basic financial statements.
D
The statistical section is an integral part of the basic financial statements.

A

Explanation:
The basic financial statements include both the government-wide statements and the fund statements, as well as the notes, but not the statistical section.

276
Q

For which of the following governmental entities that use proprietary fund accounting should a statement of cash flows be presented?

Public benefit corporations	Governmental utilities
A	No	No
B	No	Yes
C	Yes	Yes
D	Yes	No
A

C

Explanation:
As per GASB, the statement of cash flows needs to be presented for all proprietary funds. Proprietary funds could either be Internal service fund and Enterprise fund. These follow accrual accounting and economic resources approach. Both Tollway Public Benefit Corporations and Governmental utilities are Enterprise funds where the services are availed through voluntary payments. Thus, the statement of cash flows needs to be presented for both the funds. Options (A), (B) and (D) are incorrect based on the above explanation.

277
Q

The modified accrual basis of accounting should be used for which of the following funds?

A
Capital projects fund
B
Enterprise fund
C
Pension trust fund
D
Proprietary fund
A

Explanation:
The correct answer is (A).

The modified accrual basis is the appropriate basis of accounting for governmental-type funds (i.e., general, special revenue, capital projects, debt service, and permanent funds). The modified accrual basis of accounting for a governmental unit recognizes revenues in the accounting period in which they become available and measurable. Proprietary funds and fiduciary funds use the accrual basis of accounting in all financial statements in the GASB 34 reporting model. The accrual basis of accounting recognizes revenues in the period in which they become earned and measurable.

278
Q

When Rolan County adopted its budget for the current year ending June 30, $20,000,000 was recorded for estimated revenues control. Actual revenues for the fiscal year amounted to $17,000,000. In closing the budgetary accounts at June 30,

A
Revenues control should be debited for $3,000,000.
B
Estimated revenues control should be debited for $3,000,000.
C
Revenues control should be credited for $20,000,000.
D
Estimated revenues control should be credited for $20,000,000.

A

D

Explanation:
The estimated revenues control account of a governmental fund type is a budgetary account (i.e., it is not used to record actual revenues). Its balance is eliminated when the budgetary accounts are closed. The entry to close the estimated revenues control and revenues control accounts to unassigned fund balance is as follows:

Revenues control 17,000,000
Unassigned Fund balance (difference) 3,000,000
Estimated revenues control 20,000,000

279
Q

For funds other than the general fund, the definition of “major” generally is based on which of the following?
A
The cumulative materiality of the fund over five reporting periods
B
The current year’s balance compared to the previous year’s balance
C
An objective assessment of the relative size compared to the government’s totals
D
None of the above

A

C

Explanation:
The definition of “major” for other than the general fund is based on an objective assessment of the relative size compared to the government’s totals. The cumulative materiality of the fund over five reporting periods does not influence the determination of a major fund. The current year’s balance compared to the previous year does not influence the determination of a major fund.

280
Q

A state had general obligation bonds outstanding that required payment of interest on July 1 and January 1 of each year. State law allowed for the general fund to make debt payments without the use of a fiscal agent. The fiscal year ended June 30. Which of the following accounts would have decreased when the state paid the interest due on July 1?

A
Interest expenditures
B
Interest payable
C
Interest expense
D
Fund balance
A

D

Explanation:
The general fund is used to account for most routine operations of the governmental entity. This fund accounts for all resources that are not required to be accounted for in other funds; in essence, it accounts for all unrestricted resources. The general fund uses modified accrual accounting. A fund balance is the difference between governmental fund assets and liabilities reported on the balance sheet. When the state paid the interest due ,it would have reduced the amount of assets (cash) and as such decreased fund balance.Option (a) is incorrect because interest expenditure would be increased not decreased. Option (b) is incorrect because interest payable is recorded when the interest is accrued under accrual basis of accounting not for general funds, which uses modified accrual accounting. Option (c) is incorrect because general fund would report interest expenditure, not interest expense.

281
Q

A city taxes merchants for various central district improvements. Which of the following accounting methods assist(s) in assuring that these revenues are expended legally?

Fund accounting	Budgetary accounting
A	Yes	No
B	No	Yes
C	No	No
D	Yes	Yes
A

D

Explanation:
In the absence of a profit motive, a net income bottom line, or other performance indicators, govern­ments are subjected to a variety of restrictions and controls. The most important are overall legal restrictions on the use of resources and exercise of financial expenditure control through the annual budget. These lead to the use of fund accounting and budgetary accounting.

282
Q

According to GASB Concepts Statement No. 1, governmental financial reporting objectives are focused on providing accountability because of which of the following for governmental entities?
A
They are subject to significant oversight by federal laws and regulations.
B
They cannot change tax rates without public referendum votes.
C
They derive their revenues in large part from involuntary payers.
D
They are exempt from federal income tax.

A

C

Explanation:
Governments derive their revenues in large part from involuntary payers. Tax assessments, license fees, permits, even many user fees are imposed in accordance with statutory authority rather than the payer’s willingness to participate in the transaction. This authority is vested in the governmental entity to protect the public welfare and promote social and economic development. The federal government does provide oversight for state and local government management of federal grant programs; however, the federal government does not establish reporting standards for state and local governments. State and local government tax-exempt status does not include reporting standards. Governmental reporting requirements are established by the Governmental Accounting Standards Board.

283
Q

GASB Statement No. 34, Basic Financial Statements and Management’s Discussion and Analysis for State and Local Governments, requires small governments (revenues of less than $10 million) to
A
Report and depreciate all buildings, and equipment, except for infrastructure assets.
B
Report and depreciate all buildings, and equipment, and report all major general infrastructure assets.
C
Report and depreciate all buildings, equipment, and major general infrastructure assets.
D
Report and depreciate all buildings and equipment as well as all major general infrastructure assets acquired in fiscal periods beginning after June 15, 2003.

A

b or C idk

Explanation:
GASB 34 requires all governments to report all new capital assets at historical cost. Capital assets include assets formerly listed in the general fixed asset account group (land, buildings, equipment) and infrastructure assets. Capital assets should be depreciated over their estimated useful lives unless they are either inexhaustible or are eligible infrastructure assets reported using the modified approach. The retroactive infrastructure reporting requirements are optional for small governments.

284
Q

The following events relating to the City of Arrow’s debt service funds occurred during the year ended December 31: All principal and interest due in the year were paid on time.
Debt principal matured $ 3,000,000
Unmatured (accrued) interest on outstanding debt at Jan 1 45,000
Interest on matured debt 700,000
Unmatured (accrued) interest on outstanding debt at Dec 31 93,000
Interest revenue from investments 800,000
Cash transferred from general fund for retirement of debt principal 2,000,000
Cash transferred from general fund for payment of matured interest 600,000How much revenue should Arrow’s debt service funds record for 20X1?
A
$800,000
B
$2,600,000
C
$2,800,000
D
$3,400,000

A

A

Explanation:
Debt service fund budgetary accounts may be used to record estimated revenues from taxes or other financing sources, or estimated investment earnings. Actual amounts are then recorded as revenues. The operating transfer from the general fund is recorded as an other financing source; not revenue.

285
Q

In preparing Chase City’s reconciliation of the statement of revenues, expenditures, and changes in fund balances to the government-wide statement of activities, which of the following items should be subtracted from changes in fund balances?

A
Capital assets purchases.
B
Payment of long-term debt principal.
C
Internal service fund increase in net position.
D
Book value of capital assets sold during the year.
A

D

Explanation:
The elimination of the book value of a previously acquired asset has no impact on fund financial statements but serves to reduce the changes in net position of the government-wide financial statements as the asset is written off (net of proceeds and accumulated depreciation). The book value of capital assets is, therefore, essentially subtracted from change in fund balance to reconcile to the change in government-wide net position. Actual reconciling items might focus more on the computation of a gain or loss and proceeds from the disposal rather than individual components such as the asset book value

286
Q

For governmental external financial reporting, the budget discussion of Management’s Discussion and Analysis should analyze significant variations between:

I. Final budget amounts and initial expectations of results

II. Final budget and actual results

III. Original and final budget

A
I only
B
I & II
C
II & III
D
I, II, & III
A

Explanation:
The correct answer is (C).

The budget discussion section should provide an analysis of significant variations between original and final budget amounts as well as the variances between budget and actual.

It should explain why the budget was changed, not just how much it changed.

287
Q

A city had to make significant repairs to a building damaged by flooding of a nearby river. Damage due to flooding is not unusual but has been infrequent in the last few years. How should the city’s general fund report the cost of the repairs in its fund financial statements?

A
As an expenditure.
B
As another financing use.
C
As a special item.
D
As an extraordinary item.
A

Explanation:
The correct answer is (A)

In accordance with GASB 33, governmental fund accounting is based on modified accrual basis and current financial resources approach. Repairs to damaged buildings are treated as current-year expenditures.

Repair is reported as expenditures and not as other uses of funds. Payment of interest and principal of long term debt is an example of other uses of funds reported on the statement of revenues, expenditures, and changes in fund balances

288
Q

Which fund would include a statement of cash flows among its financial statements?

A
General fund
B
Capital projects funds
C
Proprietary funds
D
None of the above
A

Governmental funds do not provide a statement of cash flows, while proprietary funds are required to include a statement of cash flows among their financial statements. This means that C is the only correct answer among the choices listed.

289
Q
For a municipal solid waste landfill, equipment and facilities included in the estimated total current cost of closure and postclosure care are reported in the statement of net position as which of the following?
A
Current expenses
B
Capital assets
C
A reduction of the liability
D
None of the above
A

Explanation:
Equipment and facilities included in the estimated total current cost of closure and postclosure care are not reported as current expenses, but as a reduction of the liability. A liability that would be excluded from the “capital-related debt” portion of the statement of net position is the liability for closure and postclosure care associated with a municipal solid waste landfill.

290
Q

Note disclosures in governmental external financial statements related to cash and cash equivalents explain which of the following?
A
How the government classified cash equivalents in the beginning and ending balances on the enterprise fund cash flow statements.
B
How the government classified cash flows among the four categories in the cash flows statements
C
Details of the maturities for cash equivalents included in the ending balance on the cash flows statements
D
Details of which cash equivalents are included in the operating flows section of the cash flows statement.

A

A

Explanation:
Note disclosures related to cash and cash equivalents explain how cash equivalents are classified in the beginning and ending balances on the cash flows statement. How governments classify cash flows in the cash flows statement is outlined in GASB Statement No. 9. Governments are not required to disclose the details of maturities of cash equivalents included in the cash flows statement. Cash equivalents are not distributed among the four categories of the cash flows statement.

291
Q
Gold County received goods that had been approved for purchase but for which payment had not yet been made. Should the accounts listed below be increased?
Encumbrances	Expenditures
A	No	No
B	No	Yes
C	Yes	No
D	Yes	Yes
A

B

Explanation:
The following entries would be made by a governmental unit when ordering and receiving goods:

To record purchase order issued when goods were approved for purchase:
Encumbrances (expected cost) XX
Reserve for encumbrances XX
To record expenditure upon receipt of the goods:
Reserve for encumbrances XX
Encumbrances XX
Expenditures (actual) XX
Vouchers payable XXThus, upon receipt of the goods, the Encumbrance account decreases and the Expenditures account increases.

292
Q

Which of the following are the three components of net position for governmental business-type activities?
A
Restricted, committed, and assigned
B
Restricted, assigned, and unrestricted
C
Net assets invested in capital assets (net of related debt), committed, and assigned
D
Net investment in capital assets, restricted, and unrestricted

A

D

Explanation:
Net investments in capital, restricted, and unrestricted are reported for net position in the proprietary fund statements and in the government-wide statements. Committed and assigned are components of governmental fund balances.

293
Q
Palm City acquired, through forfeiture as a result of nonpayment of property taxes, a parcel of land that the city intends to use as a parking lot for general governmental purposes. The total amount of taxes, liens, and other costs incurred by Palm incidental to acquiring ownership and perfecting title was $20,000. The land's fair market value at the forfeiture date was $60,000. What amount should be reported in the governmental activities column of the governmentwide financial statements for this land?
A
$0
B
$20,000
C
$60,000
D
$80,000
A

B

Explanation:
General fixed assets acquired by foreclosure are recorded at the lower of (1) the amount due for taxes, special assessments, penalties and interest, plus foreclosure costs or (2) appraised fair market value. Therefore, since the $20,000 Palm incurred for taxes, liens, and other costs incidental to acquiring ownership and perfecting title is less than the land’s $60,000 fair market value at the forfeiture date, the land should be reported in the governmental activities column of government-wide financial statements at $20,000.

294
Q

The following information pertains to Comb City:
Year 3 real estate property taxes assessed and collected in year 3 $14,000,000
Year 2 real estate property taxes assessed in year 1 and collected in year 3 1,000,000
Year 3 sales taxes collected by merchants in year 3 but not required
to be remitted to Comb until January of year 4 2,000,000For the year ending December 31, year 3, Comb should recognize revenues of
A
$14,000,000
B
$15,000,000
C
$16,000,000
D
$17,000,000

A

D

Explanation:
Governmental-type funds use the modified accrual basis of accounting, with revenues recognized when they become measurable and available for use. “Available for use” means that the revenues will be collected within the current period or collected early enough in the next period to be used to pay for expenditures incurred in the current period. (The rule of thumb for available for use is that the revenue is collected within 60 days.) GASB No. 22 amends the method continued by GASB No. 1 to include self-assessed income and sales taxes among accruable revenues. Five classes of revenue that may be accrued if available for use and measurable are: (1) personal property taxes, (2) real estate property taxes, (3) income taxes, (4) sales taxes, and (5) taxes collected by another governmental unit. ($14,000,000 + $1,000,000 + $2,000,000 = $17,000,000).

295
Q

Tott City’s serial bonds are serviced through a debt service fund with cash provided by the general fund. In a debt service fund’s statements, how are cash receipts and cash payments reported?
Cash receipts Cash payments
A Revenues Expenditures
B Revenues Operating transfers
C Operating transfers Expenditures
D Operating transfers Operating transfers

A

C

Explanation:
The debt service fund of a governmental unit is used to account for accumulation of resources for, and the payment of, general long-term debt principal and interest. The debt service fund reports cash receipts from the general fund as operating transfers. Cash payments for long-term debt principal and related interest are reported as expenditures in the fund.

296
Q

Cal City maintains several major fund types. The following were among Cal’s cash receipts:

Unrestricted state grant $1,000,000
Interest on bank accounts held for employees’ pension plan 200,000
What amount of these cash receipts should be accounted for in Cal’s general fund?

A
$1,200,000
B
$1,000,000
C
$ 200,000
D
$0
A

Explanation:
The $1,000,000 unrestricted grant received from the state should be accounted for as revenue in Cal City’s general fund. The $200,000 of interest received on bank accounts held for employees’ pension plans should be accounted for in Cal City’s pension trust fund.

297
Q

Arbor City has adopted GASB Statement No. 34, Basic Financial Statements and Management’s Discussion and Analysis for State and Local Governments. What basis of accounting does Arbor use to present the general fund in the financial statements?

Modified accrual basis in all financial statements.
Accrual basis in the government-wide statement of activities and statement of net position.
Modified accrual basis in the fund financial statements with a reconciliation to the accrual basis.
A
I only
B
II only
C
III only
D
Both II and III

A

C maybe

Explanation:
Government-wide statements aggregate information for all governmental and business-type activities on the accrual basis of accounting. Fund financial statements use modified accrual and a reconciliation to the government-wide statement must appear on the face of the governmental-type fund financial statements or in a separate schedule.

298
Q

Which of the following is a governmental fund that uses the current financial resources measurement focus?

A
Enterprise fund
B
Internal service fund
C
Special revenue fund
D
Private-purpose trust fund
A

Explanation:
The correct answer is (C).

Governmental Funds – General Fund, Special Revenue Fund, Capital Projects Fund, Debt Service Fund, and Permanent Fund use modified accrual accounting and current financial resources measurement focus.

(A), (B) and (D) are incorrect because Enterprise Fund and Internal Service Fund are Proprietary Funds and a Private-Purpose Trust Fund is a Fiduciary Fund which all use the economic resource approach

299
Q

At the beginning of the current year, Paxx County’s enterprise fund had a $125,000 balance for accrued compensated absences. At the end of the year, the balance was $150,000. During the year, Paxx paid $400,000 for compensated absences. What amount of compensated absences expense should Paxx County’s enterprise fund report for the year?

A
$375,000
B
$400,000
C
$425,000
D
$550,000
A

Explanation:
Enterprise funds must be used to account for a government’s business-type operations that are financed and operated like private businesses–where the government’s intent is that all costs of providing goods or services to the general public on a continuing basis are to be recovered primarily through user charges (operating revenue). Enterprise funds use normal accrual accounting. Compensated absence expense is $425,000 ($400,000 + $150,000-$125,000).

300
Q

Property taxes levied in fiscal year 1 to finance the general fund budget of fiscal year 2 should be reported as general fund revenues in fiscal year 2 of governmental funds Financial Statements:

A
Regardless of the fiscal year in which collected
B
For the amount collected before the end of fiscal year 2 or shortly thereafter
C
For the amount collected before the end of fiscal year 2 only
D
For the amount collected in fiscal year 1 only

A

Explanation:
The correct answer is (B).

Governmental funds use the modified accrual basis of accounting, under which, revenues are recognized when they become measurable and available for use.

“Available for use” means that the revenues will be collected within the current period or collected early enough in the next period to be used to pay for expenditures incurred in the current period.

Therefore, the property taxes collected before the end of fiscal year 2 or shortly thereafter should be reported as revenues in fiscal year 2.

301
Q

The government-wide financial statements present which of the following?
A
Consolidation of governmental, proprietary, and fiduciary funds
B
Information to assess fiscal accountability
C
The corporate model for the operating statements
D
Information to assess operational accountability

A

D

Explanation:
Key objectives for the government-wide financial statements are to view the primary government’s financial position and to provide operational accountability for providing public programs and services. The government-wide financial statements do not include fiduciary funds because the governmentwide financial statements are intended to give the reader a view of the financial health of the government. Since fiduciary funds are not resources that the government can use or tap into, the balances of these funds would not positively or negatively impact the government’s financial health. The governmental fund statements provide fiscal accountability based on the modified accrual basis of accounting and current financial resources measurements.

302
Q

Central County received proceeds from various towns and cities for capital projects financed by Central’s long-term debt. A special tax was assessed by each local government, and a portion of the tax was restricted to repay the long-term debt of Central’s capital projects. Central should account for the restricted portion of the special tax in which of the following funds?

A
Internal service fund
B
Enterprise fund
C
Capital projects fund
D
Debt service fund
A

Explanation:
The correct answer is (D).

Debt service funds account for the accumulation of resources for, and the payment of, general long-term debt principal and interest.

Therefore, the portion of the special tax that is restricted to repay the long-term debt of the government’s capital projects fund should be accounted for in a debt service fund.

Internal service funds are used to account for the activities of a department that provides services to other departments of the same government in a manner similar to a business. Internal service funds (a type of proprietary fund) account for only their own assets, liabilities, revenues, and expenses.

Enterprise funds are used to account for the activities of a department that provides services to others in a manner similar to a business. Enterprise funds (a type of proprietary fund) account for only their own assets, liabilities, revenues, and expenses.

Capital project funds are used to account for the construction of capital projects.

303
Q
King City Council will be establishing a library fund. Library fees are expected to cover 55% of the library's annual resource requirements. King has decided that an annual determination of net income is desirable in order to maintain management control and accountability over library. What type of fund should King establish in order to meet their measurement objectives?
A
Special revenue fund.
B
General fund.
C
Internal service fund.
D
Enterprise fund.
A

Explanation:
Enterprise funds must be used to account for a government’s business-type operations that are financed and operated like private businesses—where the government’s intent is that all costs of providing goods or services to the general public on a continuing basis are to be financed or recovered primarily through users charges. Governments are permitted to account for virtually any type of self-contained business-type activity in enterprise funds if it prefers to do business-type accounting rather than general government accounting. An enterprise fund is a proprietary fund.

304
Q

The significant accounting policies note in governmental financial statements must explain the measurement focus and basis of accounting for which of the following?

A
The fund and government-wide financial statements, and required supplementary information (RSI)
B
Only the government-wide financial statements
C
Only the fund financial statements
D
Both the fund and government-wide financial statements

A

Explanation:
The correct answer is (D)

The GASB requirement calls for an explanation of the measurement focus and basis of accounting for the government-wide statements and fund financial statements.

305
Q

Brandon County’s general fund had the following transactions during the year:

Transfer to a debt service fund $100,000
Payment to a pension trust fund $500,000
Purchase of equipment $300,000
What amount should Brandon County report for the general fund as other financing uses in its governmental funds statement of revenues, expenditures, and changes in fund balances?

A
$100,000
B
$400,000
C
$800,000
D
$900,000
A

A

Explanation:
Transfer to a debt service fund is an operating transfer (or inter-fund transfers). Since government funds use modified accrual basis of accounting $100,000 is reported as other financing uses. Payment to pension trust and purchase of equipment are both expenditures. Option (b) is incorrect because it includes purchase of equipment which is expenditure, not to be classified as other financing uses. Option (c) is incorrect because payment to pension trust and purchase of equipment are both expenditures, not other financing uses. Option (d) is incorrect because it includes payment to pension trust and purchase of equipment are both expenditures, not other financing uses.

306
Q

The primary authoritative body for determining the measurement focus and basis of accounting standards for state and local governments is:

A
Governmental Accounting Standards Board (GASB).
B
Financial Accounting Standards Board (FASB).
C
Government Accounting and Auditing Committee of the AICPA (GAAC).
D
National Council on Governmental Accounting (NCGA).

A

Explanation:
The correct answer is (A).

The GASB sets accounting and financial reporting standards for state and local governments. The FASB sets standards for financial reports published by business and nonprofit enterprises.

Option (B), (C) and (D) are incorrect as per above explanation.

307
Q

Which of the following statements meet the measurement and recognition criteria for landfill closure and postclosure costs?
A
Landfills should only be accounted for in the general fund.
B
Total landfill liabilities should be recognized in the general long-term debt account group.
C
Expense recognition should begin when waste is accepted and should continue through the post closure period.
D
Equipment and facilities included in estimated total current cost of closure and post closure care should not be reported as capital assets.

A

D

Explanation:
Proprietary funds may account for landfills. Liabilities are recognized in a proprietary fund when a proprietary fund accounts for the landfill. Expense recognition should be finished when the post-closure period begins. Capital assets should be fully depreciated by the date the MSWLF stops accepting solid waste.

308
Q

On January 2, Basket ville City purchased equipment with a useful life of three years to be used by its water and sewer enterprise fund. Which of the following is the correct treatment for the asset?

A
Record the purchase of the equipment as an expenditure
B
Capitalize; depreciation is optional.
C
Capitalize; depreciation is required.
D
Capitalize; depreciation is not permitted.
A

C

Explanation:
Enterprise fund is a proprietary fund which uses accrual basis of accounting and applies economic resources approach. Preferred approach also called depreciation approach is to account for the assets and recognize depreciation. Option (a) is incorrect because purchase of equipment is an expenditure in modified accrual accounting. Option (b) and (d) are incorrect as per the above explanation

309
Q
Proceeds of General Obligation Bonds is an account of a governmental unit that would be included in the
A
Debt service fund.
B
Internal service fund.
C
Capital projects fund.
D
Enterprise fund.
A

Explanation:
Capital projects funds are used to account for the acquisition and use of financial resources to construct or otherwise acquire long-lived “general government” real property and equipment. Project resources, which include proceeds of general obligation bonds, are recorded as received or accrued.

310
Q
During the current year, Wythe County levied $2,000,000 property taxes, 1% of which is expected to be uncollectible. During the year, the county collected $1,800,000 and wrote off $15,000 as uncollectible. What amount should Wythe County report as property tax revenue in its government-wide statement of activities for the current year?
A
$1,800,000
B
$1,980,000
C
$1,985,000
D
$2,000,000
A

B

Explanation:
Imposed nonexchange revenues represent assessments imposed on non-governmental entities and include property taxes and fines or forfeitures. Governments should recognize revenues from property taxes, net of estimated refunds and estimated uncollectible amounts, in the period for which the taxes are levied, even if the enforceable legal claim arises or the due date for payment occurs in a different period. All other imposed nonexchange revenues should be recognized in the same period that the assets are recognized unless the enabling legislation includes time requirements. If so, revenues should be recognized in the period when the resources are required to be used or when use is first permitted. (Resources received or recognized as receivable before that period should be reported as deferred revenues.) The $1,980,000 is derived from the total $2,000,000 levied less $20,000 (the 1% of $2,000,000) expected to be uncollectible. The $15,000 written off as uncollectible does not factor anywhere in the equation.

311
Q

At the end of the fiscal year, a state government reported capital assets of $20 million, accumulated depreciation of $5 million, restricted assets of $ 3 million, and liabilities of $7 million. What amount should the government report as the total net position in its government-wide financial statements?

A
$8 million
B
$11 million
C
$15 million
D
$18 million
A

Explanation:
The correct answer is (B).

Total net position in the government-wide financial statements consists of:

Net investment in capital assets (Capital assets net of accumulated depreciation and obligations) = $20,000,000 - $5,000,000 = $15,000,000
Restricted net position (Restricted assets net of liabilities and deferred inflows of resources related to restricted assets)
Unrestricted net position (Any residual amount after considering net capital assets and net restricted assets).
Net position in its government-wide financial statements = Assets – Liabilities = $15,000,000 + $3,000,000 - $7,000,000 = $11,000,000.

312
Q

GASB Statement No. 34, Basic Financial Statements and Management’s Discussion and Analysis for State and Local Governments, requires large governments (annual revenues in the range above $100 million) to

Report all fixed assets (land, buildings, and equipment).
In fiscal periods beginning after June 15, 2001, report major general infrastructure asset acquisitions.
In fiscal periods beginning after June 15, 2005, retroactively report major general infrastructure assets acquired in fiscal years ending after June 30, 1980, and before adoption of GASB Statement No. 34.
A
I only
B
I and II only
C
I and III only
D
I, II, and III

A

D maybe

Explanation:
GASB 34 requires all governments to report all current fixed assets and new capital assets. Capital assets include assets formerly listed in the general fixed asset account group (land, buildings, and equipment) and infrastructure assets. The retroactive infrastructure asset reporting requirements are required for the largest two categories of governments and optional for small governments (annual revenues under $10 million). If determining the actual historical cost of general infrastructure assets is not practical because of inadequate records, governments should report the estimated historical cost for major general infrastructure assets that were acquired or significantly reconstructed, or that received significant improvements, in fiscal years ending after June 30, 1980. Capital assets should be depreciated over their estimated useful lives unless they are either inexhaustible or are eligible infrastructure assets reported using the modified approach; however, the modified approach condition assessment is not one of the answer options.

313
Q
Except in specified circumstances, interest costs on long-term liabilities are reported under governmental accounting rules in which of the following manners?
A
Separately as a direct program cost
B
Separately as an indirect cost
C
In a particular note to the financial statements
D
None of the above
A

Explanation:
In general, interest costs on long-term liabilities will be reported separately as an indirect cost (not in a particular note). Note disclosures will include information about how interest costs are reported and what amounts are included as direct expenses.

314
Q

GASB Statement No. 34, Basic Financial Statements and Management’s Discussion and Analysis for State and Local Governments, does not require depreciation of which of the following assets purchased after June 30, 1980?

Land and land improvements
Historical treasures and works of art that meet the conditions of an inexhaustible collection
Infrastructure assets that are part of a network or subsystem of a network, when the modified approach is used
A
I only
B
I and II only
C
I and III only
D
I, II, and III
A

D

Explanation:
Unless held as a collection, works of art, historical treasures, and similar assets should be capitalized at historical cost or fair value at date of donation. GASB 34 merely encourages capitalization of these assets when held as collections. Collections are held for public exhibition, education, or research in furtherance of public service rather than financial gain; protected, kept unencumbered, cared for, and preserved; and are subject to policies that require sales proceeds be used to acquire collection items. An exhaustible capitalized collection must be depreciated over its estimated useful life. Eligible infrastructure assets are not required to be depreciated, if the government meets certain conditons, including documenting that the assets are preserved approximately at (or above) a condition level established and disclosed by the government. Examples of infrastructure assets include roads, bridges, tunnels, drainage systems, water and sewer systems, dams, and lighting systems. Buildings, except those that are an ancillary part of a network of infrastructure assets, should not be considered infrastructure assets for purposes of GASB 34. Land and land improvements are not depreciable assets.

315
Q
Farmer Township has adopted all provisions of GASB Statement No. 34, Basic Financial Statements and Management's Discussion and Analysis for State and Local Governments. In which financial statements does Farmer present its general-use capital assets?
Fund	Government-wide
A	Yes	Yes
B	Yes	No
C	No	Yes
D	No	No
A

C

Explanation:
General capital assets are not specifically related to activities reported in proprietary or fiduciary funds. They are associated with, and generally arise from, governmental activities. They should not be reported as assets in governmental funds, but should be reported in the governmental activities column in the government-wide statement of net position.

316
Q

What is the basic criterion used to determine the reporting entity for a governmental unit?

A
Special financing arrangement.
B
Geographic boundaries.
C
Scope of public services.
D
Financial accountability.
A

Explanation:
Governmental reporting model requires financial accounting and disclosures to show (a) operational accountability when the entity is taken as a whole, the extent to which operating objectives have been met (effectively and efficiently) and (b) fiscal accountability for fund financial statements to demonstrate that the entity has been in compliance with public decisions concerning the raising and spending of public funds during a budgetary cycle. Options (a), (b) and (c) are not the basic criterion used to determine the reporting entity of a governmental unit.

317
Q
Operating transfers received by a governmental-type fund should be reported in the Statement of Revenues, Expenditures, and Changes in Fund Balance as a(an)
A
Addition to contributed capital.
B
Addition to retained earnings.
C
Other financing source.
D
Reimbursement.
A

C

Explanation:
Operating transfers should be recorded in distinctive Operating Transfers From (To) [Name of Fund] or Operating Transfers In (Out) accounts and reported after revenues and expenditures or expenses, but before determining the results of operations in the fund’s operating statement.

318
Q
Powell City purchased a piece of equipment to be used by a department financed by the general fund. How should Powell report the acquisition in the general fund?
A
As an expenditure
B
Capitalize, depreciation is optional
C
Capitalize, depreciation is required
D
Capitalize, depreciation is not permitted
A

Explanation:
The modified accrual basis of accounting is used in the governmental-type fund statements such as the general fund. Under the modified accrual basis of accounting, fixed assets are expenditures and not capitalized.

319
Q

Dogwood City’s water enterprise fund received interest of $10,000 on long-term investments. How should this amount be reported on the Statement of Cash Flows?

A
Operating activities
B
Non-capital financing activities
C
Capital and related financing activities
D
Investing activities
A

Explanation:
The correct answer is (D).

According to GASB 9, interest received by a governmental proprietary fund will be reported in the investing activities section of the cash flow statement. Thus, Dogwood, a water enterprise fund would report $10,000 interest received on long term investments in the investing activities section.

Option (A) is incorrect because only commercial enterprises that follow GAAP report interest and dividend received in the operating section of the cash flow statement. Governmental proprietary funds that need to present their cash flow statement follow GASB and accordingly present any interest received in the investing section of the cash flow statement.

Option (B) is incorrect because non-capital financing section of the cash flow statement includes borrowing money for purposes other than to acquire, construct or improve capital assets and repaying those amounts borrowed, including interest. It also includes certain inter-fund and intergovernmental receipts and payments such as grants and subsidies for purposes other than those related to capital assets. Thus, interest received on long term investments, would not form part of non-capital financing section.

Option (C) is incorrect because capital and related financing section include

acquiring and disposing of capital assets used in providing services or producing goods,
borrowing money for acquiring, constructing, or improving capital assets and repaying the amounts borrowed, including interest, and
paying for capital assets obtained from vendors on credit.
Thus, interest received on long term investments, would not form part of non-capital financing section

320
Q

Which of the following statements are required to be presented for special-purpose governments engaged only in business-type activities (such as utilities)?

A
Statement of net assets only.
B
Management’s Discussion and Analysis (MD&A) and Required Supplementary Information (RSI) only.
C
The financial statements required for governmental funds, including MD&A.
D
The financial statements required for enterprise funds, including MD&A and RSI.

A

Explanation:
(d) Activities that are normally financed through user charges and reports the consolidated results of all enterprise funds. It does not include internal service funds, which are accounted for in the governmental activities. The required financial statements are:

Statement of net assets
Statement of revenues and expenses
Statement of cash flows
Notes to the F/S
Required supplementary information other than MD&A
Option (a) is incorrect because along with basic F/S, MD&A and RSI are also required. Option (b) is incorrect because basic F/S should also be included. Option (c) is incorrect because it is not governmental funds, but enterprise funds.

321
Q

Several Bay City employees accepted a voluntary termination plan during year 1. The city will pay a set amount to the terminated employees as follows:

During year 1 $20,000
End of year 2 $30,000
End of year 3 $15,000
The discounted present value of the payment for year 2 and year 3 is $42,000. What amount of expenditures should be reported in the governmental fund-level financial statements in year 1?

A
$20,000
B
$50,000
C
$62,000
D
$65,000
A

Explanation:
The correct answer is (A).

Because governmental fund-level financial statements use the current financial resources approach, only the $20,000 due from current financial resources will be included. When the flow of current financial resources measurement focus is used, the focus is on cash and/or assets that are expected to be converted to cash within the accounting period.

322
Q

Government-wide statements use which of the following measurement principles?
A
Modified accrual for governmental activities
B
Economic resources for all activities
C
Modified accrual for fiduciary activities
D
Current financial resources for governmental activities

A

Explanation:
The government-wide statements, both the statement of net position and the statement of activities, are prepared using the economic resource measurement focus and accrual accounting to present the long-term perspective of financial position. Modified accrual is only used in the governmental fund statements. Fiduciary activities are not reported in the government-wide statements. Current financial resources measurement focus is only used in the governmental fund statements

323
Q
Wood City, which is legally obligated to maintain a debt service fund, issued the following general obligation bonds on July 1:
Term of bonds	10 years
Face amount	$1,000,000
Issue price	101
Standard interest rate	6%Interest is payable January 1 and July 1. What amount of bond premium should be amortized in Wood's debt service fund for the year ended December 31?
A
$1,000
B
$ 500
C
$ 250
D
$0
A

Explanation:
Governmental funds do not defer and amortize a bond premium or discount over the life of the bonds. Bond issue proceeds are recorded in the appropriate governmental fund at the amount received, net of any bond premium or discount.

324
Q

Which of the following is a required financial statement for an investment trust fund?
A
Statement of revenues, expenditures, and changes in fiduciary net position
B
Statement of activities
C
Statement of revenues, expenses, and changes in fiduciary net position
D
Statement of changes in fiduciary net position

A

Explanation:
Because an investment trust fund accounts for resources (and related liabilities) held by governments in a trustee capacity, it is considered a fiduciary fund type. Required statements for fiduciary funds are (1) statement of fiduciary net position and (2) statement of changes in fiduciary net position.

325
Q

The city accountant for a newly established municipality is setting up the new fund structure for the city’s accounting system. How many funds should the accountant establish for the city?

A
Two, a general fund and a special revenue fund as required by GAAP.
B
Two, a special revenue fund and a general fund as required by the city manager.
C
The minimum number of funds consistent with the needs of the city accountant.
D
The minimum number of funds consistent with legal requirements and sound financial administration.

A

Explanation:
A government unit must have one general fund, but can establish multiple special revenue, capital projects, and permanent funds as needed to account for the various activities that fit these categories. A debt service fund is only needed if the entity issues general obligation debts. The accountant must establish the minimum number of funds consistent with legal requirements and sound financial administration. Option (a) is incorrect because one general fund should be set up and multiple special revenue funds can be set up as needed. Option (b) is incorrect because more than 2 special revenue funds can be set up if required. Option (c) is incorrect because minimum number of funds consistent with GASB 34 should be setup not as per the needs of the accountant.

326
Q

In Soan County’s general fund statement of revenues, expenditures, and changes in fund balances, which of the following has an effect on the excess of revenues over expenditures?

A
Purchase of fixed assets
B
Payment to a debt-service fund
C
Special items
D
Proceeds from the sale of capital assets
A

Explanation:
General fund uses modified accrual accounting, in which purchase of fixed asset is an expenditure, which would affect the excess of revenue over expenditure. Option (b) is incorrect because transfers out are reported as other financing uses which is presented below the excess of revenues (deficiency) over expenditure. Option (c) is incorrect because special items are presented below the excess of revenues (deficiency) over expenditure. Option (d) is incorrect because proceeds from sale are reported in special items, which is presented below the excess of revenues (deficiency) over expenditure

327
Q

In which situations(s) should property taxes due to a governmental unit be recorded as deferred revenue?

Property taxes receivable are recognized in advance of the year for which they are levied.
Property taxes receivable are collected in advance of the year in which they are levied.
A
I only
B
Both I and II
C
II only
D
Neither I nor II
A

B

Explanation:
Revenues are recognized when measurable and available. In the case of property taxes, receivables are recognized in the period in which the taxes are levied. When property taxes are either recognized as receivable in advance or collected in advance of the year in which they are levied, deferred inflows of resources are recorded.I.

Debit (Dr) Credit (Cr)
Property taxes receivable $ XXX
Deferred inflows of resources $ XXX
II.

Debit (Dr) Credit (Cr)
Cash $ XXX
Deferred inflows of resources $ XXX
Note: Some individuals may elect to pay their property taxes in installments. In order to accomplish this, taxes must be estimated and billed and collected in advance of the levy.Choices “b”, “c”, and

328
Q

A government-wide statement of net position must include which of the following?

A
Prior-year comparative financial data
B
Primary government fiduciary fund data
C
A consolidation of all government-wide activities
D
A distinction between governmental and business-type activities
A

Explanation:
The correct answer is (D).

A government-wide statement of net position must include a distinction between governmental and business-type activities.

The statement contains four columns:-

Governmental activities: This column contains details of activities that are financed primarily through taxes and other non-exchange transactions. This column reports on the consolidated results of all the governmental funds and internal service funds.
Business-type activities: This column contains activities normally financed through user charges. This column reports the consolidated results of all enterprise funds.
Total: This column simply adds together the amounts from the two primary government columns for governmental activities and business-type activities.
Component units: This column reports the combined results of all the component units for which separate reporting was selected.
Therefore, the governmental activities and business-type activities of the primary government are separately presented.

329
Q
For state and local governmental units, generally accepted accounting principles require that encumbrances outstanding at year-end be reported as
A
Expenditure.
B
Reservations of fund balance.
C
Deferred liabilities.
D
Current liabilities.
A

Explanation:
To assure that budgets are not overspent, most governmental units use an encumbrance system to keep track of outstanding purchase commitments that have not yet resulted in liabilities. When a purchase order or commitment is placed, the governmental unit reduces the amount of budgetary authority remaining in the budget category and records an outstanding encumbrance. It is only when the goods or services have been received that the encumbrance balance is reduced and the expenditure and a liability are recorded. Thus, encumbrances represent neither expenditures nor liabilities. Encumbrances outstanding at the end of a period should be carried forward as a reservation of fund balance, with a corresponding reduction in unreserved fund balance.

330
Q

Elliott City has a general fund, a water billing fund, a capital projects fund, and a special revenue fund. In which fund is depreciation recorded on the statement of revenues, expenses, and changes in net position?

A
Water billing fund
B
General fund
C
Capital projects fund
D
Special revenue fund
A

Explanation:
Of the funds listed, one is a proprietary fund (water billing fund) and the remainder are governmental funds. At the fund level, governmental funds record purchases of capital assets as expenditures and the assets are not capitalized. For this reason, governmental funds do not record depreciation at the fund level. The correct answer is the water billing fund as proprietary funds DO record depreciation.

331
Q

Which of the following funds of a governmental unit uses the modified accrual basis of accounting?

A
Internal service funds
B
Enterprise funds
C
Special revenue funds
D
Private-purpose trust funds
A

Explanation:
The correct answer is (C).

The modified accrual basis is the appropriate basis of accounting for governmental funds (i.e., general, special revenue, capital projects, permanent, and debt service).

332
Q

Which of the following is discussed in GASB Concepts Statement No. 1 as a financial reporting objective for governmental financial statements?
A
Assisting users in understanding funding objectives for public retirement systems and the potential tax burden on future taxpayers
B
Assisting users in identifying management’s policies for selection of critical programs and services to provide and the annual tax burden to finance those programs
C
Assisting users in assessing the operating results, level of services provided, and the government’s ability to meet cash requirements
D
Assisting users in determining the annual tax burden per capita for each of the major programs provided by the governmental entity

A

Explanation:
Assisting users with an assessment of the government’s operating results, level of services provided, and the ability to meet cash requirements is a broad principle and is the second objective outlined in Concepts Statement No. 1. While assisting users in understanding funding objectives for public retirement systems and the potential tax burden on future taxpayers may be one of the significant elements for any governmental financial report, it is not specifically discussed as a reporting objective. Assisting users in identifying management’s policies for selection of critical programs and services to provide and the annual tax burden to finance those programs is an example of a specific reporting issue, not a broad principle. Assisting users in determining the annual tax burden per capita for each of the major programs provided by the governmental entity is an example of specific information that is critical to users, but not discussed as a primary broad objective.

333
Q
A city council designates funds in the enterprise fund for future equipment replacement. The enterprise fund should report this as
A
A restricted component of net position.
B
A net investment in capital assets.
C
A designated component of net position.
D
An unrestricted component of net position.
A

Explanation:
Enterprise funds are a type of proprietary fund. Their statement of financial position can use either the balance sheet format or the net position format from the government-wide statements. Net position is presented in three components: net investment in capital assets, restricted, and unrestricted. The invested total provides a clear picture of whether the government owns the capital assets or creditors have a stronger claim. The restricted component carves out those resources that are not generally available for any programs, but will be used for specific purposes. The unrestricted component gives readers a measure of the government’s financial strength. The city designating funds in the enterprise fund for future equipment replacement displays the city’s forward looking view and provides the readers with a measure of the government’s financial strength. Although they are designated for that purpose, they are not restricted from being used for any other programs.

334
Q

Which of the following is the purpose for governmental fund accounting?
A
Track subsidiary profit centers
B
Evaluate revenues and expenditures for specific programs and monitor compliance with legal restrictions
C
Separate revenues and expenditures into as many self-supporting accounts as possible
D
Identify statutory requirements for budget management

A

Explanation:
A government’s primary mission is the delivery of programs and services. Fund accounting allows governments to track specific functions and ensure that operations comply with any legal restrictions that apply. Governments may have component units that are similar to subsidiaries in the private sector, but there are no profit objectives in governmental financial reporting. The GASB discourages the use of too many funds; the general fund is the only required fund. GASB Statement No. 34 reorganized funds to create a primary focus for each fund category, but statutory identification is not a primary objective for fund organization.

335
Q
Which of the following items is an example of imposed nonexchange revenue for a governmental entity?
A
Personal income taxes
B
Retail sales tax
C
Federal grant money
D
Property taxes
A

Explanation:
Imposed nonexchange revenues represent assessments imposed on non-governmental entities and include property taxes and fines or forfeitures. Taxes on earnings or consumption are classified as derived tax revenues. Certain grants, entitlements, and donations are classified as voluntary nonexchange transactions. Voluntary nonexchange transactions result from legislative or contractual agreements involving parties that may or may not be governmental entities, but do not involve an exchange of equal value.

336
Q
An unrestricted grant received from another government to support enterprise fund operations should be reported as
A
Contributed capital.
B
Nonoperating revenues.
C
Operating revenues.
D
Revenues and expenditures.
A

Explanation:
Interest income, interest expense, gain or loss on sales of capital assets, grants to or from other governments, operating transfers to or from other funds, and residual equity transfers are examples of transactions that are reported as nonoperating revenue.

337
Q

Which of the following describes the reporting for depreciation in the governmental statement of activities?
A
Depreciation is only reported for business-type activities.
B
Depreciation is reported separately in the business-type activities.
C
Governmental activities only report depreciation for infrastructure assets.
D
Depreciation is included with each department’s program expenses for all activities.

A

D maybe

Explanation:
Depreciation expense is reported with the appropriate program when associated capital assets are clearly associated with specific programs. Governmental activities will report depreciation for all activities unless the modified approach is used for infrastructure; however, depreciation is still reported for all other assets.

338
Q

Financial reports of governments should allow users to do which of the following?

A
Evaluate operating results and evaluate the level of service provided
B
Determine return on capital
C
Assess the government’s ability to generate profit
D
Calculate the per capita share of capital assets

A

Explanation:
Financial reports of governments should allow users to evaluate operating results and evaluate the level of service provided. Governments are not funded with “owner equity,” so the return on capital is not a valid measure for most governmental services. Governments do not generally try to generate a profit like commercial entities.Capital assets are held by the governmental entity, but per capita“shares”are not a normal financial ratio.

339
Q

The measurement focus of governmental fund accounting is on which of the following?

A
Current financial resources
B
Economic resources
C
Cash
D
Working capital
A

Explanation:
The correct answer is (a). Governmental funds measurement focus is current financial resources approach and basis of accounting is modified accrual accounting Option (b) is incorrect because economic resources as a measurement focus is applicable to government-wide F/S. Option (c) and (d) are incorrect as per the above explanation.

340
Q

Which of the following factors influences governmental generally accepted accounting and reporting principles?
A
The lack of SEC oversight for municipal financial instruments
B
State statutes that created the Governmental Accounting Standards Board (GASB)
C
Governmental structure and its mission to provide critical public services
D
Population levels for individual governments

A

C

Explanation:
The essential mission of government is delivering services, financed with taxes. The SEC does have limited oversight for tax-exempt financial instruments, but the GASB is focused on the operating and organizational characteristics to define governmental accounting and reporting standards. The GASB was created by the Financial Accounting Foundation and is not subject to any individual state regulation. While the GASB has sometimes considered “small GAAP” versus “big GAAP,” the size of individual state and local governments is not a key factor for the Conceptual Framework.

341
Q

For governmental external financial reporting, Management’s Discussion and Analysis (MD&A) should provide which of the following?
A
Management’s perspective on the government’s significant financial transactions
B
Management’s plans for the immediate subsequent year
C
Management’s long-term capital improvement plan
D
All of the above

A

Explanation:
The MD&A allows management to highlight significant issues and discuss key issues from the financial statements. GASB requirements for the MD&A do not require management to make projections for future years. The section on capital assets only discusses current year projects and changes in capital assets.

342
Q
A city government would report each of the following categories in its government-wide statement of net position except
A
Governmental activities
B
Business-type activities
C
Fiduciary activities
D
Component units
A

Explanation:
The focus of fiduciary funds is resources that are restricted to third parties and cannot be used for any general programs of the primary government. This emphasis carries through the financial statements where the fiduciary funds are included in fund financial statements but not in the government-wide statements. Governmental activities, business-type activities, and component units would all appear in the government-wide statement of net position.

343
Q

Which of the following is a financial statement that would be found in the annual report of a government with only governmental funds?

A
Statement of revenues, expenses, and changes in fund net position
B
Statement of cash flows
C
Statement of revenues, expenditures, and changes in fund balance
D
None of the above
A

Explanation:
The only answer choice listed that relates to governmental funds is the statement of revenues, expenditures, and changes in fund balance. The first two options do not apply to governmental funds. Remember that at the fund level, governmental funds report expenditures rather than expenses and fund balance rather than net position. Governmental funds do not provide a statement of cash flows.

344
Q

The debt service fund of a governmental unit is used to account for the accumulation of resources for, and the payment of, principal and interest in connection with a
Private-purpose trust fund Proprietary funds
A No No
B No Yes
C Yes Yes
D Yes No

A

A

Explanation:
Only general obligation long-term debt should be serviced through debt service funds. Fiduciary and proprietary fund debt are rarely general government obligations.

345
Q

A government’s police department reports appropriations of $10,000, encumbrances of $2,000, and expenditures of $5,000. What is the amount of available appropriations for the police department?

A
$3,000
B
$5,000
C
$7,000
D
$8,000
A

Explanation:
The correct answer is (A).

Available appropriations can be calculated as follows:

Appropriations $10,0000
Encumbrances $(2,000)
Expenditures $(5,000)
Balance $3,000

346
Q
It is inappropriate to record depreciation expense in a(n)
A
Enterprise fund.
B
Internal service fund.
C
Nonexpendable trust fund.
D
Capital projects fund.
A

Explanation:
Capital project funds are designed only for construction in progress and have a limited life encompassing the construction period. Depreciation expense is not recorded during construction.

347
Q

The basic financial statements of a state government that has adopted GASB Statement No. 34, Basic Financial Statements and Management’s Discussion and Analysis for State and Local Governments
A
Are comprised of the government-wide financial statements and related notes.
B
Are comprised of the primary government funds’ financial statements and related notes.
C
Contain more detailed information regarding the state government’s finances than is contained in the comprehensive annual financial report.
D
Do not include management’s discussion and analysis (MD&A).

A

Explanation:
Management’s discussion and analysis (MD&A) is classified as required supplementary information, not basic financial statements. The MD&A is a component of the minimum reporting requirements for state and local governments. The basic financial statements consist of both government-wide and fund financial statements along with the accompanying notes. A comprehensive annual financial report (CAFR) includes the basic fiancial statements.

348
Q

A local municipality had the following account balances and information related to interest on its bonds payable. What amount of interest expense would be reported its statement of activities for the year ended December 31, Year 2?

$50,000 accrued interest on December 31, Year 1
$250,000 in interest payments made during Year 2
$47,000 accrued interest on December 31, Year 2
A
$253,000
B
$250,000
C
$247,000
D
$0

A

Explanation:
Because the statement of activities in a government-wide financial statement, it uses the full accrual method of accounting. This means that the change in interest accrual will affect the expense reported at the government-wide level. The reduction of $3,000 in accrued interest would reduce the amount of interest expense from the amount paid of $250,000 to $247,000.

349
Q

Which of the following are the minimum two primary columns/sections that the government-wide statements are separated into?
A
Governmental activities and business-type activities
B
Governmental activities and component unit activities
C
Business-type activities and trust fund activities
D
Business-type activities and component unit activities

A

Explanation:
The government-wide statements are separated into at least two primary columns/sections: governmental activities are reported in one column, and businesstype activities are reported in another column on the statement of net position and in two different sections of the statement of net position. Additional columns/sections are necessary if the primary government has discretely presented component units. Trust fund activities are not one of the two primary sections.

350
Q

In preparing combined financial statements for a governmental entity, interfund receivables and payables should be
A
Reported as reservations of fund balance.
B
Reported as additions to or reductions from the unassigned fund balance.
C
Reported as amounts due to and due from other funds.
D
Eliminated.

A

Explanation:
Interfund receivables and payables are reported as amounts due to and due from other funds. They are not reported as reservations of fund balance or additions to or reductions from the unassigned fund balance. The option of eliminating the interfund assets and liabilities is allowed, but requires that such eliminations be apparent from the headings, or be disclosed in the notes to the financial statements.